Abdomen: Gastrointestinal and Genitourinary I and II Flashcards

1
Q

Which of the following radiographic signs relates to a pneumoperitoneum?

a. Poppyseed calcifications
b. Football sign
c. Bird-beak appearance
d. Double track sign

A

b. Football sign

How well did you know this?
1
Not at all
2
3
4
5
Perfectly
2
Q

Which of the following statements is correct regarding the renal disease characterized by the presence of grossly elongated, dilated collecting ducts throughout the renal parenchyma which resemble a cluster of straws when seen on end?

a. ureteral strictures are a common associated complication
b. the surface of the kidney remains smooth
c. the disease develops secondarily to a pyelonephritic infection
d. associated changes may include cystic involvement of the liver

A

d. associated changes may include cystic involvement of the liver

How well did you know this?
1
Not at all
2
3
4
5
Perfectly
3
Q

A supine KUB of a patient with gastric ulcer perforation demonstrates visualization of both the intraluminal and peritoneal surfaces of the bowel wall. Which of the following signs describes this appearance?

a. Football sign
b. Kirklin sign
c. String sign
d. Rigler’s sign

A

d. Rigler’s sign

How well did you know this?
1
Not at all
2
3
4
5
Perfectly
4
Q

Reflux of colon contents through the ileocecal is abnormal. During a lower GI study, it is normal to see contrast material in the ileum. What is the frequency of retro flow of contrast into the ileum during a lower GI study.

a. 10%
b. 40%
c. 70%
d. 90%

A

d. 90%

How well did you know this?
1
Not at all
2
3
4
5
Perfectly
5
Q

A thin, sharply demarcated lucent line with parallel straight margins at the base of an ulcer crater is called?

a. ulcer colla
b. Carmen’s line
c. Hampton line
d. Cresent sign

A

c. Hampton line

How well did you know this?
1
Not at all
2
3
4
5
Perfectly
6
Q

Following renal trauma, which imaging techniques offers physiologic information on renal function?

a. doppler ultrasound
b. retrograde pylogram
c. radionuclide urogram
d. abdominal CT

A

c. radionuclide urogram

How well did you know this?
1
Not at all
2
3
4
5
Perfectly
7
Q

Bowel obstruction during the immediate post-natal period, termed meconium ileus, is related to which of the following disorders?

a. Systemic lupus erythematosis
b. Inspissated milk syndrome
c. Cystic fibrosis
d. Hirsprong’s disease

A

c. Cystic fibrosis

How well did you know this?
1
Not at all
2
3
4
5
Perfectly
8
Q

The most common portion of the colon to be affected by adenocarcinoma is?

a. rectosigmond region
b. descending colon
c. transverse colon
d. ascending colon

A

a. rectosigmond region

How well did you know this?
1
Not at all
2
3
4
5
Perfectly
9
Q

What is the common benign ureteral tumor?

a. fibroepithelial polyp
b. leioyoma
c. papilloma
d. hemangioma

A

c. papilloma

How well did you know this?
1
Not at all
2
3
4
5
Perfectly
10
Q

Zenker’s divercilulum can be found where?

a. Superior-posterior esophagus
b. Superior-anterior esophagus
c. Mesenteric side of ileum
d. Anti-mesenteric side of ileum

A

a. Superior-posterior esophagus

How well did you know this?
1
Not at all
2
3
4
5
Perfectly
11
Q

Which of the following special imaging techniques is best utilized for the diagnosis of a cavernous hemangioma of the liver?

a. Spiral computed tomography with the administration of intravenous contrast and images taken in a hepatic arterial phase and a portal venous phase.
b. Catheterized administration of contrast into the SMA with computed tomography.
c. Magnetic resonance imaging of the liver utilizing T1 and T2 weighted images.
d. Radionuclide imaging of the liver.

A

a. Spiral computed tomography with the administration of intravenous contrast and images taken in a hepatic arterial phase and a portal venous phase.

How well did you know this?
1
Not at all
2
3
4
5
Perfectly
12
Q

Regarding pneumatosis intestinalis, which is false?

a. may be caused by bowel necrosis, ulceration or increased membrane permeability
b. findings include air bubbles in bowel wall on CT
c. it is not considered benign, even when asymptomatic
d. can be related to pulmonary origin

A

c. it is not considered benign, even when asymptomatic

How well did you know this?
1
Not at all
2
3
4
5
Perfectly
13
Q

Which of the following is not a cause of adynamic ileus?

a. Surgical manipulation
b. Electrolyte imbalance
c. Renal stones
d. Crohn’s disease

A

d. Crohn’s disease

How well did you know this?
1
Not at all
2
3
4
5
Perfectly
14
Q

A large, atonic bladder with little or no trabeculation, producing either incontinence or retention or urine is suggestive of…

a. Bladder diverticulum
b. Vesicoureteral reflux
c. Neurogenic bladder
d. Megacystica syndrome

A

c. Neurogenic bladder

How well did you know this?
1
Not at all
2
3
4
5
Perfectly
15
Q

Some consider crossed renal ectopia a variant of the horseshoe kidney. The ectopic kidney lies above the pelvis but on the wrong side. What frequency of the ectopic kidney being fused with the normal kidney?

a. never
b. 30%
c. 60%
d. 90%

A

d. 90%

How well did you know this?
1
Not at all
2
3
4
5
Perfectly
16
Q

It is thought that upper urinary tract calculi originate as ________ deep in the lining of the collecting ducts in the renal papillae which may detach and pass into the renal collecting system.

a. areas of nodular heterotopia
b. areas of acute focal pyelonephritis
c. Brucellosis infectious foci
d. Randall’s plaques

A

d. Randall’s plaques

How well did you know this?
1
Not at all
2
3
4
5
Perfectly
17
Q

A “cloverleaf deformity” is associated with:

a. tuberculosis
b. Crohn’s disease
c. Zollinger-Ellison disease
d. duodenal ulcer disease

A

d. duodenal ulcer disease

How well did you know this?
1
Not at all
2
3
4
5
Perfectly
18
Q

Which of the following is associated with the drooping lily appearance? a. ureteral duplication b. bifid renal pelvis c. ureteropelvic junction obstruction d. retrocaval ureter

A

a. ureteral duplication

How well did you know this?
1
Not at all
2
3
4
5
Perfectly
19
Q

What is a common location for gastric diverticulum to occur?

a. body
b. fundus
c. cardia
d. antrum

A

b. fundus

How well did you know this?
1
Not at all
2
3
4
5
Perfectly
20
Q

Complete loss the mucosal folds in the small intestine, seen in advanced sprue, is called what?

a. String sign
b. Mexican hat sign
c. Steppladder appearance
d. Moulage sign

A

d. Moulage sign

How well did you know this?
1
Not at all
2
3
4
5
Perfectly
21
Q

What is the most cause of large bowel obstrution?

a. Fecal impaction
b. Ischemia
c. Primary malignancies of the colon
d. Metastatic disease

A

c. Primary malignancies of the colon

How well did you know this?
1
Not at all
2
3
4
5
Perfectly
22
Q

Single or multiple deep ulcers of the esophagus associated with sinus tracts and fistula formation within the wall of the esophagus and extending to the mediastinum are common are characteristic of:

a. tuberculous esophagitis
b. herpes esophagitis
c. cytomegalovirus esophagitis
d. candida esophagitis

A

a. tuberculous esophagitis

How well did you know this?
1
Not at all
2
3
4
5
Perfectly
23
Q

Which characteristic listed below is not associated with ulcerative colitis?

a. starts distally and moves proximally through the colon
b. rarely involves the terminal ileum
c. eccentric disease
d. risk of toxic megacolon

A

c. eccentric disease

How well did you know this?
1
Not at all
2
3
4
5
Perfectly
24
Q

Dilated veins in the subepithelial connective tissue of the esophagus are most commonly a result of:

a. obstruction of the portal or splenic veins by carcinoma of the panceras
b. pancreatitis
c. slow-flow states (e.g. polycythemia)
d. portal hypertension

A

d. portal hypertension

How well did you know this?
1
Not at all
2
3
4
5
Perfectly
25
Q

Calcification of a fetus without involvement of the membranes is called….

a. Still birth rocky syndrome
b. Lithopedion
c. Dish baby syndrome
d. Ankylosing fetal syndrome

A

b. Lithopedion

How well did you know this?
1
Not at all
2
3
4
5
Perfectly
26
Q

Which of the following types of diverticuli arise in a relatively weak area on the left anterolateral wall of the esophagus between the inferior border of the aortic arch and the upper external margin of the left main bronchus and are only demonstrated on a right anterior oblique projection?

a. pulsion diverticuli
b. traction diverticuli
c. epiphrenic diverticuli
d. lateral diverticuli

A

a. pulsion diverticuli

How well did you know this?
1
Not at all
2
3
4
5
Perfectly
27
Q

Retrocardiac calcification in a retrocardiac esophageal mass is diagnostic of:

a. rhabdomyoma
b. adenomatous polyp
c. leiomyoma
d. squamous papilloma

A

c. leiomyoma

How well did you know this?
1
Not at all
2
3
4
5
Perfectly
28
Q

Dsyphagia in a person older than 40 years of age must be assumed to be ______ until proven otherwise?

a. Prebyesophagus
b. Esophageal webs
c. Carcinoma
d. Infectious/inflammatory esophagitis

A

c. Carcinoma

How well did you know this?
1
Not at all
2
3
4
5
Perfectly
29
Q

Rupture of berry aneurysms of the arteries at the base of the brain is associated with which renal cystic disease?

a. autosomal recessive polycystic kidney disease
b. autosomal dominant polycystic kidney disease
c. medullary sponge kidney
d. multicystic dysplastic kidney

A

b. autosomal dominant polycystic kidney disease

How well did you know this?
1
Not at all
2
3
4
5
Perfectly
30
Q

A kidney that crosses the midline is known as a(n):

a. ectopic kidney
b. subdiaphragmatic kidney
c. pelvic kidney
d. kidney malrotation

A

a. ectopic kidney

How well did you know this?
1
Not at all
2
3
4
5
Perfectly
31
Q

All of the following are causes of nephrocalcinosis except…

a. renal tubular acidosis
b. sarcoidosis
c. hypothyroidism
d. Wilson’s disease

A

c. hypothyroidism

How well did you know this?
1
Not at all
2
3
4
5
Perfectly
32
Q

Patients with massive localized rugal thickening along with hypoproteinemia and intestinal edema will often demonstrate which lab findings?

a. positive carcinoembryonic antigen and anemia
b. increased alkaline phosphatase and anemia
c. eosinophilia and anemia
d. positive blood cultures

A

c. eosinophilia and anemia

How well did you know this?
1
Not at all
2
3
4
5
Perfectly
33
Q

The “double bubble” sign relates to which of the following?

a. Duodenal atresia
b. Pneumoperitoneum
c. Sigmoid volvulus
d. Pyloric stenosis

A

a. Duodenal atresia

How well did you know this?
1
Not at all
2
3
4
5
Perfectly
34
Q

Which of the following characteristics is not associated with Crohn disease of the colon?

a. pseudodiverticula formation
b. fistula formation
c. rectal sparing
d. collar button ulcers

A

d. collar button ulcers

How well did you know this?
1
Not at all
2
3
4
5
Perfectly
35
Q

Which of the following is associated with bilateral engagement of the kidneys?

a. medullary sponge kidney
b. multicystic dyplastic kidney
c. autosomal recessive polycystic disease
d. none of the above

A

c. autosomal recessive polycystic disease

How well did you know this?
1
Not at all
2
3
4
5
Perfectly
36
Q

What is the most common solid mesenteric mass?

a. lymphoma
b. metastases
c. GIST
d. mesenteric desmoid

A

a. lymphoma

How well did you know this?
1
Not at all
2
3
4
5
Perfectly
37
Q

What is the most likely location to visualize an abdominal abscess?

A. Subphrenic space

B. Subhepatic space

C. Lesser sac

D. Pelvis

A

D. Pelvis

Brant & Helms Fundamental of Diagnostic Radiology p.663

How well did you know this?
1
Not at all
2
3
4
5
Perfectly
38
Q

A conventional radiographic examination of the abdomen demonstrates multiple mottled calcifications visualized on either side of the first lumbar vertebra. Which of the following should not be included in your differential diagnosis?

a. Chronic pancreatitis
b. Addison’s disease
c. Hydatid disease of the suprarenals
d. Nephrocalcinosis

A

C. Hydatid disease of the suprarenals

Brant & Helms Fundamental of Diagnostic Radiology p.662

How well did you know this?
1
Not at all
2
3
4
5
Perfectly
39
Q

Which of the following conventional radiographic examinations should be followed when a patient presents with an “acute abdomen”?

a. Erect PA chest, lateral decubitus chest, erect KUB
b. Erect PA chest, lateral decubitus chest, supine KUB
c. Erect PA chest, supine KUB, erect KUB
d. Erect PA chest, supine KUB, lateral decubitus abdomen

A

C. Erect PA chest, supine KUB, erect KUB

Brant & Helms Fundamental of Diagnostic Radiology p.656

How well did you know this?
1
Not at all
2
3
4
5
Perfectly
40
Q

What is the most common cause of toxic megacolon?

a. Pseudomenbranous colitis
b. Chron’s colitis
c. Ulcerative colitis
d. Ischemic colitis

A

C. Ulcerative colitis

Brant & Helms Fundamental of Diagnostic Radiology p.658

How well did you know this?
1
Not at all
2
3
4
5
Perfectly
41
Q

Which of the following represents an adrenal cortical tumor?

a. neuroblastoma
b. pheochromocytoma
c. adenoma
d. hemangioblastoma

A

C. adenoma

P&J 720

How well did you know this?
1
Not at all
2
3
4
5
Perfectly
42
Q

The most common cause of calcification of the adrenal glands is believed to be…

a. Dystrophism
b. Metabolic
c. Congenital
d. Hemorrhagic

A

d. Hemorrhagic

P&J

How well did you know this?
1
Not at all
2
3
4
5
Perfectly
43
Q

Which of the following is not true concerning adrenogenital syndrome?

a. usually occurs in newborns and infants who have an enzyme deficiency
b. associated with deficient production of cortisol and aldosterone and excess androgens
c. infants have adrenal hypoplasia masculinizing and feminizing syndromes
d. in older patients may be associated with adrenal neoplasm

A

c. infants have adrenal hypoplasia

b+h 3rd pg 868 infants have adrenal hyperplasia

How well did you know this?
1
Not at all
2
3
4
5
Perfectly
44
Q

Which of the following is not true concerning Addison disease?

a. requires destruction of only 10% of the adrenal cortex
b. most common cause in the U.S. is idiopathic
c. atrophy cases with adrenal calcification suggests prior tuberculosis or histoplasmosis
d. cases with adrenal enlargement suggest active infection

A

a. requires destruction of only 10% of the adrenal cortex

b+h 3rd pg 868 requires 90% destruction

How well did you know this?
1
Not at all
2
3
4
5
Perfectly
45
Q

Which of the following is not part of the rule of tens for pheochromocytoma?

a. 10% are unilateral
b. 10% are extra-adrenal
c. 10% are malignant
d. 10% are familial

A

a. 10% are unilateral

b+h 3rd pg b+h 3rd pg 868 10% are bilateral

How well did you know this?
1
Not at all
2
3
4
5
Perfectly
46
Q

Which of the following is not true concerning pheochromocytoma?

a. most arise in the adrenal cortex
b. most are larger than 2 cm in diameter
c. it is the most common adrenal tumor to hemorrhage spontaneously
d. extra-adrenal sites include the organ of Zuckerkandl, the bladder and para-aortic sympathetic chain

A

a. most arise in the adrenal cortex

b+h 3rd pg 868-9 most arise in the adrenal medulla

How well did you know this?
1
Not at all
2
3
4
5
Perfectly
47
Q

Which of the following is true concerning adrenal hyperplasia?

a. causes 70% of cases of non-iatrogenic Cushing syndrome
b. causes 20% of cases of Conn syndrome
c. may show adrenal limbs longer than 5 cm that exceed 10 mm in thickness
d. all of the above are true

A

d. all of the above are true

b+h 3rd pg 869

How well did you know this?
1
Not at all
2
3
4
5
Perfectly
48
Q

Which of the following is false concerning adrenal adenomas?

a. nonhyperfunctional adenomas found incidentally in 3-5% of the population
b. tend to be low in density on CT
c. tend to appear hypointense on T1 and hyperintense to liver on T2 compared to the liver
d. none of the above are false

A

d. none of the above are false

b+h 3rd pg 869

How well did you know this?
1
Not at all
2
3
4
5
Perfectly
49
Q

What is the most likely location to visualize an abdominal abscess?

  1. Subphrenic space
  2. Subhepatic space
  3. Lesser sac
  4. Pelvis
A

D. Pelvis

Brant & Helms Fundamental of Diagnostic Radiology p.663

How well did you know this?
1
Not at all
2
3
4
5
Perfectly
50
Q

A conventional radiographic examination of the abdomen demonstrates multiple mottled calcifications visualized on either side of the first lumbar vertebra. Which of the following should not be included in your differential diagnosis?

  1. Chronic pancreatitis
  2. Addison’s disease
  3. Hydatid disease of the suprarenals
  4. Nephrocalcinosis
A

C. Hydatid disease of the suprarenals

Brant & Helms Fundamental of Diagnostic Radiology p.662

How well did you know this?
1
Not at all
2
3
4
5
Perfectly
51
Q

Which of the following conventional radiographic examinations should be followed when a patient presents with an “acute abdomen”?

  1. Erect PA chest, lateral decubitus chest, erect KUB
  2. Erect PA chest, lateral decubitus chest, supine KUB
  3. Erect PA chest, supine KUB, erect KUB
  4. Erect PA chest, supine KUB, lateral decubitus abdomen
A

C. Erect PA chest, supine KUB, erect KUB

Brant & Helms Fundamental of Diagnostic Radiology p.656

How well did you know this?
1
Not at all
2
3
4
5
Perfectly
52
Q

What is the most common cause of toxic megacolon?

  1. Pseudomenbranous colitis
  2. Chron’s colitis
  3. Ulcerative colitis
  4. Ischemic colitis
A

C. Ulcerative colitis

Brant & Helms Fundamental of Diagnostic Radiology p.658

How well did you know this?
1
Not at all
2
3
4
5
Perfectly
53
Q

Which of the following represents an adrenal cortical tumor?

  1. neuroblastoma
  2. pheochromocytoma
  3. adenoma
  4. hemangioblastoma
A

C. adenoma

P&J 720

How well did you know this?
1
Not at all
2
3
4
5
Perfectly
54
Q

The most common cause of calcification of the adrenal glands is believed to be…

  1. Dystrophism
  2. Metabolic
  3. Congenital
  4. Hemorrhagic
A

D. Hemorrhagic

P&J

How well did you know this?
1
Not at all
2
3
4
5
Perfectly
55
Q

Which of the following is not true concerning adrenogenital syndrome?

  1. usually occurs in newborns and infants who have an enzyme deficiency
  2. associated with deficient production of cortisol and aldosterone and excess androgens
  3. infants have adrenal hypoplasia
  4. masculinizing and feminizing syndromes in older patients may be associated with adrenal neoplasm
A

C. Infants have adrenal hypoplasia

b+h 3rd pg 868
infants have adrenal hyperplasia

How well did you know this?
1
Not at all
2
3
4
5
Perfectly
56
Q

Which of the following is not true concerning Addison’s disease?

  1. requires destruction of only 10% of the adrenal cortex
  2. most common cause in the U.S. is idiopathic atrophy
  3. cases with adrenal calcification suggest prior tuberculosis or histoplasmosis
  4. cases with adrenal enlargement suggest active infection
A

A. requires destruction of only 10% of the adrenal cortex

b+h 3rd pg 868
requires 90% destruction

How well did you know this?
1
Not at all
2
3
4
5
Perfectly
57
Q

Which of the following is not part of the rule of tens for pheochromocytoma?

  1. 10% are unilateral
  2. 10% are extra-adrenal
  3. 10% are malignant
  4. 10% are familial
A

A. 10% are unilateral

b+h 3rd pg b+h 3rd pg 868
10% are bilateral

How well did you know this?
1
Not at all
2
3
4
5
Perfectly
58
Q

Which of the following is not true concerning pheochromocytoma?

  1. most arise in the adrenal cortex
  2. most are larger than 2 cm in diameter
  3. it is the most common adrenal tumor to hemorrhage spontaneously
  4. extra-adrenal sites include the organ of Zuckerkandl, the bladder and para-aortic sympathetic chain
A

A. most arise in the adrenal cortex

b+h 3rd pg 868-9
most arise in the adrenal medulla

How well did you know this?
1
Not at all
2
3
4
5
Perfectly
59
Q

Which of the following is true concerning adrenal hyperplasia?

  1. causes 70% of cases of noniatrogenic Cushing syndrome
  2. causes 20% of cases of Conn syndrome
  3. may show adrenal limbs longer than 5 cm that exceed 10 mm in thickness
  4. all of the above are true
A

D. all of the above are true

b+h 3rd pg 869

How well did you know this?
1
Not at all
2
3
4
5
Perfectly
60
Q

Which of the following is false concerning adrenal adenomas?

  1. non-hyperfunctional adenomas found incidentally in 3-5% of the population
  2. tend to be low in density on CT
  3. tend to appear hypointense on T1 and hyperintense to liver on T2 compared to the liver
  4. none of the above are false
A

D. none of the above are false

b+h 3rd pg 869

How well did you know this?
1
Not at all
2
3
4
5
Perfectly
61
Q

What is the most likely cause of an adrenal mass found incidentally?

  1. benign non-hyperfunctioning adenoma
  2. adenocarcinoma
  3. adrenal myelolipoma
  4. adrenal cyst
A

A. benign non-hyperfunctioning adenoma

b+h 3rd pg 869

How well did you know this?
1
Not at all
2
3
4
5
Perfectly
62
Q

Which of the following is false concerning adrenal myelolipomas?

  1. composed of bone marrow elements
  2. have 50% risk of malignant degeneration
  3. calcifications are present in 20% of cases
  4. range in size up to 30 cm
A

B. have 50% risk of malignant degeneration​
b+h 3rd pg 870
adrenal myelolipomas have no malignant potential

How well did you know this?
1
Not at all
2
3
4
5
Perfectly
63
Q

Which of the following is not true concerning adrenal hemorrhage?

  1. most common in newborn infants
  2. trauma and infection are the most common causes in adults
  3. the left is more commonly affected in unilateral cases
  4. most cases in newborns are bilateral while unilateral cases are more common in adults
A

C. the left is more commonly affected in unilateral cases
b+h 3rd pg 870
right more often affected in unilateral cases

How well did you know this?
1
Not at all
2
3
4
5
Perfectly
64
Q

Which of the following is least likely to cause adrenal calcifications?

  1. adrenal hemorrhage
  2. adrenal myelolipoma
  3. pheochromocytoma
  4. benign adenoma
A

D. benign adenoma
b+h 3rd pg 871

How well did you know this?
1
Not at all
2
3
4
5
Perfectly
65
Q

Which of the following is not true concerning adrenal cysts?

  1. adrenal cysts are more common in men
  2. paracytic cysts are usually echinococcal in origin
  3. endothelial cysts tend to be multilocular with septal calcification
  4. hemorrhagic pseudocysts tend to be unilocular with wall calcification
  5. all of the above are true
A

A. adrenal cysts are more common in men
b+h 3rd pg 871
more common in women

How well did you know this?
1
Not at all
2
3
4
5
Perfectly
66
Q

What percentage of patients with malignant disease show adrenal metastases at autopsy?

  1. 3%
  2. 11%
  3. 27%
  4. 42%
A

C. 27%
b+h 3rd pg 871

How well did you know this?
1
Not at all
2
3
4
5
Perfectly
67
Q

Up to 50% of small adrenal masses in patients with a known primary malignancy are benign adenomas.

A. True

B. False

A

A. True

b+h 3rd pg 871

How well did you know this?
1
Not at all
2
3
4
5
Perfectly
68
Q

Which of the following is not true concerning adrenal carcinoma?

  1. most are large and invasive at presentation
  2. 50% are hyperfunctions and cause endocrine syndromes
  3. calcification is present in 30%
  4. signal intensity is hypointense relative to the liver on T2 images
A

D. signal intensity is hypointense relative to the liver on T2 images

b+h 3rd pg 871-2
signal is markedly hyperintense relative to liver on T2 images

How well did you know this?
1
Not at all
2
3
4
5
Perfectly
69
Q

Conn’s syndrome is associated with:

  1. a unilateral adrenal adenoma
  2. hyperplastic adrenals
  3. adrenal carcinoma
  4. ganglioneuroma
A

A. a unilateral adrenal adenoma
P&J pg. 720

How well did you know this?
1
Not at all
2
3
4
5
Perfectly
70
Q

Elevated levels of urine catecholamines are associated with:

  1. neuroblastoma
  2. pheochromocytoma
  3. ganglioneuroblastoma
  4. hemangioma
A

B. pheochromocytoma
P&J pg. 721

How well did you know this?
1
Not at all
2
3
4
5
Perfectly
71
Q

Which one of the following will most likely appear as an abdominal mass in a neonate?

  1. appendix abscess
  2. hemangioma
  3. teratoma
  4. neuroblastoma
A

D. neuroblastoma
Chapman 3rd p.198

How well did you know this?
1
Not at all
2
3
4
5
Perfectly
72
Q

Calcified adrenal glands may be secondary to which of the following…

  1. Dermoid cyst
  2. Cystic fibrosis
  3. Cushing’s disease
  4. Addison’s disease
A

D. Addison’s disease
P&J

How well did you know this?
1
Not at all
2
3
4
5
Perfectly
73
Q

Which of the following is not true concerning Cushing syndrome?

  1. caused by excessive amounts of aldosterone
  2. adrenal hyperplasia causes 70% of noniatrogenic cases
  3. 90% of hyperplasia cases are associated with microadenoma of the pituitary gland
  4. benign adrenal adenomas are responsible for 20% of noniatrogenic cases
A

A. caused by excessive amounts of aldosterone

b+h 3rd pg 868
caused by excessive amounts of hydrocortisone and corticosterone
remaining 10% of noniatrogenic cases caused by adrenal carcinoma
remaining 10% of hyperplasia cases caused by ectopic source of ACTH, usually lung malignancy

How well did you know this?
1
Not at all
2
3
4
5
Perfectly
74
Q

Which of the following is true concerning Conn syndrome?

  1. accounts for 50% of systemic hypertension cases
  2. hyperfunctioning adrenal adenoma is the cause of 20% of cases
  3. 80% of cases are caused by adrenal hyperplasia
  4. adenomas that produce Conn syndrome tend to be less than 2 cm in size
A

D. adenomas that produce Conn syndrome tend to be less than 2 cm in size
b+h 3rd pg 868
accounts for 1 to 2% of system hypertension
hyperfunctioning adenoma causes 80% of cases
adrenal hyperplasia account for 20% of cases

How well did you know this?
1
Not at all
2
3
4
5
Perfectly
75
Q

What is the most dependent portion of the abdominal cavity in a supine patient?

  1. Douglas’ pouch
  2. Morrison’s pouch
  3. Rectouterine pouch
  4. Rectovesicle pouch
A

B. Morrison’s pouch
Brant & Helms Fundamentals of Diagnostic Radiology p.651

How well did you know this?
1
Not at all
2
3
4
5
Perfectly
76
Q

Which of the following is a true statement regarding the compartmental anatomy of the
abdomen?

  1. There is constant communication between the right subphrenic space, right subhepatic space, right paracolic gutter, and Douglas’ pouch.
  2. There is constant communication between the left subphrenic space, left subhepatic space, left paracolic gutter, and Douglas’ pouch.
  3. There is constant communication between the right subphrenic space, right subhepatic space, left subhepatic space, left subphrenic space and Morrison’s pouch.
  4. There is constant communication between the left and right subphrenic spaces.
A

A. There is constant communication between the right subphrenic space, right subhepatic space, right paracolic gutter, and Douglas’ pouch​.

Brant & Helms Fundamentals of Diagnostic Radiology p.651

How well did you know this?
1
Not at all
2
3
4
5
Perfectly
77
Q

Which of the following is not true concerning acute appendicitis?

  1. it is the most common cause of acute abdomen
  2. plain film shows appendiceal calculus in 14% of patients
  3. non-filling of the appendix is diagnostic for appendicitis
  4. appendix considered dilated when greater than 6 mm in diameter
A

C. non-filling of the appendix is diagnostic for appendicitis​

b+h 3rd pg 861
non-filling of the appendix is of no diagnostic value

How well did you know this?
1
Not at all
2
3
4
5
Perfectly
78
Q

Which of the following is not true concerning appendiceal mucoceles?

  1. contain sterile mucus
  2. rupture may result in pseudomyxoma peritonei
  3. peripheral calcification may be present
  4. all of the above are true
A

D. all of the above are true

b+h 3rd pg 862

How well did you know this?
1
Not at all
2
3
4
5
Perfectly
79
Q

What is the most common tumor of the appendix?

  1. carcinoid
  2. adenocarcinoma
  3. adenoma
  4. mucinous cystadenoma
A

A. carcinoid

b+h 3rd pg 862
85% of appendiceal tumors

How well did you know this?
1
Not at all
2
3
4
5
Perfectly
80
Q

What is the most common location for carcinoid tumors?

  1. lung
  2. small bowel
  3. large bowel
  4. appendix
A

D. appendix

b+h 3rd pg 862
appendix accounts for 60% of all carcinoids

How well did you know this?
1
Not at all
2
3
4
5
Perfectly
81
Q

Appendiceal adenomas are usually associated with familial multiple polyposes:

A. True

B. False

A

A. True

b+h 3rd pg 863

How well did you know this?
1
Not at all
2
3
4
5
Perfectly
82
Q

Which of the following is not true concerning biliary duct dilatation?

  1. normal intrahepatic ducts should not exceed 40% of diameter of adjacent portal vein
  2. peripheral intrahepatic ducts are considered dilated when greater than 1.8mm in diameter
  3. central intrahepatic ducts are considered dilated when greater than 3 mm in diameter
  4. common bile duct is considered dilated when greater than 7mm
A

C. central intrahepatic ducts are considered dilated when greater than 3 mm in diameter​

b+h 3rd pg 772
central ihbd’s dilated when greater than 2mm

How well did you know this?
1
Not at all
2
3
4
5
Perfectly
83
Q

Which is correct concerning biliary obstruction?

  1. gradual tapering of dilated common duct suggest malignant process
  2. abrupt termination of dilated common duct suggests benign stricture
  3. benign disease is responsible for 75% of cases of obstructive jaundice
  4. infected bile is present in 80% of complete biliary obstruction
A

C. benign disease is responsible for 75% of cases of obstructive jaundice

b+h 3rd pg 772
gradual tapering suggests a benign process; abrupt termination suggests malignant process; 10% of complete obstruction have infected bile

How well did you know this?
1
Not at all
2
3
4
5
Perfectly
84
Q

Which is not true concerning choledocholithiasis?

  1. responsible for 20% of cases of obstructive jaundice
  2. 1 to 3% of cases of choledocholithiasis have no stones in the gallbladder
  3. US is the most sensitive exam for stone detection
  4. associated signs on advanced imaging are “target sign” or “crescent sign”
A

C. US is the most sensitive exam for stone detection

b+h 3rd pg 772-3
mrcp and contrast ct are most sensitive 95-99%
us is 20-80%
noncontrast ct is 70-80%

How well did you know this?
1
Not at all
2
3
4
5
Perfectly
85
Q

Which is NOT true concerning benign stricture of the common bile duct?

  1. accounts for 45% of obstructive jaundice
  2. associated with previous stone passage
  3. duct wall enhances greatly during portal venous phase
  4. can follow radiation therapy to the area
A

C. duct wall enhances greatly during portal venous phase​

b+h 3rd pg 773
enhances minimally if benign
hyperenhancement associated with malignant stricture

How well did you know this?
1
Not at all
2
3
4
5
Perfectly
86
Q

The patient presenting with a history of ulcerative colitis with small saccular outpouchings of the intrahepatic and extrahepatic duct walls most likely has:

  1. primary sclerosing cholangitis
  2. pancreatitis
  3. cholangiocarcinoma
  4. caroli disease
A

A. primary sclerosing cholangitis

b+h 3rd pg 773

How well did you know this?
1
Not at all
2
3
4
5
Perfectly
87
Q

Identify the false statement concerning Caroli’s disease:

  1. characterized by dilation of intrahepatic bile ducts
  2. associated with medullary sponge kidney
  3. associated with “central dot sign”
  4. most cases present in adulthood
A

D. most cases present in adulthood
b+h 3rd pg 774

How well did you know this?
1
Not at all
2
3
4
5
Perfectly
88
Q

Which type (according to Todani classification) of choledochal cyst is the most common?

  1. I
  2. II
  3. III
  4. IV
  5. V
A

A. I

b+h 3rd pg 774
I - fusiform dilation of CHD or CBD
II - diverticula of CHD or CBD with narrow connecting stalk
III - choledochoceles
IV - multifocal dilations of IHBDs and EHBDs
V - Caroli’s disease

How well did you know this?
1
Not at all
2
3
4
5
Perfectly
89
Q

Which is NOT true concerning cholangiocarcinoma?

  1. most are adenocarcinomas arising from bile duct epithelium
  2. second most common malignant primary hepatic tumor
  3. Klatskin tumor is a subtype occurring at the junction of the CBD and CHD
  4. extrahepatic cholangiocarcinoma is the most common subtype
A

C. Klatskin tumor is a subtype occurring at the junction of the CBD and CHD​
b+h 3rd pg 775
klatskin tumor occurs at the junction of right and left hepatic ducts
90% are adenomas
extrahepatic form accounts for 65%

How well did you know this?
1
Not at all
2
3
4
5
Perfectly
90
Q

Cholecystoduodenal fistula is more common in women and caused by a perforating duodenal ulcer:

A. True

B. False

A

B. False

b+h 3rd pg 776

How well did you know this?
1
Not at all
2
3
4
5
Perfectly
91
Q

Gas in the biliary tree may follow sphincterotomy to facilitate stone passage:

A. True

B. False

A

A. True

b+h 3rd pg 776

How well did you know this?
1
Not at all
2
3
4
5
Perfectly
92
Q

What is the most common cause of bladder obstruction in older-aged males?

  1. Prostate carcinoma
  2. Benign prostatic hypertrophy
  3. Urethral valves
  4. Neurogenic dysfunction
A

B. Benign prostatic hypertrophy
P&J

How well did you know this?
1
Not at all
2
3
4
5
Perfectly
93
Q

Carcinoma of the bladder is usually of which type…

  1. Transitional cell
  2. Squamous cell
  3. Epithelial cell
  4. Basal cell
A

A. Transitional cell
P.&J.

How well did you know this?
1
Not at all
2
3
4
5
Perfectly
94
Q

A large, atonic bladder with little or no trabeculation, producing either incontinence or retention of urine is suggestive of…

  1. Bladder diverticulum
  2. Vesicoureteral reflux
  3. Neurogenic bladder
  4. Megacystica syndrome
A

C. Neurogenic bladder
P&J

How well did you know this?
1
Not at all
2
3
4
5
Perfectly
95
Q

Congenital enlargement of the bladder associated with hydronephrosis, hydroureter and
absence or hypoplasia of the abdominal muscles is called…

  1. Bladder ears
  2. Bladder exstrophy
  3. Prune-belly syndrome
  4. Agenetic abdominal syndrome
A

C. Prune-belly syndrome
P&J

How well did you know this?
1
Not at all
2
3
4
5
Perfectly
96
Q

When present, the ureteral jet phenomenon excludes the possibility of…

  1. Retrocaval ureter
  2. Ureterocele
  3. Ureteral diverculum
  4. Vesicoureteral reflux
A

D. Vesicoureteral reflux​
P.&J.

How well did you know this?
1
Not at all
2
3
4
5
Perfectly
97
Q
  • *Severe widening of the symphysis pubis can accompany which of the following bladder
    disorders. ..**
  1. Duplication of the bladder
  2. Exstrophy of the bladder
  3. Agenesis of the bladder
  4. Bladder ears
A

B. exstrophy of the bladder
P.&J.

How well did you know this?
1
Not at all
2
3
4
5
Perfectly
98
Q

Cystitis cystica may progress to show mucous gland proliferation in the bladder wall. This
disease, which may be a precursor of bladder adenocarcinoma, is called:

  1. bullous edema
  2. emphysematous cystitis
  3. hemorrhagic cystitis
  4. cystitis glandularis
A

D. cystitis glandularis
b+h 3rd pg 900

How well did you know this?
1
Not at all
2
3
4
5
Perfectly
99
Q

Which of the following is true concerning the various types of cystitis?

  1. hemorrhagic cystitis is caused by a bacterial or adenovirus infection
  2. interstitial cystitis is a viral infection usually found in men
  3. bullous edema is associated with the use of indwelling catheters
  4. emphysematous cystitis may be seen in cases of poorly controlled diabetes mellitus
A

B. interstitial cystitis is a viral infection usually found in men
b+h 3rd pg 900
interstitial cystitis is an idiopathic inflammation usually found in women

How well did you know this?
1
Not at all
2
3
4
5
Perfectly
100
Q

Concerning calcification of the bladder wall and ureters, which is not true?

  1. schistosomiasis most commonly affects the distal ureters and bladder
  2. tuberculosis most commonly affects the proximal ureters
  3. 25% of bladder carcinomas calcify
  4. cystitis may cause curvilinear or flocculent bladder wall calcification
A

C. 25% of bladder carcinomas calcify
b+h 3rd pg 900-1
only 1-7% of bladder carcinomas calcify

How well did you know this?
1
Not at all
2
3
4
5
Perfectly
101
Q

A cobra head or spring onion appearance is suggestive of what diagnosis in the bladder?

  1. simple ureterocele
  2. ectopic ureterocele
  3. transitional cell carcinoma
  4. bladder stone
A

A. simple ureterocele
b+h 3rd pg 901

How well did you know this?
1
Not at all
2
3
4
5
Perfectly
102
Q

Ectopic ureteroceles are associated with incontinence in:

A. males
B. females

A

B. Females

How well did you know this?
1
Not at all
2
3
4
5
Perfectly
103
Q

What is the most common urinary tract neoplasm?

  1. transitional cell carcinoma of the bladder
  2. transitional cell carcinoma of the ureter
  3. renal cell carcinoma
  4. squamous cell carcinoma of the urethra
A

A. Transitional cell carcinoma of the bladder

b+h 3rd pg 901
tcc of the bladder is 50x more common than tcc of the ureter

How well did you know this?
1
Not at all
2
3
4
5
Perfectly
104
Q

What percentage of transitional cell carcinomas of the bladder have distant metastases at initial
diagnosis?

  1. 5%
  2. 15%
  3. 25%
  4. 40%
A

A. 5%
b+h 3rd pg 901

How well did you know this?
1
Not at all
2
3
4
5
Perfectly
105
Q

What percentage of transitional cell carcinomas of the bladder demonstrate calcifications?

  1. 1%
  2. 5%
  3. 15%
  4. 25%
A

B. 5%
b+h 3rd pg 903

How well did you know this?
1
Not at all
2
3
4
5
Perfectly
106
Q

Which of the following is least associated with squamous cell carcinoma of the bladder?

  1. presence of bladder stones
  2. chronic infection
  3. schistosomiasis
  4. tuberculosis
A

D. tuberculosis
b+h 3rd pg 903
tuberculosis associated with wall calcification
schistosomiasis highly associated with squamous cell carcinoma

How well did you know this?
1
Not at all
2
3
4
5
Perfectly
107
Q

Which of the following are not true for adenocarcinoma of the bladder?

  1. accounts for 15% of bladder malignancies
  2. associated with bladder exstrophy
  3. associated with urachal remnants
  4. none of the above are false
A

A. accounts for 15% of bladder malignancies​
b+h 3rd pg 903
adenocarcinoma accounts for fewer than 1% of cases bladder malignancy

How well did you know this?
1
Not at all
2
3
4
5
Perfectly
108
Q

An irregular filling defect that moves with changes in patient position and changes size and shape with time is most suggestive of which diagnosis?

  1. bladder stones
  2. blood clots
  3. leiomyoma
  4. neurofibroma
A

B. blood clots
b+h 3rd pg 903

How well did you know this?
1
Not at all
2
3
4
5
Perfectly
109
Q

A pear-shaped bladder is suggestive of an extrinsic mass impression on the bladder by
pathologies including but not limited to lymphadenopathy, pelvic hemorrhage,
lipomatosis or aneurysm.

A. True

B. False

A

A. True
b+h 3rd pg 903

How well did you know this?
1
Not at all
2
3
4
5
Perfectly
110
Q

What is the most common location for bladder diverticula?

  1. anterolaterally
  2. anteromedially
  3. posterolaterally
  4. posteromedially
A

C. posterolaterally
b+h 3rd pg 903
near the ureterovesical junction

How well did you know this?
1
Not at all
2
3
4
5
Perfectly
111
Q

Match the fistula below with the most likely associated cause:

  1. vesicocolonic
  2. vesicovaginal
  3. vesicoenteric
  4. Crohn disease
  5. gynecologic surgery
  6. diverticulitis
  7. ulcerative colitis
  8. obstetric injury
A

  1. vesicocolic (large intestine and bladder) –> MC diverticulitis (60%) then colorectal cancer (20%) then Crohn’s (10%)
  2. vesicovaginal (colon - usually rectum and sigmoid - and vagina) –> MC complicated diverticular disease (20-40%); others include pelvic or colonic malignancies, inflammatory bowel disease and following pelvic radiotherapy
  3. vesicoenteric (colon and small bowel) –> MC Crohn’s
How well did you know this?
1
Not at all
2
3
4
5
Perfectly
112
Q

Which of the following is not true concerning bladder trauma?

  1. the distended bladder is more prone to injury than a collapsed bladder
  2. extraperitoneal rupture is less common than intraperitoneal rupture
  3. extraperitoneal bladder rupture most commonly extravasates to the retropubic space of Retzius
  4. intraperitoneal bladder rupture most commonly extravasates to the paracolic gutters
A

B. extraperitoneal rupture is less common than intraperitoneal rupture​

b+h 3rd pg 904-5

How well did you know this?
1
Not at all
2
3
4
5
Perfectly
113
Q

A “hurley-stick ureter” is associated with:

  1. urinary bladder duplication
  2. bladder ears
  3. bladder exstrophy
  4. vesicoureteral reflux
A

C. Bladder exstrophy
P&J pg. 709

How well did you know this?
1
Not at all
2
3
4
5
Perfectly
114
Q

Incontinence or retention of urine is a complication of:

  1. malakoplakia
  2. neurogenic bladder
  3. bladder diverticula
  4. radiation cystitis
A

B. neurogenic bladder
P&J pg. 714

How well did you know this?
1
Not at all
2
3
4
5
Perfectly
115
Q

An hourglass bladder is a sign of:

  1. vesicoureteral reflux
  2. megaureter megacystis syndrome
  3. cystitis cystica
  4. neurogenic bladder
A

D. neurogenic bladder
P&J pg. 714

How well did you know this?
1
Not at all
2
3
4
5
Perfectly
116
Q

What is the most common benign tumor of the bladder?

  1. rhabdomyoma
  2. hemangioma
  3. leiomyoma
  4. nephrogenic adenoma
A

C. leiomyoma
Taveras Chpt 127, pg. 1

How well did you know this?
1
Not at all
2
3
4
5
Perfectly
117
Q

The major radiographic finding in bacterial cystitis is:

  1. bladder edema
  2. small hemorrhages in the bladder mucosa
  3. prominent thickened mucosal folds with a cobblestone appearance
  4. gas within the bladder wall
A

A. bladder edema
Taveras Chpt. 128, pg. 1

How well did you know this?
1
Not at all
2
3
4
5
Perfectly
118
Q

Which of the following statements regarding bladder diverticuli is true?

  1. bladder diverticuli represent protrusions of the bladder submucosa between the muscle fibers of the bladder
  2. most diverticuli are acquired because of bladder outlet obstruction or neurogenic bladder dysfunction in which intravesical pressure increase
  3. the most common location of diverticuli are at the inferior medial aspect of the bladder
  4. congenital diverticuli are most commonly multiple
A

B. most diverticuli are acquired because of bladder outlet obstruction or neurogenic bladder
Taveras, Chpt. 128, pg. 8

How well did you know this?
1
Not at all
2
3
4
5
Perfectly
119
Q

The most common cause of a pear-shaped or a tear-shaped bladder is:

  1. cystitis cystica
  2. vesical fistula
  3. endometriosis
  4. pelvic lipomatosis
A

D. pelvic lipomatosis
Taveras, Chpt. 128, pg. 10

How well did you know this?
1
Not at all
2
3
4
5
Perfectly
120
Q

“Hooking” of the distal ureters (a hockey-stick appearance) and significant elevation of the
bladder trigone is indicative of:

  1. bladder calculi
  2. benign prostatic hypertrophy
  3. urethral valves
  4. postoperative bladder-neck contracture
A

B. benign prostatic hypertrophy
Taveras, Chpt. 129, pg. 3, P&J pg. 712

How well did you know this?
1
Not at all
2
3
4
5
Perfectly
121
Q

What is the most common cause of vesicoureteral reflux in adults?

  1. urethral stricture
  2. median enlargement of the prostate
  3. neurogenic bladder dysfunction
  4. infection
A

D. infection
P & J pg. 714

How well did you know this?
1
Not at all
2
3
4
5
Perfectly
122
Q

The recommended imaging procedure of choice for differentiating bladder calculi from other
causes of calcification is:

  1. cystography, with air or diluted contrast medium
  2. CT pelvis with contrast
  3. retrograde pyelography
  4. MRI pelvis
A

A. cystography, with air or diluted contrast medium
P&J pg. 710

How well did you know this?
1
Not at all
2
3
4
5
Perfectly
123
Q

Which of the following is true concerning exstrophy of the bladder?

  1. the bladder mucosa is continuous with the skin
  2. there is an associated widening of the pubic symphysis
  3. there is an association with epispadias
  4. all of the above are true
A

D. all of the above are true​
b+h 3rd pg 898

How well did you know this?
1
Not at all
2
3
4
5
Perfectly
124
Q

Which type of urachal remnant is most common?

  1. patent urachus
  2. umbilical-urachal sinus
  3. vesical-urachal diverticulum
  4. urachal cyst
A

A. patent urachus
b+h 3rd pg 898-9
patent urachus - 50%
urachal cyst - 30%
umbilical-urachal sinus - 15%
vesical-urachal divertuculum - 5%

How well did you know this?
1
Not at all
2
3
4
5
Perfectly
125
Q

Which of the following is false concerning urachal carcinoma?

  1. most are adenocarcinoma
  2. represents .5% of bladder carcinomas
  3. most common between the ages of 20 and 40
  4. none of the above are false
A

C. most common between the ages of 20 and 40
b+h 3rd pg 898-9
most common between 40 and 70
adenocarcinoma is 90% of cases

How well did you know this?
1
Not at all
2
3
4
5
Perfectly
126
Q

The normal wall of a well-distended bladder should not exceed:

  1. 3 mm
  2. 6 mm
  3. 9 mm
  4. 12 mm
A

B. 6 mm
b+h 3rd pg 899

How well did you know this?
1
Not at all
2
3
4
5
Perfectly
127
Q

J-hooking of the distal ureters is suggestive of which of the following?

  1. benign prostatic hypertrophy
  2. urethral stricture
  3. neurogenic bladder
  4. ectopic ureter insertion
A

A. benign prostatic hypertrophy
b+h 3rd pg 899

How well did you know this?
1
Not at all
2
3
4
5
Perfectly
128
Q

Which of the following is characterized by multiple fluid-filled submucosal cysts associated with bladder infection?

  1. cystitis cystica
  2. bullous edema
  3. interstitial cystitis
  4. eosinophilic cystitis
A

A. cystitis cystica
b+h 3rd pg 900

How well did you know this?
1
Not at all
2
3
4
5
Perfectly
129
Q

Cathartic colon most commonly involves the:

  1. entire colon.
  2. left colon.
  3. right colon.
  4. mesenteric small bowel.
A

C. right colon
Paul and Juhl’s, Essentials of Radiological Imaging, 7th ed., Lippincott, 1998, p. 616.

How well did you know this?
1
Not at all
2
3
4
5
Perfectly
130
Q

________ is a developmental rotation anomaly that results from clockwise rotation and results
in the transverse colon being positioned posterior to the duodenum, instead of its normal ventral location.

  1. Reversal of rotation
  2. Malrotation
  3. Double rotation
  4. Non-rotation
A

A. reversal of rotation
Paul and Juhl’s, Essentials of Radiological Imaging, 7th ed., Lippincott, 1998, p. 616.

How well did you know this?
1
Not at all
2
3
4
5
Perfectly
131
Q

The radiographic appearance of a cathartic colon includes all but one of the following?

  1. Appearance of UC
  2. Gaping ileocecal valve
  3. Narrowed right colon
  4. Ulcerations
A

D. ulcerations
Eisenberg 3rd ed. p.649

How well did you know this?
1
Not at all
2
3
4
5
Perfectly
132
Q

Computed tomography of an adult patient reveals a tubular cystic mass that demonstrates peripheral calcification in the right lower abdominal quadrant. This finding is associated with a round calcific density demonstrating a laminated pattern. Rupture of the previously described cystic mass would result in:

  1. hemorrhagic peritonei
  2. chylous ascites
  3. pseudomyxoma peritonei
  4. pseudochylous ascites
A

C. pseudomyxoma peritonei​
Brant and Helms, Fundamentals of Diagnostic Radiology, 2nd ed., Lippincott, 1999, p.764.

How well did you know this?
1
Not at all
2
3
4
5
Perfectly
133
Q

Double-contrast barium enema on a 36-year-old female patient demonstrates a thumbprinting pattern of the proximal portion of the transverse colon. Computed tomography reveals thickening of the wall of the transverse colon, pneumatosis, and associated irregular lumen narrowing. The patient is currently on oral contraceptives and presented clinically with abdominal pain and bloody stools. Which of the following is most likely the correct diagnosis?

  1. Ulcerative colitis
  2. Crohn’s disease
  3. Necrotizing enterocolitis
  4. Ischemic colitis
A

D. Ischemic colitis
Paul and Juhl’s, Essentials of Radiological Imaging, 7th ed., Lippincott, 1998, p.622.
Brant and Helms, Fundamentals of Diagnostic Radiology, 2nd ed., Lippincott, 1999, p.759.

How well did you know this?
1
Not at all
2
3
4
5
Perfectly
134
Q

Which of the following represents a rare combination of intracerebral gliomas and colonic
polyps?

  1. Peutz-Jeghers syndrome
  2. Turcot’s syndrome
  3. Cronkhite-Canada syndrome
  4. Cowden’s disease
A

B. Turcot’s syndrome
Paul and Juhl’s, Essentials of Radiological Imaging, 7th ed., Lippincott, 1998, p.624.

How well did you know this?
1
Not at all
2
3
4
5
Perfectly
135
Q

Double-contrast barium enema of a 30-year-old male reveals innumerable filling defects
throughout the large bowel. The patient’s past medical history includes multiple dental caries,
localized bony overgrowth in the skull and osteomas. Which of the following best represents the course of action that should be taken in this patient’s case?

  1. Follow-up radiographs taken in six-month intervals should be performed to evaluate for increasing numbers of polyps.
  2. Biopsy of the polyps should be done to evaluate for malignant potential.
  3. The patient should be made aware of the findings and instructed to consume a high fiber diet.
  4. Total colectomy is warranted since there is almost a 100% risk of developing carcinoma in this patient’s condition.
A

D. Total colectomy is warranted since there is almost a 100% risk of developing carcinoma in this patient’s condition.​
Eisenberg, Gastrointestinal Radiology, 1st ed., Lippincott, 1983, p. 715-717.

How well did you know this?
1
Not at all
2
3
4
5
Perfectly
136
Q
  • *Barium enema in a 59-year-old male demonstrates a classic bilateral contour defect with ulcerated mucosa, eccentric and irregular lumen, and overhanging margins (“apple-core”
    lesion) in the distal sigmoid colon. Clinical presentation includes crampy abdominal pain, rectal bleeding, and cachexia. Which of the following represents the best course of action in this patient’s case?**
  1. Intravenous glucagon should be administered to rule out the presence of sphincter spasm secondary to localized nerve and muscle imbalance.
  2. Nothing since this most likely represents normal bowel peristalsis.
  3. This patient most likely has carcinoma and CT of the abdomen should be ordered to evaluate for the presence of a second colorectal neoplasm and/or other distal metastases, such as to the liver.
  4. The findings in this patient most likely represent diverticulitis and he should follow-up with his medical doctor.
A

C. This patient most likely has carcinoma and CT of the abdomen should be ordered to evaluate for the presence of a second colorectal neoplasm and/or other distal metastases, such as to the liver.​
Eisenberg, Gastrointestinal Radiology, 1st ed., Lippincott, 1983, p. 651-56.
Brant and Helms, Fundamentals of Diagnostic Radiology, 2nd ed., Lippincott, 1999, p.752-54.

How well did you know this?
1
Not at all
2
3
4
5
Perfectly
137
Q

Which one of the following hernias present with symptoms 75% of the time of intermittent
abdominal pain with occasional chest pain, cardiovascular symptoms and dyspnea?

  1. Hiatal
  2. Paraesophageal
  3. Morgagni
  4. Bockdalek
A

D. Bockdalek
Eisenberg 3rd ed. p.171

How well did you know this?
1
Not at all
2
3
4
5
Perfectly
138
Q

Which of the following is false about Gardner’s Syndrome?

  1. All patients develop colorectal carcinoma
  2. All patients present with osteomas
  3. All develop multiple hemangiomas of the colon
  4. All patients have a risk of developing small bowel carcinoma
A

C. All develop multiple hemangiomas of the colon​
Eisenberg 3rd ed. p.508

How well did you know this?
1
Not at all
2
3
4
5
Perfectly
139
Q

The reflux of colon contents through the ileocecal valve is abnormal. During a lower GI study, it is normal to see contrast material in the ileum. What is the frequency of retro flow of contrast into the ileum during a lower GI study?

  1. 10%
  2. 40%
  3. 70%
  4. 90%
A

D. 90%
Eisenberg 3rd ed. p.555

How well did you know this?
1
Not at all
2
3
4
5
Perfectly
140
Q

The etiology of ulcerative colitis is unknown, some speculate that there is a connection to
connective tissue disorders and others speculate an infectious origin. Although the etiology is unknown the incidence of UC is higher among Caucasians, European Jews, and people living in developed countries. What age group is the peak incidence?

  1. 15-25 years
  2. 30-50 years
  3. 50 plus
  4. Bimodal 15-25 and 50 plus
A

D. bimodal 15-25 and 50 plus
Eisenberg 3rd ed. p.602

How well did you know this?
1
Not at all
2
3
4
5
Perfectly
141
Q

The radiographic description “bouquet of flowers” is used to describe which of the following?

  1. Colonic hamartoma
  2. Villous adenoma
  3. Villoglalandular polyp
  4. Hyperplastic polyp
A

B. Villous adenoma
Eisenberg 3rd ed. p.694

How well did you know this?
1
Not at all
2
3
4
5
Perfectly
142
Q

An ameboma is a focal hyperplastic granuloma caused by secondary bacterial infection of an
amebic abscess in the bowel wall. What percentage of patients with amebic colitis develop amebomas?

  1. less than 10%
  2. 50%
  3. greater than 90%
  4. There is no such thing as an ameboma
A

A. less than 10%
Eisenberg 3rd ed. p.701

How well did you know this?
1
Not at all
2
3
4
5
Perfectly
143
Q

A mother brings her child into your office and complains that little Trevor says his belly is sore. The child describes a classic colicky pain. On examination, you notice the patient have a mucocutaneous pigmentation. Seen in his mouth on his face, abdomen, hands and feet. What is the cause of colicky pain?

  1. Gallstone ileus
  2. Polyp
  3. Ulceration
  4. Ingestion of a foreign body
A

B. polyp
Eisenberg 3rd ed. p.722

How well did you know this?
1
Not at all
2
3
4
5
Perfectly
144
Q

Double tracking is the presence of longitudinal extraluminal tracts of barium paralleling the
lumen of the sigmoid colon. Which of the following does not exhibit this radiographic sign?

  1. Dissecting peridiverticulitis
  2. Crohn’s disease
  3. Carcinoma of the colon
  4. Ulcerative colitis
A

D. ulcerative colitis
Eisenberg 3rd ed. p.784-787

How well did you know this?
1
Not at all
2
3
4
5
Perfectly
145
Q

Although there are normal variations what is considered to be an enlarged retrorectal space
measurement?

  1. greater than 1.0cm
  2. greater than 1.5cm
  3. greater than 2.0 cm
  4. greater than 2.5cm
A

B. greater than 1.5 cm
Eisenberg 3rd ed. p.789

How well did you know this?
1
Not at all
2
3
4
5
Perfectly
146
Q

Almost 60% of large bowel obstructions are secondary to a primary carcinoma. What is the
second most common cause of large bowel obstruction?

  1. Volvulus
  2. Diverticulitis
  3. Fecal impaction
  4. Colonic bezoars
A

B. diverticulitis
Eisenberg 3rd ed. p.749

How well did you know this?
1
Not at all
2
3
4
5
Perfectly
147
Q

What is the most common location of colonic tuberculosis?

  1. Rectum
  2. Cecum
  3. Descending colon
  4. Hepatic flexure
A

B. cecum
Eisenberg 3rd ed. p.619

How well did you know this?
1
Not at all
2
3
4
5
Perfectly
148
Q

Toxic megacolon develops during relapses of chronic intermittent ulcerative colitis. Of all toxic
megacolon episodes what percentage are associated with UC?

  1. 25%
  2. 50%
  3. 75%
  4. 100%
A

C. 75%
Eisenberg 3rd ed. p.768

How well did you know this?
1
Not at all
2
3
4
5
Perfectly
149
Q

Which is the most common type of colonic polyp?

  1. Hyperplastic
  2. Adenomatous
  3. Hamartoma
  4. Villous adenoma
A

B. adenomatous
Eisenberg 3rd ed. p.684

How well did you know this?
1
Not at all
2
3
4
5
Perfectly
150
Q

Other than the appendix, where is the most common location for carcinoid tumors in the colon?

  1. Sigmoid colon
  2. Rectum
  3. Splenic flexure
  4. Hepatic flexure
A

B. rectum
Eisenberg 3rd ed. p.698,700

How well did you know this?
1
Not at all
2
3
4
5
Perfectly
151
Q

Air seen under the left diaphragm in a patient that is mentally retarded or psychotic patients
with chronic colonic enlargement is known as?

  1. Garrett superimposition
  2. Chilaiditi’s syndrome
  3. Saracens peritonitis
  4. Teutonic infection
A

B. Chilaiditi’s syndrome
Eisenberg 3rd ed. p.964

How well did you know this?
1
Not at all
2
3
4
5
Perfectly
152
Q

Crypt abscesses may be seen in association with:

  1. Crohn’s disease
  2. Whipple’s disease
  3. Ulcerative colitis
  4. Amebiasis
A

C. ulcerative colitis
Eisenberg, third edition p. 602

How well did you know this?
1
Not at all
2
3
4
5
Perfectly
153
Q

The clinical presentation of abrupt onset of lower abdominal pain and rectal bleeding in a
young woman taking birth control pills, which resolves radiographically in a few weeks is
characteristic of:

  1. Lymphogranuloma venereum
  2. Ischemic colitis
  3. Amebiasis
  4. Tuberculous colitis
A

B. ischemic colitis
Eisenberg, third edition p. 613

How well did you know this?
1
Not at all
2
3
4
5
Perfectly
154
Q

The most common extracolonic complication of amebiasis is:

  1. abdominal cramping
  2. muscular aches
  3. hepatic abscess
  4. severe septicemia
A

C. Hepatic abscess
Eisenberg, third edition p. 616

How well did you know this?
1
Not at all
2
3
4
5
Perfectly
155
Q

The Frei intradermal skin test can confirm a diagnosis of:

  1. actinomycosis
  2. Campylobacter fetus colitis
  3. Yersinia colitis
  4. lymphogranuloma venereum
A

D. lymphogranuloma venereum
Eisenberg, third edition p. 622

How well did you know this?
1
Not at all
2
3
4
5
Perfectly
156
Q

What is the most important cause of severe lower gastrointestinal bleeding in renal transplant recipients in whom immunosuppressant therapy has been initiated?

  1. cytomegalovirus
  2. strongyloidiasis
  3. staphylococcus
  4. chlamydia
A

A. cytomegalovirus
Eisenberg, third edition p. 624

How well did you know this?
1
Not at all
2
3
4
5
Perfectly
157
Q

The most common location of localized radiation injury in the colon is:

  1. ascending colon
  2. splenic flexure
  3. sigmoid colon
  4. rectum
A

D. rectum
Eisenberg, third edition p. 629

How well did you know this?
1
Not at all
2
3
4
5
Perfectly
158
Q

An uncommon multiple-system disease, characterized by ulcerations of the buccal and genital mucosa, ocular inflammation, and a variety of skin lesions, commonly presents with which of the following radiographic presentations?

  1. diverticuli
  2. skip lesions
  3. premalignant hyperplastic polyps
  4. fecal impaction
A

B. skip lesions
Eisenberg, third edition p. 634

How well did you know this?
1
Not at all
2
3
4
5
Perfectly
159
Q
  • *Macrocephaly, pigmented genital lesions, and intestinal polyposis (Ruvalcaba-Myhre-Smith
    syndrome) is also associated with which of the following:**
  1. accelerated psychomotor development in childhood
  2. prominent corneal nerves
  3. loss of fingernails and toenails
  4. brain tumors
A

B. prominent corneal nerves
Eisenberg, third edition p. 725

How well did you know this?
1
Not at all
2
3
4
5
Perfectly
160
Q

Exomphalos is:

  1. an asymptomatic condition with the small bowel on the right side of the abdomen and the colon on the left side
  2. a condition in which the colon lies dorsal to the SMA with the jejunum and duodenum anterior to it
  3. the duodenojejunal flexure lies to the right and caudad of its usual position which is to the left of the midline and approximately in the same axial plane as the 1st part of the duodenum
  4. total failure of the bowel to return to the abdomen from the umbilical cord
A

D. total failure of the bowel to return to the abdomen from the umbilical cord
Chapman 3rd, p. 204

How well did you know this?
1
Not at all
2
3
4
5
Perfectly
161
Q

Thumbprinting in the colon may be seen in association with:

  1. Yersinia enterocolitis
  2. polyarteritis nodosa
  3. radiotherapy
  4. pseudomembranous colitis
A

D. pseudomembranous colitis
Chapman, 3rd p. 248

How well did you know this?
1
Not at all
2
3
4
5
Perfectly
162
Q

In acute colonic obstruction, the possibility of perforation is likely when the cecum distends to
more than _____ cm?

  1. 3 cm
  2. 6.5 cm
  3. 10 cm
  4. 12 cm
A

C. 10 cm
Eisenberg 3rd p. 748

How well did you know this?
1
Not at all
2
3
4
5
Perfectly
163
Q

Barium enema demonstrates prominent mucosal edema with the surface being irregular and
“shaggy” secondary to diffuse superficial ulcerations. The patient has recently been on
antimicrobial drug therapy and was diagnosed with antibiotic colitis. Which of the following is
the most current term used to define this condition?

  1. Whipple’s Disease
  2. Granulomatosis Colitis
  3. Idiopathic Colitis
  4. Pseudomembranous Colitis
A

D. pseudomembranous colitis
Paul and Juhl’s, Essentials of Radiological Imaging, 7th ed., Lippincott, 1998, p. 618.

How well did you know this?
1
Not at all
2
3
4
5
Perfectly
164
Q

Which of the following systemic diseases is not considered a polyposis syndrome?

  1. Peutz-Jeghers
  2. Cronkhite-Canada
  3. Carcinoid
  4. Gardner’s
A

C. carcinoid
P.&J.

How well did you know this?
1
Not at all
2
3
4
5
Perfectly
165
Q

Most colonic malignancies are of this type…

  1. Adenocarcinoma
  2. Rhabdomyosarcoma
  3. Leiomyosarcoma
  4. Squamous cell carcinoma
A

A. adenocarcinoma
P.&J.

How well did you know this?
1
Not at all
2
3
4
5
Perfectly
166
Q

Which polyposis syndrome is not associated with malignant degeneration?

  1. Multiple polyposis syndrome
  2. Peutz-Jeghers syndrome
  3. Familial polyposis syndrome
  4. Cronkhite-Canada syndrome
A

D. Cronkhite-Canada syndrome
P.&J.

How well did you know this?
1
Not at all
2
3
4
5
Perfectly
167
Q

A second stage error of bowel rotation results in…

  1. Non-rotation
  2. Malrotation
  3. Mickel’s diverticulum
  4. Vitelline duct persistency
A

A. Non-rotation
P.&J.

How well did you know this?
1
Not at all
2
3
4
5
Perfectly
168
Q

The earliest sign of subdiaphragmatic abscess is…

  1. Pleural effusion
  2. Restricted hemidiaphragmatic motion
  3. Pneumoperitoneum
  4. Sentinel loops
A

B. Restricted hemidiaphragmatic motion
P&J

How well did you know this?
1
Not at all
2
3
4
5
Perfectly
169
Q

A cobblestone appearance of the mucosa at the gastric antrum and proximal duodenum is most suggestive of…

  1. Carcinoid tumor
  2. Squamous cell carcinoma
  3. Amyloidosis
  4. Crohn’s disease
A

D. Crohn’s disease

P&J

How well did you know this?
1
Not at all
2
3
4
5
Perfectly
170
Q

The “double bubble” sign relates to….

  1. Duodenal atresia
  2. Pneumoperitoneum
  3. Sigmoid volvulus
  4. Pyloric stenosis
A

A. Duodenal atresia
P.&J.

Triple bubble sign: jejunal atresia

How well did you know this?
1
Not at all
2
3
4
5
Perfectly
171
Q

Which is true concerning duodenal adenocarcinoma?

  1. 2nd most common malignant tumor of the duodenum
  2. most common in the duodenal bulb
  3. metastases to regional lymph nodes present in two-thirds of patients at presentation
  4. rarely seen in the periampullary region
A

C. metastases to regional lymph nodes present in two-thirds of patients at presentation​
b+h 3rd pg 825
most common malignant tumor of the duodenum
rare in the duodenal bulb
most common in the periampullary region

How well did you know this?
1
Not at all
2
3
4
5
Perfectly
172
Q

Which is not true concerning duodenal adenoma?

  1. presents as a polypoid mass
  2. multiple polyps are seen in polyposis syndromes
  3. villous adenomas have a low incidence of malignant degeneration
  4. villous adenomas have a characteristic cauliflower appearance on double-contrast exams
A

C. villous adenomas have a low incidence of malignant degeneration

b+h 3rd pg 826
villous adenomas have a high rate of malignant degeneration

How well did you know this?
1
Not at all
2
3
4
5
Perfectly
173
Q

Which is not true concerning gastrointestinal stromal tumors in the duodenum?

  1. most commonly located in the second or third portion of the duodenum
  2. malignant tumors are more common in the proximal duodenum
  3. malignant tumors range up to 20cm in size
  4. malignant GIST is the second most common primary malignancy in the duodenum
A

B. malignant tumors are more common in the proximal duodenum​
b+h 3rd pg 826
malignant tumors are more common distally

How well did you know this?
1
Not at all
2
3
4
5
Perfectly
174
Q

Which is not true concerning duodenal filling defects?

  1. Brunner gland hyperplasia/adenoma is associated with hyperacidity
  2. ectopic pancreas is the duodenum occurs most commonly in the proximal descending duodenum
  3. the appearance of areae gastrique in the duodenal bulb is associated with heterotopic gastric mucosa
  4. lymphoid hyperplasia in the duodenum is a precursor for lymphoma
A

D. lymphoid hyperplasia in the duodenum is a precursor for lymphoma
b+h 3rd pg 826-7
no evidence supports the connection between lymphoid hyperplasia and lymphoma

How well did you know this?
1
Not at all
2
3
4
5
Perfectly
175
Q

Which is not true concerning thickened duodenal folds?

  1. folds are considered to thicken when wider than 3 mm
  2. most common cause of duodenitis is h. pylori
  3. Crohn disease usually affects the first and second portion and is rarely associated with stomach involvement
  4. may be a normal variant
A

C. Crohn disease usually affects the first and second portion and is rarely associated with stomach involvement
b+h 3rd pg 827
Crohn involvement of the duodenum is almost always associated with contiguous
stomach involvement

How well did you know this?
1
Not at all
2
3
4
5
Perfectly
176
Q

Which statement is false concerning duodenal ulcers?

  1. most are associated with h. pylori infection
  2. most occur in the anterior wall of the duodenal bulb
  3. most are larger than 2 cm
  4. post-bulbar ulcers are characteristic of Zollinger-Ellison syndrome
A

C. most are larger than 2 cm
b+h 3rd pg 827-8
most are smaller than 1 cm

How well did you know this?
1
Not at all
2
3
4
5
Perfectly
177
Q

Which is false concerning duodenal diverticulum?

  1. usually incidental finding
  2. most common along inner aspect of the ascending duodenum
  3. differentiated by change in appearance during peristalsis
  4. intraluminal diverticula are congenital and have a windsock configuration
A

B. most common along inner aspect of the ascending duodenum​
b+h 3rd pg 828-9
most common along inner aspect of descending duodenum

How well did you know this?
1
Not at all
2
3
4
5
Perfectly
178
Q

Which is false concerning duodenal narrowing?

  1. annular pancreas causes congenital narrowing of the descending duodenum
  2. postbulbar ulcer is associated with narrowing of the second and third portion of the duodenum
  3. ulceration is rare in circumferential duodenal adenocarcinoma
  4. duodenal narrowing may result from inflammation in the pancreas
A

C. ulceration is rare in circumferential duodenal adenocarcinoma​
b+h 3rd pg 829-30
ulceration is common in duodenal adenocarcinoma

How well did you know this?
1
Not at all
2
3
4
5
Perfectly
179
Q

The most common cause of upper GI bleed is:

  1. duodenal ulcer
  2. esophageal varices
  3. gastric ulcer
  4. mallory-weiss tear
A

A. duodenal ulcer
b+h 3rd pg 830

How well did you know this?
1
Not at all
2
3
4
5
Perfectly
180
Q

Adenocarcinoma constitutes 80%-90% of all duodenal malignancies. What is the most common location for the tumor?

  1. Just distal to the pyloric sphincter
  2. At or proximal to the ampulla of Vater
  3. At or distal to the ampulla of Vater
  4. Localized in the duodenal bulb
A

C. at or distal to the ampulla of Vater
Eisenberg 3rd ed. p.407

How well did you know this?
1
Not at all
2
3
4
5
Perfectly
181
Q

The most common cause of duodenal fold thickening is:

  1. Zollinger-Ellison syndrome
  2. Whipple’s disease
  3. peptic ulcer disease
  4. eosinophilic enteritis
A

C. peptic ulcer disease
Eisenberg, third edition p. 337

How well did you know this?
1
Not at all
2
3
4
5
Perfectly
182
Q

A central collection of barium (dimple) within a smooth, well-demarcated filling defect in the
duodenum, representing the filling of miniature duct-like structures, is associated with:

  1. prolapsed antral mucosa
  2. papilla of Vater
  3. Crohn’s disease
  4. ectopic pancreas
A

D. ectopic pancreas
Eisenberg, third edition p. 378

How well did you know this?
1
Not at all
2
3
4
5
Perfectly
183
Q

Malignant tumors of the duodenum are most commonly found in which portion?

  1. first
  2. second
  3. third
  4. fourth
A

D. fourth
Eisenberg, third edition p. 384

How well did you know this?
1
Not at all
2
3
4
5
Perfectly
184
Q

Small hamartomatous polyps resulting in single or multiple filling defects in the duodenum are
associated with:

  1. Turcot syndrome
  2. Peutz-Jeghers syndrome
  3. Carcinoid tumor
  4. Gardner’s syndrome
A

B. Peutz-Jegher’s Syndrome
Eisenberg, third edition p. 387

How well did you know this?
1
Not at all
2
3
4
5
Perfectly
185
Q

Frostberg’s inverted sign of three is most commonly associated with:

  1. acute pancreatitis
  2. pancreatic carcinoma
  3. ectopic pancreatic pseudocyst
  4. choledochal cyst
A

A. acute pancreatitis
Eisenberg, third edition p. 357

How well did you know this?
1
Not at all
2
3
4
5
Perfectly
186
Q

A cystic dilatation of the intraduodenal portion of the common bile duct in the region of the
ampulla of Vater is known as:

  1. duplication cyst
  2. choledochocele
  3. prolapsed antral mucosa
  4. ectopic pancreas
A

B. choledochocele
Eisenberg, third edition p. 381

How well did you know this?
1
Not at all
2
3
4
5
Perfectly
187
Q

A “cloverleaf deformity” is associated with:

  1. tuberculosis
  2. Crohn’s disease
  3. Zollinger-Ellison disease
  4. duodenal ulcer disease
A

D. duodenal ulcer disease
Eisenberg, third edition p. 403

How well did you know this?
1
Not at all
2
3
4
5
Perfectly
188
Q

Which portion of the duodenum is intraperitoneal?

  1. first
  2. second
  3. third
  4. fourth
A

A. first
b+h 3rd pg 817

How well did you know this?
1
Not at all
2
3
4
5
Perfectly
189
Q

Which of the following is not true concerning tumors in the duodenum?

  1. 90% of duodenal bulb tumors are benign
  2. 50% of tumors in the second portion of the duodenum are benign
  3. 30% of tumors in the third portion of the duodenum are malignant
  4. most tumors in the fourth portion of the duodenum are malignant
A

C. 30% of tumors in the third portion of the duodenum are malignant​
b+h 3rd pg 825
50% are malignant in the 3rd portion

How well did you know this?
1
Not at all
2
3
4
5
Perfectly
190
Q

Esophageal narrowing, buccal contractures, esophageal blebs, and characteristic blisters of
the mucous membranes can be seen in which of the following?

  1. Radiation injury
  2. Graft-versus-host disease
  3. Epidermolysis bullosa
  4. Sclerotherapy
A

C. epidermolysis bullosa
Eisenberg 3rd ed. p.91

How well did you know this?
1
Not at all
2
3
4
5
Perfectly
191
Q

Barrett’s adenocarcinomas comprise _____ of all esophageal cancers.
A. less than 5%

B. 10% to 20%

C. 50%

D. 75% to 90%

A

B. 10-20%
Paul and Juhl’s, Essentials of Radiological Imaging, 7th ed., Lippincott, 1998, p 558.
Brant and Helms, Fundamentals of Diagnostic Radiology, 2nd ed., Lippincott, 1999, p 714.

How well did you know this?
1
Not at all
2
3
4
5
Perfectly
192
Q

Thrombosis in the superior vena cava may lead dilatation of upper esophageal collateral veins
and is referred to as_______.

  1. downhill varices
  2. post-thrombotic esophageal varices
  3. uphill varices
  4. caval thrombosis syndrome
A

A. downhill varices
Paul and Juhl’s, Essentials of Radiological Imaging, 7th ed., Lippincott, 1998, p.563.

downhill is less common

Uphill varices (MC) –> portal hypertension (MC cause liver cirrhosis)

How well did you know this?
1
Not at all
2
3
4
5
Perfectly
193
Q

An upper gastrointestinal study shows a smooth, well-defined filling defect, demonstrating
obtuse mucosal angles, within the esophagus. Computed tomography of the same patient
shows this lesion to be an intramural mass of soft tissue density with focal areas of
calcification. Which of the following best represents the most likely diagnosis?

  1. Lipoma
  2. Fibroepithelial polyp
  3. Fibrovascular polyp
  4. Leiomyoma
A

D. Leiomyoma
Brant and Helms, Fundamentals of Diagnostic Radiology, 2nd ed., Lippincott, 1999, p.719.

How well did you know this?
1
Not at all
2
3
4
5
Perfectly
194
Q

Which of the following best represents an area of pharyngeal mucosa protrusion through weak
areas of the lateral aspect of the esophagus near the tonsillar fossa and thyrohyoid membrane,
and are seen most commonly in wind instrument players secondary to increased
intra-pharyngeal pressure?

  1. Zenker’s diverticulum
  2. Lateral pharyngeal Diverticulum
  3. Epiphrenic diverticulum
  4. Sacculation
A

B. Lateral pharyngeal diverticulum
Brant and Helms, Fundamentals of Diagnostic Radiology, 2nd ed., Lippincott, 1999, p.712.

How well did you know this?
1
Not at all
2
3
4
5
Perfectly
195
Q

If a patient has Killian’s dehiscence what are they susceptible to developing?

  1. Zenker’s diverticulum
  2. Esophageal varices
  3. Carcinoma of the esophagus
  4. Achalasia
A

A. Zenker’s diverticulum
Eisenberg 3rd ed. p.6, 119

How well did you know this?
1
Not at all
2
3
4
5
Perfectly
196
Q

A patient is brought into your clinic complaining of dysphagia. He reports having had pneumonia last winter and since feels very week. He has also recently been diagnosed with having cataracts. What is the diagnosis?

  1. Parkinson’s
  2. Multiple sclerosis
  3. Myasthenia gravis
  4. Scleroderma
A

C. Myasthenia gravis
Eisenberg 3rd ed. p.7

How well did you know this?
1
Not at all
2
3
4
5
Perfectly
197
Q

What is the reverse figure 3 or “figure E” impression on a barium filled esophagus is a sign of?

  1. Varicosities
  2. Normal finding
  3. Coarctation of the aorta
  4. Double aortic arch
A

C. Coarctation of the aorta
Eisenberg 3rd ed. p.35

How well did you know this?
1
Not at all
2
3
4
5
Perfectly
198
Q

The most common cause of esophageal ulceration is due to?

  1. Gastric or duodenal reflux
  2. Candida albicans
  3. Tuberculosis
  4. Ingestion of corrosive agents
A

A. gastric or duodenal reflux
Eisenberg 3rd ed. p.45

How well did you know this?
1
Not at all
2
3
4
5
Perfectly
199
Q

Dysphagia in a person older than 40 years of age must be assumed to be _____ until proven
otherwise?

  1. Presbyesophagus
  2. Esophageal webs
  3. Carcinoma
  4. Infectious/inflammatory esophagitis
A

C. Carcinoma
Eisenberg 3rd ed. p.76

How well did you know this?
1
Not at all
2
3
4
5
Perfectly
200
Q

What is the 5-year survival rate for “early esophageal cancer” defined as limited to mucosa or submucosa without lymphatic metastases?

  1. 10%
  2. 30
  3. 60
  4. 90%
A

D. 90%
Eisenberg 3rd ed. p.79

How well did you know this?
1
Not at all
2
3
4
5
Perfectly
201
Q

Of the congenital tracheoesophageal fistulas, Type II and Type III are the most common forms and produce identical symptoms. Which of the following radiographic studies can differentiate the two entities?

  1. Cervical films
  2. Thoracic films
  3. KUB
  4. Chest
A

C. KUB
Eisenberg 3rd ed. p.132

How well did you know this?
1
Not at all
2
3
4
5
Perfectly
202
Q

Which of the following demonstrates a premalignant skin disorder characterized by
papillomatosis, pigmentation and hyperkeratosis which also produces a radiographic appearance of a finely nodular filling defect throughout the esophagus?

  1. Acanthosis nigricans
  2. Melanoma
  3. Benign mucous membrane pemphigoid
  4. Dermatomyositis
A

A. Acanthosis nigricans
Eisenberg 3rd ed. p.148

How well did you know this?
1
Not at all
2
3
4
5
Perfectly
203
Q

Dilated veins in the subepithelial connective tissue of the esophagus are most commonly a
result of:

  1. obstruction of the portal or splenic veins by carcinoma of the pancreas
  2. pancreatitis
  3. slow-flow states (eg. polycythemia)
  4. portal hypertension
A

D. Portal hypertension
Eisenberg, third edition p. 125

How well did you know this?
1
Not at all
2
3
4
5
Perfectly
204
Q

A pulsion diverticulum at a point of anatomic weakness at Killian’s dehiscence is known as:

  1. traction diverticula
  2. Zenker’s diverticula
  3. Epiphrenic diverticula
  4. Meckel’s diverticula
A

B. Zenker’s diverticula
Eisenberg, third edition p. 119

How well did you know this?
1
Not at all
2
3
4
5
Perfectly
205
Q
  • *Diverticula of the thoracic portion of the esophagus are most commonly found in which
    location: **
  1. upper third
  2. middle third
  3. lower third
  4. at the lower esophageal sphincter
A

B. middle third
Eisenberg, third edition p. 121

How well did you know this?
1
Not at all
2
3
4
5
Perfectly
206
Q

Which of the following types of diverticula arise in a relatively weak area on the left anterolateral wall of the esophagus between the inferior border of the aortic arch and the upper external margin of the left main bronchus, and are only demonstrated on a right anterior oblique projection?

  1. pulsion diverticuli
  2. traction diverticuli
  3. epiphrenic diverticuli
  4. lateral diverticuli
A

A. pulsion diverticula
Eisenberg, third edition p. 122

How well did you know this?
1
Not at all
2
3
4
5
Perfectly
207
Q

Which of the following esophageal diverticuli are associated with motor abnormalities of the
esophagus and are probably related to incoordination of esophageal peristalsis?

  1. intraluminal diverticuli
  2. epiphrenic diverticuli
  3. traction diverticuli
  4. lateral diverticuli
A

B. epiphrenic diverticuli
Eisenberg, third edition p. 122

How well did you know this?
1
Not at all
2
3
4
5
Perfectly
208
Q

Strictures of the esophagus can appear as soon as ______________ after the ingestion of a
caustic substance?

  1. 2 hours
  2. 2 days
  3. 2 weeks
  4. 2 months
A

C. 2 weeks
Eisenberg, third edition p. 85

How well did you know this?
1
Not at all
2
3
4
5
Perfectly
209
Q

An ascending or migrating esophageal stricture, in which there is a progressive upward extension of both the squamocolumnar junction and the level of the stricture on serial examination, is associated with an increased risk of:

  1. squamous cell carcinoma
  2. adenocarcinoma
  3. actinomycosis
  4. cytomegalovirus
A

B. adenocarcinoma
Eisenberg, third edition p. 85

How well did you know this?
1
Not at all
2
3
4
5
Perfectly
210
Q

Which of the following advanced imaging modalities can be used to specifically demonstrate Barrett’s esophagus?

  1. MRI
  2. ultrasound
  3. CT
  4. radionuclide imaging
A

D. radionuclide imaging
Eisenberg, third edition p. 52

How well did you know this?
1
Not at all
2
3
4
5
Perfectly
211
Q

The most common infectious disease of the esophagus is:

  1. herpes
  2. cytomegalovirus
  3. candidiasis
  4. tuberculosis
A

C. candidiasis
Eisenberg, third edition p. 53

How well did you know this?
1
Not at all
2
3
4
5
Perfectly
212
Q

Recall phenomenon, repeated episodes of esophagitis and ulceration, is associated with:

  1. candidiasis
  2. postsclerotherapy
  3. caustic esophagitis
  4. chemotherapy
A

D. chemotherapy
Eisenberg, third edition p. 64

How well did you know this?
1
Not at all
2
3
4
5
Perfectly
213
Q

Single or multiple deep ulcers of the esophagus associated with sinus tracts and fistula formation within the wall of the esophagus and extending to the mediastinum are common are characteristic of:

  1. tuberculous esophagitis
  2. herpes esophagitis
  3. cytomegalovirus esophagitis
  4. candida esophagitis
A

A. tuberculous esophagitis
Eisenberg, third edition p. 59

How well did you know this?
1
Not at all
2
3
4
5
Perfectly
214
Q

“Downhill” esophageal varices are produced when venous blood from the head and neck
cannot reach the heart because of an obstruction of the _________?

  1. subclavian artery
  2. superior vena cava
  3. common carotid artery
  4. brachiocephalic trunk
A

B. superior vena cava
Eisenberg, third edition p. 125

How well did you know this?
1
Not at all
2
3
4
5
Perfectly
215
Q

What is the most common non-aortic vascular lesion producing an impression on the barium-filled thoracic esophagus?

  1. persistent truncus arteriosus
  2. aberrant left pulmonary artery
  3. aberrant right pulmonary artery
  4. anomalous pulmonary venous return
A

C. aberrant right pulmonary artery
Eisenberg, third edition p. 35

How well did you know this?
1
Not at all
2
3
4
5
Perfectly
216
Q

In patients with a hiatal hernia, a bolus of swallowed barium usually hesitates briefly at the lower end of the esophagus before filling a somewhat flared portion of the esophagus
(vestibule) just proximal to the stomach. This site of hesitation represents the uppermost
portion of an area of high resting pressure known as the:

  1. A ring
  2. B ring
  3. angle of His
  4. phrenic ampulla
A

A. A ring
Eisenberg, third edition p. 165

How well did you know this?
1
Not at all
2
3
4
5
Perfectly
217
Q

The lower esophageal ring which indicates the transition zone between esophageal and gastric
mucosa is known as:

  1. B ring
  2. cardioesophageal ring
  3. Oddi ring
  4. Schatzki’s ring
A

D. Schatzki’s ring
Eisenberg, third edition p. 164

How well did you know this?
1
Not at all
2
3
4
5
Perfectly
218
Q

Failure of sphincter relaxation is defined radiographically as barium retention above the lower esophageal sphincter for longer than:

  1. 0.5 seconds
  2. 1.5 seconds
  3. 2.5 seconds
  4. 3.5 seconds
A

C. 2.5 seconds
Eisenberg, third edition p. 11

How well did you know this?
1
Not at all
2
3
4
5
Perfectly
219
Q

A barium swallow demonstrating a normal stripping peristaltic wave that clears the upper esophagus but stops at about the level of the aortic arch is characteristic of which disease process?

  1. epidermolysis bullosa
  2. scleroderma
  3. achalasia
  4. feline esophagus
A

B. Scleroderma
Eisenberg, third edition p. 8

How well did you know this?
1
Not at all
2
3
4
5
Perfectly
220
Q

Destruction of the myenteric plexuses is due to an infection that develops from the bite of an infected reduviid bug, creating an achalasia pattern in the distal esophagus. This is characteristic of:

  1. epidermolysis bullosa
  2. Riley-Day syndrome
  3. Cowden’s disease
  4. Chagas disease
A

D. Chagas disease
Eisenberg, third edition p. 15

How well did you know this?
1
Not at all
2
3
4
5
Perfectly
221
Q

Retrocardiac calcification in a retrocardiac esophageal mass is diagnostic of:

  1. rhabdomyoma
  2. adenomatous polyp
  3. leiomyoma
  4. squamous papilloma
A

C. leiomyoma
Eisenberg, third edition p. 99

How well did you know this?
1
Not at all
2
3
4
5
Perfectly
222
Q

Which of the following is NOT a risk factor/etiologic factor in the development of carcinoma of the esophagus?

  1. smoking
  2. consumption of hot tea
  3. excessive alcohol intake
  4. consumption of spicy/hot foods
A

D. consumption of spicy/hot foods
Eisenberg, third edition p. 75

How well did you know this?
1
Not at all
2
3
4
5
Perfectly
223
Q

Which of the following diseases is associated with a disturbance in iron metabolism, predominantly in middle-aged women?

  1. Plummer-Vinson syndrome
  2. Barrett’s esophagus
  3. Epidermolysis bullosa
  4. Chagas disease
A

A. Plummer-Vinson syndrome
Eisenberg, third edition p. 72

How well did you know this?
1
Not at all
2
3
4
5
Perfectly
224
Q

The oral administration of tartaric acid and sodium bicarbonate to produce carbon dioxide that distends the esophagus may be used as an intervention technique in association with:

  1. impacted foreign body
  2. esophageal varices
  3. verrucous carcinoma
  4. reflux esophagitis
A

A. impacted foreign body
Eisenberg, third edition p. 111

How well did you know this?
1
Not at all
2
3
4
5
Perfectly
225
Q

In patients with severe vomiting, whether from dietary or alcoholic indiscretion or from any other cause, the sudden development of severe epigastric pain, with radiographic evidence of pneumomediastinum, or cervical emphysema, is suggestive of:

  1. rupture of a mid-esophageal traction diverticula
  2. Mallory-Weiss syndrome
  3. Boerhaave’s syndrome
  4. intramural abscess
A

C. Boerhaave’s syndrome
Eisenberg, third edition p.140

How well did you know this?
1
Not at all
2
3
4
5
Perfectly
226
Q

Mallory-Weiss syndrome usually occurs in which patient group?

  1. pregnant women
  2. obese middle-aged men and women
  3. newborns with congenital achalasia
  4. men over the age of 50 with a history of alcohol excess
A

D. men over the age of 50 with a history of alcohol excess
Eisenberg, third edition p. 141

How well did you know this?
1
Not at all
2
3
4
5
Perfectly
227
Q

Corrosive esophagitis is most commonly due to which of the following substances?

  1. sulfate
  2. lye
  3. nitrate
  4. hydrochloric acid
A

B. lye
Eisenberg, third edition p. 63

How well did you know this?
1
Not at all
2
3
4
5
Perfectly
228
Q

Amyloidosis, when found in the esophagus, produces what radiographic appearance?

  1. dilated esophagus with decreased peristalsis
  2. failure of relaxation of the lower esophageal sphincter
  3. repetitive nonperistaltic tertiary contractions
  4. superficial erosions that appear as streaks or dots of barium superimposed on the flat mucosa of the distal esophagus
A

A. dilated esophagus with decreased peristalsis​
Eisenberg, third edition p. 11

How well did you know this?
1
Not at all
2
3
4
5
Perfectly
229
Q

Which of the following represents a neuromuscular cause of dysphagia in an adult?

  1. pharyngeal pouch
  2. scleroderma
  3. monilia
  4. Plummer-Vinson web
A

B. scleroderma
Chapman, 3rd p. 207

How well did you know this?
1
Not at all
2
3
4
5
Perfectly
230
Q

Which of the following types of diverticuli are found in the lower third of the esophagus?

  1. Traction diverticuli
  2. Epiphrenic diverticuli
  3. Zenker’s diverticuli
  4. Intramural diverticuli
A

B. Epiphrenic diverticuli
Chapman, 3rd p. 209

How well did you know this?
1
Not at all
2
3
4
5
Perfectly
231
Q

Which of the following conditions produces smooth esophageal strictures?

  1. radiotherapy
  2. leiomyosarcoma
  3. corrosive ingestion
  4. fundoplication
A

C. Corrosive ingestion
Chapman, 3rd p. 212

How well did you know this?
1
Not at all
2
3
4
5
Perfectly
232
Q

Which of the following syndromes is described as cervicoesophageal webs associated with iron-deficiency anemia and pharyngeal or esophageal carcinoma?

  1. Alport’s Syndrome
  2. Zenker’s
  3. Patterson-Kelly Syndrome
  4. Pharyngeal Carcinomatosis Syndrome (PCS)
A

C. Patterson-Kelly Syndrome
Paul and Juhl’s, Essentials of Radiological Imaging, 7th ed., Lippincott, 1998, p 554.

How well did you know this?
1
Not at all
2
3
4
5
Perfectly
233
Q

The compression of the superior vena cava may lead to which abnormality in the esophagus?

  1. Presbyesophagus
  2. Achalasia
  3. Downhill varices
  4. Uphill varices
A

C. Downhill varices
P.&J.

How well did you know this?
1
Not at all
2
3
4
5
Perfectly
234
Q

The expected radiographic finding associated with Plummer-Vinson syndrome is…

  1. Traction diverticula in the esophagus adjacent to the pulmonary hilar region
  2. Epiphrenic diverticula
  3. Anterior esophageal webs
  4. “Corkscrew” esophagus
A

C. Anterior esophageal webs
P.&J.

How well did you know this?
1
Not at all
2
3
4
5
Perfectly
235
Q

Zenker’s diverticulum can be found where?

  1. Superior-posterior esophagus
  2. Superior-anterior esophagus
  3. Mesenteric side of ileum
  4. Anti-mesenteric side of ileum
A

A. superior-posterior esophagus
P.&J.

How well did you know this?
1
Not at all
2
3
4
5
Perfectly
236
Q

An upper gastrointestinal examination with contrast reveals a filling defect of the esophagus with sharply demarcated edges (a sharp angle where the defect joins the esophageal wall). This is most consistent with…

  1. Leiomyoma
  2. Carcinoma
  3. Presbyesophagus
  4. Lye stricture
A

B. Carcinoma
P.&J.

How well did you know this?
1
Not at all
2
3
4
5
Perfectly
237
Q

The radiographic signs of obstruction of the esophagus at the level of the diaphragm, moderate dilatation of the esophagus and the presence of an air-fluid level are consistent with…

  1. Achalasia
  2. Chalasia
  3. Esophageal varices
  4. Esophageal strictures
A

A. Achalasia
P.&J.

How well did you know this?
1
Not at all
2
3
4
5
Perfectly
238
Q

The most common constituent of gallstones is…

  1. Calcium salts
  2. Bile pigments
  3. Cholesterol
  4. Calcium carbonate
A

C. Cholesterol
P.&J.

How well did you know this?
1
Not at all
2
3
4
5
Perfectly
239
Q

The most common cause for a non-functioning gallbladder is…

  1. Acute cholecystitis
  2. Chronic cholecystitis secondary to cholelithiasis
  3. Cholecystokinia deficiency
  4. Choledocal cyst
A

B. Chronic cholecystitis secondary to cholelithiasis
P.&J.

How well did you know this?
1
Not at all
2
3
4
5
Perfectly
240
Q

Most common cause of acute cholecystitis is obstruction from gallstones.

A. True

B. False

A

A. True

b+h 3rd pg 777

How well did you know this?
1
Not at all
2
3
4
5
Perfectly
241
Q

Which is not a risk factor for acalculous cholecystitis?

  1. over-consumption of fatty foods
  2. lack of oral intake
  3. post-burn
  4. post-trauma
A

A. over-consumption of fatty foods​
b+h 3rd pg 778
acalculous disease is associated with biliary stasis

How well did you know this?
1
Not at all
2
3
4
5
Perfectly
242
Q

Which of the following is not a complication of acute cholecystitis?

  1. gangrenous cholecystitis
  2. gallbladder perforation
  3. emphysematous cholecystitis
  4. sludge
  5. mirizzi syndrome
A

D. sludge
b+h 3rd pg 778

How well did you know this?
1
Not at all
2
3
4
5
Perfectly
243
Q

Which of the following is not associated with chronic cholecystitis?

  1. thickened gallbladder wall
  2. porcelain gallbladder
  3. xanthogranulomatous cholecystitis
  4. perforation of the gallbladder
A

D. perforation of the gallbladder
b+h 3rd pg 778-9

How well did you know this?
1
Not at all
2
3
4
5
Perfectly
244
Q

Which is not true concerning adenomyomatosis in the gallbladder?

  1. most frequent benign condition affecting the gallbladder
  2. forms characteristic Rokitansky-Aschoff sinuses
  3. has no malignant potential
  4. rarely has co-existing gallstones
A

D. rarely has co-existing gallstones
b+h 3rd pg 780
gallstones commonly co-exist

How well did you know this?
1
Not at all
2
3
4
5
Perfectly
245
Q

Which is not true concerning gallbladder carcinoma?

  1. cell type is squamous cell
  2. gallstones present in 70-80% of cases
  3. most common in women over 60 yoa
  4. porcelain gallbladder is a risk factor
A

A. cell type is squamous cell
b+h 3rd pg 780
it is an adenocarcinoma

How well did you know this?
1
Not at all
2
3
4
5
Perfectly
246
Q

What percentage of a patient’s with gallbladder carcinoma present with stones?

  1. 20%
  2. 40%
  3. 60%
  4. 80%
A

D. 80%
Handbook of GI and GU Radiology p. 126

How well did you know this?
1
Not at all
2
3
4
5
Perfectly
247
Q

Sclerosing cholangitis is a rare inflammatory condition of the bile ducts in which stenosis occurs for no known reason. Which of the following conditions is associated with sclerosing cholangitis?

  1. Crohn’s disease
  2. Primary biliary cirrhosis
  3. Wilson’s disease
  4. Hepatitis
A

A. Crohn’s disease
Handbook of GI and GU Radiology p. 127

How well did you know this?
1
Not at all
2
3
4
5
Perfectly
248
Q

The normal gallbladder length should not exceed?

  1. 2cm
  2. 6cm
  3. 12cm
  4. 20cm
A

C. 12 cm
Handbook of GI and GU Radiology p. 128

How well did you know this?
1
Not at all
2
3
4
5
Perfectly
249
Q

An older woman presents to your office with a history of right upper quadrant pain and constipation. You take a KUB and discover gas in the biliary tree. What is the most likely diagnosis?

  1. Gallstone ileus
  2. Appendicitis
  3. Kidney stones
  4. Behcet’s syndrome
A

A. Gallstone ileus
Eisenberg 3rd ed. p.517

How well did you know this?
1
Not at all
2
3
4
5
Perfectly
250
Q

The gallbladder is frequently folded on itself, but the position of the fold may not be fixed. When the gallbladder fundus folds on itself, it produces an appearance similar to a:

  1. Hartmann’s pouch
  2. junctional fold
  3. quadrate lobe
  4. Phrygian cap
A

D. Phrygian Cap
Taveras, Chpt 66, pg. 1

How well did you know this?
1
Not at all
2
3
4
5
Perfectly
251
Q

What percentage of gallstones are calcified and readily identifiable on plain films?

  1. 5-10%
  2. 10-15%
  3. 20-25%
  4. 90-95%
A

B. 10-15%
P&J pg. 522

How well did you know this?
1
Not at all
2
3
4
5
Perfectly
252
Q

What is the most common positional anomaly of the gallbladder?

  1. left-sided gallbladder
  2. intrapelvic gallbladder
  3. intrahepatic gallbladder
  4. double gallbladder
A

C. intrahepatic gallbladder
Taveras, Chpt 66, pg. 1

How well did you know this?
1
Not at all
2
3
4
5
Perfectly
253
Q

Patients with an uncommon congenital focal dilation of the extrahepatic bile duct, are at risk of rupture and biliary and duodenal obstruction, and are at increased risk of developing:

  1. Caroli’s disease
  2. biliary ductal atresia
  3. primary sclerosing cholangitis
  4. cholangiocarcinoma
A

D. cholangiocarcinoma
P&J pg. 532

How well did you know this?
1
Not at all
2
3
4
5
Perfectly
254
Q

Which of the following statements regarding gallbladder ultrasonography is true?

  1. six hours of fasting is required
  2. jaundice may interfere with the accuracy of the examination
  3. oral cholecystography can detect smaller stones than ultrasound
  4. GB ultrasonography is the preferred follow-up imaging modality of choice for the detection of biliary dilatation
A

A. six hours of fasting is required
P&J pg. 524

How well did you know this?
1
Not at all
2
3
4
5
Perfectly
255
Q

What is the gallbladder imaging procedure which allows the operator (radiographic technician) to institute biliary drainage if necessary?

  1. percutaneous transhepatic cholangiography
  2. oral cholecystography
  3. cholescintigraphy
  4. T-tube cholangiography
A

A. percutaneous transhepatic cholangiography​
P&J pg. 524

How well did you know this?
1
Not at all
2
3
4
5
Perfectly
256
Q

What is the most common cause of acute cholecystitis?

  1. post-surgical inflammation
  2. patients receiving hyperalimentation
  3. trauma
  4. calculus
A

D. calculus
P&J pg. 527

How well did you know this?
1
Not at all
2
3
4
5
Perfectly
257
Q

A strawberry gallbladder is associated with:

  1. adenomyomatosis
  2. cholesterolosis
  3. annular pancreas
  4. hydrops
A

B. cholesterolosis
P&J pg.. 536

How well did you know this?
1
Not at all
2
3
4
5
Perfectly
258
Q

Prophylactic cholecystectomy is generally performed in patients with:

  1. acute cholecystitis
  2. choledochocele
  3. emphysematous cholecystitis
  4. porcelain gallbladder
A

D. porcelain gallbladder
Taveras, Chpt. 67, pg. 1

How well did you know this?
1
Not at all
2
3
4
5
Perfectly
259
Q

Milk of calcium of bile is usually composed of:

  1. calcium carbonate
  2. calcium magnesium
  3. calcium phosphate
  4. ammonium
A

A. calcium carbonate
Taveras, Chpt 67, pg. 1

How well did you know this?
1
Not at all
2
3
4
5
Perfectly
260
Q

An overgrowth of gallbladder mucosa with development of mural thickening and intramural diverticuli is known as:

  1. hyperplastic cholecystoses
  2. elastosis neuromatosis
  3. adenomyomatosis
  4. primary sclerosing cholangitis
A

C. adenomyomatosis
P&J pg. 536

How well did you know this?
1
Not at all
2
3
4
5
Perfectly
261
Q

Radiographic findings on a KUB including air in the biliary tree, dilated loops of the small bowel, and calcification in an unusual position within the abdomen, usually in the right lower quadrant are suggestive of:

  1. emphysematous cholecystitis
  2. gallstone ileus
  3. pneumobilia
  4. Mirizzi syndrome
A

B. gallstone ileus
Taveras, Chpt. 67, pg. 3

How well did you know this?
1
Not at all
2
3
4
5
Perfectly
262
Q

Diffuse inflammatory fibrosis of the biliary tree may result in obliteration of the peripheral ducts, creating a “pruned tree” appearance. This is associated with:

  1. primary sclerosing cholangitis
  2. choledochal cysts
  3. acute cholecystitis
  4. primary biliary cirrhosis
A

A. primary sclerosing cholangitis​
Taveras, Chpt. 68A, pg. 2

How well did you know this?
1
Not at all
2
3
4
5
Perfectly
263
Q

Achalasia is due to a deficiency of ganglion cells in the _________ plexus.

a. Meissner
b. Auerbach
c. celiac
d. cervical

A

b. Auerbach

How well did you know this?
1
Not at all
2
3
4
5
Perfectly
264
Q

Achalasia can resemble carcinoma of the gastroesophageal junction but carcinoma involves a _________ segment and causes more __________ tapering of the distal segment.

a. shorter, irregular
b. shorter, regular
c. longer, irregular
d. longer, regular

A

c. longer, irregular

How well did you know this?
1
Not at all
2
3
4
5
Perfectly
265
Q

A _________ diverticulum is in the posterior midline of the hypopharynx at Killian’s dehiscence and __________ likely to be symptomatic.

a. Zenker, is not
b. Zenker, is
c. Killian-Jamieson, is not
d. Killian-Jamieson, is

A

b. Zenker, is

How well did you know this?
1
Not at all
2
3
4
5
Perfectly
266
Q

Which is the most common type of carcinoma of the esophagus?

a. Adenocarcinoma
b. Squamous cell
c. Small cell
d. Humongous cell

A

b. Squamous cell

How well did you know this?
1
Not at all
2
3
4
5
Perfectly
267
Q

What is the most common benign neoplasm of the esophagus?

a. Leiomyoma
b. Polyp
c. Pharyngeal retention cyst
d. Esophageal duplication cyst

A

a. Leiomyoma

How well did you know this?
1
Not at all
2
3
4
5
Perfectly
268
Q

Where is the most common location for the tears associated with Boerhaave Syndrome?

a. near the left crus of the diaphragm
b. near the right crus of the diaphragm
c. at the midpoint of the esophagus
d. at the upper esophageal sphincter

A

a. near the left crus of the diaphragm

How well did you know this?
1
Not at all
2
3
4
5
Perfectly
269
Q

Which type of esophageal perforation involves only the mucosal layer?

a. Boerhaave
b. Zenker
c. Mallory-Weiss
d. Killian-Jamieson

A

c. Mallory-Weiss

How well did you know this?
1
Not at all
2
3
4
5
Perfectly
270
Q

What is the most common cause for esophageal perforation?

a. Boerhaave syndrome
b. Esophageal instrumentation
c. Adenocarcinoma
d. Mallory-Weiss tears

A

b. Esophageal instrumentation

How well did you know this?
1
Not at all
2
3
4
5
Perfectly
271
Q

T or F: H. pylori causes 95% of duodenal ulcers.

A

TRUE

How well did you know this?
1
Not at all
2
3
4
5
Perfectly
272
Q

Rate the incidence of gastrointestinal malignancy based on location:

a. stomach > pancreas > colon
b. pancreas > stomach > colon
c. colon > stomach > pancreas
d. colon > pancreas > stomach

A

d. colon > pancreas > stomach

How well did you know this?
1
Not at all
2
3
4
5
Perfectly
273
Q

T or F: Breast cancer metastasis is the most common cause of linitis plastica.

A

TRUE

How well did you know this?
1
Not at all
2
3
4
5
Perfectly
274
Q

Which of the following is not a growth pattern associated with gastric carcinoma?

a. Polypoid mass
b. Ulcerative mass
c. Hemostatic tumor
d. Infiltrating tumor

A

c. Hemostatic tumor

How well did you know this?
1
Not at all
2
3
4
5
Perfectly
275
Q

Why does gastrointestinal lymphoma have a better prognosis than gastrointestinal carcinoma?

a. Lymphoma grows more slowly
b. Lymphoma responds better to chemotherapy
c. Lymphoma remains confined to the bowel wall
d. Lymphoma is more likely in younger, healthier people who are more likely to recover

A

c. Lymphoma remains confined to the bowel wall

How well did you know this?
1
Not at all
2
3
4
5
Perfectly
276
Q

Which of the following is NOT used to gauge the malignancy of a gastrointestinal stromal tumor?

a. Size of tumor
b. Gross appearance
c. Behavior of tumor
d. Age of patient at presentation

A

d. Age of patient at presentation

How well did you know this?
1
Not at all
2
3
4
5
Perfectly
277
Q

Which portion of the duodenum is most commonly constricted by an annular pancreas?

a. Descending
b. Transverse
c. Ascending
d. All portions are equally likely to be constricted

A

a. Descending

How well did you know this?
1
Not at all
2
3
4
5
Perfectly
278
Q

Which of the following signs and symptoms is NOT associated with Ménétrier disease?

a. Massive gastric folds
b. Self-limiting in adults
c. Excessive amounts of transforming growth factor α
d. Protein-losing gastropathy

A

b. Self-limiting in adults

How well did you know this?
1
Not at all
2
3
4
5
Perfectly
279
Q

Flattened or absent villi and a decrease in the number of intestinal absorptive cells are characteristics of which disease?

a. Ulcerative colitis
b. Celiac disease
c. Ménétrier disease
d. Peutz-Jegher syndrome

A

b. Celiac disease

How well did you know this?
1
Not at all
2
3
4
5
Perfectly
280
Q

Behçet disease resembles what other disease when presenting at the ileocecal junction?

a. Ulcerative colitis
b. Celiac disease
c. Ménétrier disease
d. Peutz-Jegher syndrome

A

b. Celiac disease

How well did you know this?
1
Not at all
2
3
4
5
Perfectly
281
Q

T or F: A Morgagni hernia is the most common diaphragmatic hernia.

A

FALSE

How well did you know this?
1
Not at all
2
3
4
5
Perfectly
282
Q

A defect in the hemidiaphragm at the site of the embryonic pleuroperitoneal canal is called a ________ hernia.

a. Bochdalek
b. Morgagni
c. ventral
d. epigastric

A

a. Bochdalek

How well did you know this?
1
Not at all
2
3
4
5
Perfectly
283
Q

Familial hamartomatous polyps with melanin freckles are symptoms of which disease?

a. Cowden disease
b. Cronkhite-Canada syndrome
c. Caroli disease
d. Peutz-Jeghers syndrome

A

d. Peutz-Jeghers syndrome

How well did you know this?
1
Not at all
2
3
4
5
Perfectly
284
Q

Which of the following is NOT associated with Caroli disease?

a. Saccular ectasia of the intrahepatic bile duct
b. Medullary sponge kidney
c. Biliary obstruction
d. Autosomal recessive polycystic kidney disease

A

c. Biliary obstruction

How well did you know this?
1
Not at all
2
3
4
5
Perfectly
285
Q

The Todani classification is applied to which type of cysts?

a. Choledocal
b. Duplication
c. Retention
d. Hydatid

A

a. Choledocal

How well did you know this?
1
Not at all
2
3
4
5
Perfectly
286
Q

Occlusion of hepatic veins from hepatic venous or inferior vena cava obstruction is characteristic of which disease/syndrome?

a. Budd-Chiari
b. Caroli
c. Cowden
d. Cronkhite-Canada

A

a. Budd-Chiari

How well did you know this?
1
Not at all
2
3
4
5
Perfectly
287
Q

T or F: Adenocarcinoma is the most common primary malignancy of the liver.

A

FALSE

How well did you know this?
1
Not at all
2
3
4
5
Perfectly
288
Q

Wall echo shadow sign is associated with which of the following?

  1. Appendicolith
  2. Cholelithiasis
  3. Entericolith
  4. Nephrolithiasis
  5. Uterine fibroid
A

b. Cholelithiasis

How well did you know this?
1
Not at all
2
3
4
5
Perfectly
289
Q

The Hurst phenomenon is associated with which of the following?

  1. Achalasia
  2. Candida esophagitis
  3. Diffuse esophageal spasm
  4. Esophageal varices
  5. Gastric adenocarcinoma
A

a. Achalasia

How well did you know this?
1
Not at all
2
3
4
5
Perfectly
290
Q

Which of the following lies in the range of causing self limiting radiation esophagitis?

  1. 100 rads
  2. 1,000 rads
  3. 3,500 rads
  4. 6,000 rads
  5. 10,000 rads
A

c. 3,500 rads

How well did you know this?
1
Not at all
2
3
4
5
Perfectly
291
Q

Which of the following represents the 3rd most common manifestation of involvement of scleroderma?

  1. Anorectal
  2. Esophagus
  3. Raynaud phenomenon
  4. Skin
  5. Stomach
A

b. Esophagus

How well did you know this?
1
Not at all
2
3
4
5
Perfectly
292
Q

Over which age does tylosis leads to squamous cell carcinoma of the esophagus in 95% of those afflicted?

  1. 15 years
  2. 25 years
  3. 35 years
  4. 55 years
  5. 70 years
A

e. 70 years

How well did you know this?
1
Not at all
2
3
4
5
Perfectly
293
Q

Which of the following could represent a cause of Curling ulcers of the stomach?

  1. Alcoholism
  2. Helicobacter pylori
  3. NSAID consumption
  4. Severe burns
  5. Spirochete infection
A

d. Severe burns

How well did you know this?
1
Not at all
2
3
4
5
Perfectly
294
Q

Which of the following blood types are most predisposed to developing duodenal ulcers?

  1. A+
  2. AB
  3. B-
  4. B+
  5. O
A

e. O

How well did you know this?
1
Not at all
2
3
4
5
Perfectly
295
Q

In the case of phlegmonous gastritis, when the patient experiences pain that abates upon assuming a seated position, this is termed which of the following?

  1. Dienenger sign
  2. Hellmer sign
  3. Kirklin sign
  4. Racemoose sign
  5. Stierlin sign
A

a. Dienenger sign

296
Q

What percent of pleural effusions associated with Meigs syndrome occurs on the right side?

  1. 45
  2. 65
  3. 80
  4. 95
  5. 100
A

b. 65

297
Q

The pleural effusions associated with acute pancreatitis are mostly _____ and those associated with chronic pancreatitis are mostly _____.

  1. Bilateral, left sided
  2. Left sided, left sided
  3. Left sided, right sided
  4. Right sided, left sided
  5. Right sided, right sided
A

b. Left sided, left sided

298
Q

Which of the following is the second most common gastrointestinal malignancy?

  1. Colon
  2. Esophageal
  3. Gallbladder
  4. Gastric
  5. Pancreas
A

e. Pancreas

299
Q

The most common type of gastric volvulus tends to occur when which of the following conditions is present?

  1. Combined hernia
  2. Paraesophageal hernia
  3. Post surgical hernia
  4. Sliding hernia
  5. Spigelian hernia
A

b. Paraesophageal hernia

300
Q

Which of the following infections can have an association with the development of Guillain-Barre syndrome within 3 weeks of the infection?

  1. Campylobacter jejuni
  2. Giardia lamblia
  3. Salmonella typhi
  4. Isospora belli
  5. Yersinia enterocolitica
A

a. Campylobacter jejuni

301
Q

Which of the following represents the second most common gastrointestinal site of involvement of scleroderma?

  1. Anorectal
  2. Colon
  3. Esophagus
  4. Small bowel
  5. Stomach
A

d. Small bowel

302
Q

A patient presents with recent development and progressive worsening of lactose intolerance and evidence of hyposplenia. There is also evidence of an increase in folds of the ileum. Which of the following is the most likely diagnosis?

  1. Celiac disease
  2. Crohn disease
  3. Mirizzi syndrome
  4. Ulcerative colitis
  5. Whipple disease
A

a. Celiac disease

303
Q
  1. When the appendix measures _____mm in diameter, appendicitis can be diagnosed with 100% sensitivity and specificity.
    1. 3
    2. 5
    3. 6
    4. 7
    5. 9
A

d. 7

304
Q

Cigarette smoking decreases the risk factor for which of the following conditions?

  1. Celiac disease
  2. Chagas
  3. Crohn disease
  4. Ulcerative colitis
  5. Whipple disease
A

d. Ulcerative colitis

305
Q

Wilson disease is a result of a mutation on which of the following chromosomes?

  1. 6
  2. 13
  3. 15
  4. 17
  5. 22
A

b. 13

306
Q

Colon cutoff sign is characterized by a paucity of gas beyond the _________ and occurs more frequently due to _________?

  1. Hepatic flexure, acute pancreatitis
  2. Hepatic flexure, cholecystitis
  3. Hepatic flexure, malignant invasion of the gastrocolic ligament
  4. Splenic flexure, acute pancreatitis
  5. Splenic flexure, malignant invasion of the gastrocolic ligament
A

d. Splenic flexure, acute pancreatitis

307
Q
  1. All of the following are a direct result of Potter syndrome except which of the following?
    1. Abnormal facies
    2. Limb anomalies
    3. Pneumothorax
    4. Pulmonary hypoplasia
    5. Renal agenesis
A

c. Pneumothorax

308
Q

Autonephrectomy is associated with which of the following conditions?

  1. Castlemann syndrome
  2. Ivemark syndrome
  3. Renal papillary necrosis
  4. Sickle cell disease
  5. Tuberculosis
A

e. Tuberculosis

309
Q

Which of the following occurs almost exclusively in individuals of African descent and have sickle cell trait?

  1. Clear cell sarcoma of the kidney
  2. Grawitz tumor
  3. Medullary carcinoma of the kidney
  4. Mesoblastic nephroma
  5. Rhabdoid tumor of the kidney
A

c. Medullary carcinoma of the kidney

310
Q

Which of the following is the most common pediatric bladder tumor?

  1. Hemangioma
  2. Leiomyoma
  3. Liposarcoma
  4. Papilloma
  5. Rhabdomyosarcoma
A

e. Rhabdomyosarcoma

311
Q

In 10% of cases of which of the following renal tumors, is there coexistent chromophobe renal cell carcinoma?

  1. Angiomyolipoma
  2. Lipomatosis
  3. Mesoblastic nephroma
  4. Multilocular cystic nephroma
  5. Oncocytoma
A

e. Oncocytoma

312
Q

Which of the following cell types represents 70% of renal cell carcinoma?

  1. Chromophobe
  2. Clear cell
  3. Granular cell
  4. Papillary
  5. Sarcomatoid
A

b. Clear cell

313
Q

Which of the following compositions of nephrolithiasis is the least fragile, thus is not responsive to extracorporeal shockwave lithotripsy?

  1. Calcium oxalate with rough surface
  2. Calcium phosphate
  3. Cystine with smooth surface
  4. Magnesium ammonium phosphate
  5. Xanthine
A

c. Cystine with smooth surface

314
Q

Growing calculus sign is a diagnostic sign of which of the following conditions?

  1. Alport syndrome
  2. Hypervitaminosis D
  3. Medullary sponge kidney
  4. Primary hyperparathyroidism
  5. Renal papillary necrosis
A

c. Medullary sponge kidney

315
Q

A 47 years of age female patient presents with low back pain. Clinical evaluation yields a history of recurrent urinary tract infections. Computed tomography images of the abdomen yield the presence of nephromegaly and fragmented stone sign. Which of the following is the most appropriate designation?

  1. Medullary sponge kidney
  2. Polyarteritis nodosa
  3. Renal milk of calcium
  4. Renal cortical necrosis
  5. Xanthogranulomatous pyelonephritis
A

e. Xanthogranulomatous pyelonephritis

316
Q

A patient is diagnosed with Stauffer syndrome. Which of the following organs enzyme level are elevated?

  1. Gallbladder
  2. Heart
  3. Liver
  4. Pancreas
  5. Stomach
A

c. Liver

317
Q

A 49 years of age female presents with abdominal pain. Computed tomography images of the abdomen demonstrate a calcified soft tissue mass that is replacing the greater omentum. Which of the following is the most likely cause?

  1. Metastatic spread of mucinous cystadenocarcinoma of the colon
  2. Metastatic spread of mucinous cystadenocarcinoma of the ovary
  3. Metastatic spread of schirrhous carcinoma of the breast
  4. Omental cyst
  5. Omental infarction
A

b. Metastatic spread of mucinous cystadenocarcinoma of the ovary

318
Q

A 35 years of age male presents with mid-back pain that is worsening in severity. He also mentions chronic indigestion that is also worsening in severity. Recently he has been experiencing worsening dysphagia. A chest series of conventional radiographs is obtained and demonstrate a soft tissue mass and an air-fluid level that is biased to the right side of the distal esophagus. Which of the following is the most likely underlying cause?

  1. Diffuse esophageal spasm
  2. Esophageal varices
  3. Extrinsic gastric mucosa
  4. Scleroderma
  5. Tuberculosis
A

a. Diffuse esophageal spasm

319
Q

A 24 years of age female presents with progressive dysphagia. Recent laboratory studies suggested iron deficiency anemia was present. Which of the following findings is suspected upon review of computed tomography images of the head, neck, and chest?

  1. Killian-Jamieson diverticulum
  2. Post-cricoid esophageal web
  3. Schatzki ring
  4. Type IVb tracheoesophageal fistula
  5. Zenker diverticulum
A

b. Post-cricoid esophageal web

320
Q

A 57 years of age male presents with severe mid-back and neck pain following an episode of severe vomiting only hours earlier. Clinical evaluation yields a history of severe alcoholism. In which of the following locations is a tear most likely?

  1. At the duodenal bulb
  2. At the magenstrasse
  3. Left lateral wall of the distal esophagus
  4. Posterior wall of the proximal esophagus
  5. Right lateral wall of the mid esophagus
A

c. Left lateral wall of the distal esophagus

321
Q

Which of the following anomalies of the pancreas commonly demonstrates the double bubble sign on conventional radiographs?

  1. Annular pancreas
  2. Divism
  3. Ectopic pancreas
  4. Hypoplasia
  5. Regression of the dorsal duct
A

a. Annular pancreas

322
Q

Which of the following is commonly associated with absence of the spleen?

  1. Antral atresia
  2. Congenital hypertrophic pyloric stenosis
  3. Duplication cyst of the stomach
  4. Microgastria
  5. Pyloric atresia
A

d. Microgastria

323
Q

A 5 weeks of age male is presented with a history of forceful vomiting and colicky behavior. An upper gastrointestinal series was recently performed. Upon review of the images, train track sign is demonstrated. Which of the following is responsible?

  1. Antral atresia
  2. Congenital hypertrophic pyloric stenosis
  3. Duplication cyst of the stomach
  4. Microgastria
  5. Pyloric atresia
A

b. Congenital hypertrophic pyloric stenosis

324
Q

A 50 years of age male presents with a history of worsening pain widely spread at and around the thoracolumbar junction. Conventional radiographs were obtained and were normal. A barium study was obtained and demonstrated the presence of an ulcer at the fourth segment of the duodenum and another at the proximal jejunum. Which of the following diagnoses is strongly suggested?

  1. Ascending cholangitis
  2. Crohn disease
  3. Gastrinoma
  4. Insulinoma
  5. Pancreatic adenocarcinoma
A

c. Gastrinoma

325
Q

Which of the following underlying conditions is most commonly associated with the formation of gastric bezoars?

  1. Diabetes
  2. Pica
  3. Post-operative stricture
  4. Schizophrenia
  5. Tourette syndrome
A

a. Diabetes

326
Q

A 43 years of age male presents with thoracolumbar pain. He has a long history of alcoholic binges and reports a prior hospitalization due to vomiting blood 2 months prior. Review of computed tomography images of the abdomen yielded normal liver signal and size, the presence of gastric varices, no appreciable abdominal masses or calcifications, and peripancreatic fat stranding. Which of the following is the most likely conclusion?

  1. Acute pancreatitis
  2. Chronic pancreatitis
  3. Lymphoma
  4. Metastatic disease
  5. Pancreatic adenocarcinoma
A

a. Acute pancreatitis

327
Q

A 54 years of age male presents with epigastric pain. Clinical evaluation yields a recent history of unexpected weight loss, chronic nausea, and peripheral edema. Upon review of computed tomography images of the chest yielded no chest masses and normal cardiovascular structures however thickening of the wall of the stomach prompted computed tomography scan of the abdomen. Abdominal images yielded the presence of thickening and enlargement of the rugae. Laboratory evaluation yielded absence of gastric acid, hypoproteinemia, and hypochlorhydria. Which of the following is most likely?

  1. Atrophic gastritis
  2. Celiac disease
  3. Cystic gastritis
  4. Sarcoidosis
  5. Scleroderma
A

c. Cystic gastritis

328
Q

Radiographic evaluation of a 4 weeks of age male demonstrates the presence of the cecum in the left upper quadrant. Which of the following is the diagnosis?

  1. Hyper-rotation
  2. Malrotation
  3. Non-rotation
  4. Normal variant
  5. Reversed rotation
A

a. Hyper-rotation

329
Q

Which of the following represents an autosomal recessive condition characterized by mental retardation, peculiar facies, ear defects, and lymphangiectasia?

  1. Hennekam syndrome
  2. Klippel-Trenaunay syndrome
  3. Noonan syndrome
  4. Turner syndrome
  5. Waldmann syndrome
A

a. Hennekam syndrome

330
Q

An intestinal hernia that occurs at the Grynfeltt-Lesshaft space represents which type of hernia?

  1. Littre
  2. Lumbar
  3. Perineal
  4. Richter
  5. Spigelian
A

b. Lumbar

331
Q

Which of the following hernias involves a Meckel diverticulum?

  1. Littre
  2. Lumbar
  3. Perineal
  4. Richter
  5. Spigelian
A

a. Littre

332
Q

A 62 years of age male presents with abdominal pain. Clinical evaluation yields a history of previous abdominal surgery. No symptoms of obstruction were detected however concern for complications resulted in imaging acquisition. Radiographs were obtained but were normal. Upon review of computed tomography images of the abdomen with barium enema, a herniation of the intestinal wall is noted with a patent lumen. Which of the following hernia designations is most appropriate?

  1. Littre
  2. Richter
  3. Spigelian
  4. Umbilical
  5. Ventral
A

b. Richter

333
Q

Which of the following represents the most common portion of bowel involved in atresia?

  1. Ascending
  2. Descending
  3. Rectum
  4. Sigmoid
  5. Transverse
A

e. Transverse

334
Q

A 56 years of age male presents with abdominal pain and prolonged diarrhea lasting 2 weeks and has not yet resolved. Upon review of a barium enema study, decreased haustration and aphthous ulcers are noted, predominately affecting the descending colon. Follow-up computed tomography with barium enema study reveals pancolitis and demonstrates the previous findings to better advantage. Which of the following is the most likely organism responsible?

  1. Campylobacter
  2. Salmonella
  3. Shigella
  4. Tuberculosis
  5. Yersinia
A

a. Campylobacter

335
Q

A 56 years of age female presents with abdominal pain and thoracolumbar pain. Radiographs reveal the presence of thumb-printing and colon cutoff sign with abrupt ending of colonic gas proximal to the splenic flexure. Which of the following is the most likely conclusion?

  1. Acute pancreatitis
  2. Cathartic colon
  3. Chronic pancreatitis
  4. Colon carcinoma
  5. Ischemic colitis
A

e. Ischemic colitis

336
Q

Which of the following polyposis syndromes has a risk of developing pancreatic carcinoma that is greater than 200 times that of the general population?

  1. Bannayan-Riley-Ruvalcaba syndrome
  2. Cowden syndrome
  3. Gardner syndrome
  4. Lynch syndrome
  5. Peutz-Jeghers syndrome
A

e. Peutz-Jeghers syndrome

337
Q

What percent of patients with primary sclerosing cholangitis also have ulcerative colitis?

  1. 10
  2. 20
  3. 40
  4. 60
  5. 80
A

e. 80

338
Q

Gallbladder carcinoma is associated with gallstones in what percentage of cases?

a. 1-3%
b. 20-40%
c. 45-55%
d. 80-90%

A

d. 80-90%

339
Q

The most common malignant tumor to metastasize to the gallbladder is:

a. adenocarcinoma
b. squamous cell carcinoma
c. renal cell carcinoma
d. melanoma

A

d. melanoma

340
Q

Identify the false statement concerning normal gallbladder anatomy:

a. the spiral valves of Heister are small folds in the cystic duct
b. phrygian cap is normal variant involving folding of the gallbladder fundus
c. normal gallbladder measures between 2 and 5 cm in diameter
d. gallbladder wall should not exceed 5 mm

A

d. gallbladder wall should not exceed 5 mm

(wall thickness should not exceed 3mm)

341
Q

Which percentage concerning gallstones is correct:

a. gallstones are found in 20% of the general population
b. 15% of gallstones are predominantly cholesterol
c. 85% are calcified enough to be seen on plain films
d. 20% of gallstones are not seen on CT

A

d. 20% of gallstones are not seen on CT

*Gallstone factoids:

  • seen in 8% of general population; 15% in those 40-60 yoa.
  • 85% of gallstones are cholesterol; 15% are bilirubin
  • 10% calcified enough to see on plain film;
  • US or MRCP is modality of choice
342
Q

Which of the following are not commonly found in choleliths?

a. calcium carbonate
b. cacium bilirubinate
c. calcium phosphate
d. all of the above are commonly found in choleliths

A

c. calcium phosphate

343
Q

Which of the following diagnostic imaging modalities of the gallbladder is based on hepatobiliary function and involves ingestion of an iodinated compound?

a. Gallbladder ultrasonography
b. Cholescintigraphy
c. Oral cholecystography
d. Percutaneus transhepatic cholangiography

A

c. Oral cholecystography

344
Q

What is the second most common malignant primary hepatic tumor?

a. Hepatocelluar carcinoma
b. Fibrolamellar carcinoma
c. Cholangicarcinoma
d. Metastases

A

c. Cholangicarcinoma

345
Q

Which of the following is not associated with Caroli’s disease?

a. It is a type V choledochal cysts.
b. It is associated with medullary sponge kidney.
c. It is associated with autosomal recessive polycystic kidney disease.
d. Cholangiocarcinoma develops in 95% of cases.

A

d. Cholangiocarcinoma develops in 95% of cases.

346
Q

What is the most common type of choledochal cyst?

a. Type I
b. Type II
c. Type III
d. Type IV

A

a. Type I

347
Q

Which of the following cholangiocarcinomas are small, aggressive, occurs at the junction of the right and left hepatic ducts causing obstruction of both ductal systems, and is commonly referred to as Klastkin’s tumor?

a. Caroli’s disease
b. Peripheral cholangiocarcinoma
c. Hilar cholangiocarcinoma
d. Extrahepatic cholangiocarcinoma

A

c. Hilar cholangiocarcinoma

348
Q

Complication of cholescystitis occurs in 25-30% of patients. Which of the following complications has a 25% mortality rate?

a. Gallbladder perforation
b. Gangrenous cholecystitis
c. Emphysematous cholecystitis
d. Hemorrhagic cholecystitis

A

a. Gallbladder perforation

349
Q

Which of the following choledochal cyst is also referred to as a choledochocele?

a. Type I
b. Type II
c. Type III
d. Type IV

A

c. Type III

350
Q

Which of the following inflammatory processes of the biliary tree is associated with ulcerative colitis in 50% of cases?

a. Choledocholithiasis
b. Sclerosing cholangitis
c. AIDS associated cholangitis
d. Oriental cholangitis

A

b. Sclerosing cholangitis

351
Q

Which of the following is not a sign of cholelithiasis on conventional radiographs?

a. Claw sign
b. Crow’s foot
c. Mercedes benz sign
d. Faceted calculi

A

a. Claw sign

352
Q

Which of the following conditions carry a 10-20% risk of gallbladder carcinoma?

a. Adenomyomatosis
b. Hypoalbuminemia
c. AIDS
d. Porcelain gallbladder

A

d. Porcelain gallbladder

353
Q

Which of the following radiographic signs relates to a pneumoperitoneum?

a. Poppy seed calcifications
b. Football sign
c. Bird-beak appearance
d. Double track sign

A

b. Football sign

354
Q

The “dog ears” sign related to which diagnosis?

a. Closed loop obstruction
b. Ascites
c. Sigmoid volvulus
d. Bladder diverticulum

A

b. Ascites

355
Q

The most dependent portion of the peritoneal cavity in a supine patient is:

a. right subphrenic space
b. anterior subhepatic space
c. right hepatorenal fossa
d. left subhepatic space

A

c. right hepatorenal fossa

(aka morison’s pouch)

356
Q

Which of the following is NOT true?

a. the right subphrenic and subhepatic spaces communicates feely with the pelvic peritoneal cavity via the right paracolic gutter
b. the left subphrenic space communicates freely with the left subhepatic space
c. left subphrenic space communicates freely with the right subphrenic space
d. the left subphrenic space is seperated from the left paracolic gutter by the phrenicocolic ligament

A

c. left subphrenic space communicates freely with the right subphrenic space

(separated by the falciform ligament)

357
Q

Which of the following is NOT true?
the lesser sac communicates with the rest of the peritoneal cavity by the foramen of
Winslow
the lesser sac is normally filled with fluid
the lesser sac is in contact with the the diaphragm
the stomach, liver and pancreas all border the lesser sacv

A

the lesser sac is normally filled with fluid

358
Q

Which is NOT located in the anterior pararenal space?

a. pancreas
b. duodenal loop
c. ascending and descending portions of the colon
d. aorta

A

d. aorta

359
Q

Which of the following spaces in extraperitoneal?

a. retropubic space of Retzius
b. pouch of Douglas
c. rectovesical pouch
d. vesicouterine pouch

A

a. retropubic space of Retzius

360
Q

Which of the following is NOT true concerning Ascites?

a. requires 500 ML of fluid to be seen on plain flim
b. can produce a “dog’s ears” appearance in the pelvis
c. produces medial displacement of liver, spleen and colon
d. can be seen first in Douglas’s pouch on a recumbent exam

A

d. can be seen first in Douglas’s pouch on a recumbent exam

(morison’s pouch is most dependent location on recumbant exam)

361
Q

Which is INCORRECT concerning pseudomyxoma peritonei?

a. associated with a ruptured mucocele
b. associated with intraperitoneal spread of mucinous adenocarcinoma
c. rarely shows calcification
d. causes mass effect on the liver and bowel

A

c. rarely shows calcification

(when calcified, shows ringlike or punctate calcifications on plain film)

362
Q

Which of the following is the most sensitive for free air in the peritoneal cavity?

a. upright chest
b. lateral decubitus chest
c. recumbant chest
d. KUB

A

a. upright chest

363
Q

Which of the following is NOT associated with pneumoperitoneum?

a. rigler’s sign
b. football sign
c. double wall sign
d. sandwich sign

A

d. sandwich sign

(which is associated with lymphadenopathy/lymphoma)

364
Q

What is the maximum number of small bowel loops of gas that may be seen on a “normal” abdominal plain film?

a. 0
b. 1-2
c. 3-4
d. 5-6

A

b. 1-2

(should measure no more than 2.5 - 3 cm)

365
Q

Which of the following is not true concerning dilated bowel?

a. large bowel is considered dilated when greater than 5 cm
b. the cecum is considered dilated when greater than 8 cm
c. the small bowel rarely exceeds 5 cm
d. small bowel is considered dilated when greater than 4 cm

A

d. small bowel is considered dilated when greater than 4 cm

(dilated when greater than 2.5-3 cm)

366
Q

Which of the following is NOT true concerning small bowel obstructions?

a. accounts for 20% of surgical admissions for acute abdominal pain
b. accounts for 80% of intestinal tract obstructions
c. most common cause in the western world is incarcerated hernia
d. may show a “string of pearls” sign on plain film

A

c. most common cause in the western world is incarcerated hernia

(the m/c cause is post-surgical adhesions)
(incarcerated hernia is the m/c cause in developing nations)

367
Q

Which of the following signs is not matched correctly with type of obstruction?

a. small-bowel feces sign for obstruction
b. beak sign for strangulation
c. whorl sign for closed-loop obstruction
d. donut sign for intussusception

A

b. beak sign for strangulation

368
Q

Which of the following is NOT an indication for bowel ischemia?

a. thumbprinting
b. pneumatosis intestinalis
c. a decreased bowel lumen diameter
d. thickening of the bowel wall

A

c. a decreased bowel lumen diameter

bowel lumen increases >3 cm for small, >5 cm for large, > 8 cm for cecum

369
Q

Regarding pneumatosis intestinalis, which is false?

a. may be caused by bowel necrosis, ulceration or increased membrane permeability
b. findings include air bubbles in bowel wall on CT
c. it is not considered benign, even when asymptomatic
d. can be related to pulmonary origin

A

c. it is not considered benign, even when asymptomatic

(usually benign and clinically insignificant in asymptomatic pts)

370
Q

Which sign is associated with lymphadenopathy?

a. sandwich sign
b. donut sign
c. hampton’s line
d. thumbprinting

A

a. sandwich sign

371
Q

Which of the following is true concerning Hodgkin’s lymphoma?

a. represents 60% of all lymphomas
b. most common in 30 to 70 year olds
c. GI tract involvement is less common than in non-Hodgkin’s disease
d. nearly all cases show abdominal adenopathy at presentation

A

c. GI tract involvement is less common than in non-Hodgkin’s disease

*Corrections to the other options:

  • Hodgkin’s represents 20-40% of all lymphomas
  • seen in 25-30 and 70+ year olds
  • only 25% show abdominal adenopathy
372
Q

Which of the following is NOT true concerning non-hodgkin’s lymphoma?

a. more commonly involve extra-nodal sites when compared to hodgkin’s disease
b. abdominal adenopathy present in 50% at presentation
c. involve the spleen in 20% of cases
d. involve the liver in 14% of cases

A

c. involve the spleen in 20% of cases

(involve the spleen in 40% of cases)

373
Q

Which is the least common site for metastatic tumor implantation?

a. pelvic cul-de-sac
b. right paracolic gutter
c. greater omentum
d. lesser omentum

A

d. lesser omentum

374
Q

What is the appearance of uncomplicated lymphangiomas on MR?

a. low T1, low T2
b. low T1, high T2
c. high T1, high T2
d. high T1, low T2

A

b. low T1, high T2

375
Q

Which of the following is the most common type of sarcoma in the retroperitoneum?

a. liposarcoma
b. fibrosarcoma
c. osteosarcoma
d. rhabdomyosarcoma

A

a. liposarcoma

376
Q

What is the most common cause of retroperitoneal fibrosis?

a. idiopathic
b. methysergide use
c. inflammatory aneurysms
d. metastatic malignancy with fibrotic reaction

A

a. idiopathic

  • idiopathic - 2/3 (67%)
  • methysergide - 12%
  • inflammatory aneurysms - 5-10%;
  • mets with fibrosis - 8-10%
377
Q

Which of the following is the hallmark of retroperitoneal fibrosis on EXU?

a. smooth extrinsic narrowing of one or both ureters
b. renal pelvis distortion
c. abrupt dilation of one or both ureters
d. renal displacement inferior and lateral

A

a. smooth extrinsic narrowing of one or both ureters

378
Q

T of F: the pelvis is the most common site for abscess formation.

A

TRUE

379
Q

Which of the following is not a component of rigler’s triad?

a. dilated small bowel loops
b. air in the biliary tree
c. calcified gallstone in an ectopic location
d. air in the portal venous system

A

d. air in the portal venous system

(rigler’s triad indicates gallstone ileus)

380
Q

The normal range of renal length, as visualized on a KUB projection, is:

a. 2.5 + 0.37 times the height of the second lumbar vertebra
b. 3.3 + 0.37 times the height of the second lumbar vertebra
c. 3.7 + 0.37 times the height of the second lumbar vertebra
d. 4.5 + 0.37 times the height of the second lumbar vertebra

A

c. 3.7 + 0.37 times the height of the second lumbar vertebra

381
Q

Which of the following is a contraindication to injection of intravenous contrast material for evaluation of the urinary tract?

a. serum creatinine levels higher than 1.5 to 2.0 mg/100mL
b. renal hypertension
c. obesity
d. oliguria

A

d. oliguria

382
Q

A procedure involving catheterization of the ureters, followed by the direct instillation of contrast material into the renal pelves through the catheters is known as:

a. retrograde urography
b. renal angiography
c. voiding cytography
d. percutaneous antegrade pyelography

A

a. retrograde urography

383
Q

Cystography is of paramount importance in evaluation of:

a. retrocaval ureters
b. bladder diverticuli
c. bladder calculi
d. bladder rupture

A

d. bladder rupture

384
Q

Which of the following is not one of the phases of a renal angiography study?

a. arterial
b. pyelographic
c. nephrographic
d. venous

A

b. pyelographic

385
Q

Some consider crossed renal ectopia a variant of the horseshoe kidney. The ectopic kidney lies above the pelvis but on the wrong side. At what frequency is the ectopic kidney fused with the normal kidney?

a. Never
b. 30%
c. 60%
d. 90%

A

d. 90%

386
Q

The most prominent feature of the normal kidney on ultrasonography due to its echogenicity is:

a. central renal sinus
b. peripheral cortex
c. renal veins
d. renal calyces

A

a. central renal sinus

387
Q

A supranumerary kidney is:

a. excessive rotation of a kidney, resulting in obstruction in a small percentage of cases
b. a cortical nodule on the lateral margin of the kidney resulting from the presence of more cortical tissue than usual in a portion of the kidney parenchyma
c. the presence of a second anomalous kidney on one side of the body with demonstration of a separate pelvis, ureter and blood supply
d. enlargement of the calyces in one or both kidneys associated with underdeveloped renal pyramids

A

c. the presence of a second anomalous kidney on one side of the body with demonstration of a separate pelvis, ureter and blood supply

388
Q

Which of the following statements regarding horseshoe kidneys is true?

a. fusion most commonly is located at the superior pole of the kidneys
b. the ureters usually descend anteriorly to the renal structures due to restriction of normal
kidney medial rotation
c. renal arteries from the aorta are singular (i.e. one kidney-one artery)
d. the bridging isthmus is usually anterior to the inferior vena cava and aorta

A

b. the ureters usually descend anteriorly to the renal structures due to restriction of normal kidney medial rotation

389
Q

A kidney that crosses the midline is known as a(n):

a. ectopic kidney
b. subdiaphragmatic kidney
c. pelvic kidney
d. kidney malrotation

A

a. ectopic kidney

390
Q

A cystic renal mass presenting with thick irregular calcifications, enhancing septa and a multilocular appearance falls into which catagory?

a. I
b. II
c. III
d. IV

A

c. III

  • categories III and IV are highly suspicious for malignancy
  • category II should be followed up after ~6 months
391
Q

It is thought that upper urinary tract calculi originate as ______________ deep in the lining of the collecting ducts in the renal papillae which may detach and pass into the renal collecting system.

a. areas of nodular heterotopia
b. areas of acute focal pyelonephritis
c. Brucellosis infectious foci
d. Randall’s plaques

A

d. Randall’s plaques

392
Q

An Ask-Upmark kidney represents focal, segmental renal ____________ ?

a. hypoplasia
b. hyperplasia
c. multicystic dysplastic kidney
d. megacalycosis

A

a. hypoplasia

393
Q

Which of the following statements regarding renal calculi is correct?

a. renal calculi are most commonly found in Asian individuals
b. most renal calculi manifest between the ages of 50 and 70 years of age
c. renal calculi are bilateral in up to 15% of patients
d. the younger the patient is at the time of onset, the less likely the stone disease is to be recurrent

A

c. renal calculi are bilateral in up to 15% of patients

394
Q

A Proteus mirabilis infection of the urinary tract most commonly results in the production of
which of the following types of renal calculi?

a. xanthine stones
b. cystine stones
c. calcium hydrogen phosphate stones
d. magnesium ammonium phosphate stones

A

d. magnesium ammonium phosphate stones

395
Q

A parenchymal renal mass that appears quite radiolucent (compared with the adjacent parenchyma) and is sharply demarcated from the renal parenchyma, often appearing as a beak-like deformity or ‘claw sign’ is characteristic of:

a. simple renal cyst
b. renal angiomyolipoma
c. urinoma
d. parapelvic cyst

A

a. simple renal cyst

396
Q

The form of cystic disease involving the medulla of the kidney is:

a. simple renal cyst
b. polycystic kidney disease
c. medullary sponge kidney
d. multicystic dysplastic kidney

A

c. medullary sponge kidney

397
Q

Which of the following statements is correct regarding the renal disease characterized by the presence of grossly elongated, dilated collecting ducts throughout the renal parenchyma which resemble a cluster of straws when seen on end?

a. ureteral strictures are a common associated complication
b. the surface of the kidney remains smooth
c. the disease develops secondarily to a pyelonephritic infection
d. associated changes may include cystic involvement of the liver

A

d. associated changes may include cystic involvement of the liver

398
Q

Rupture of berry aneurysms of the arteries at the base of the brain is associated with which renal cystic disease?

a. autosomal recessive polycystic kidney disease
b. autosomal dominant polycystic kidney disease
c. medullary sponge kidney
d. multicystic dysplastic kidney

A

b. autosomal dominant polycystic kidney disease

399
Q

What is the primary treatment for renal cell carcinoma?

a. radiation therapy
b. chemotherapy
c. hormonal therapy with medroxyprogesterone acetate
d. radical nephrectomy

A

d. radical nephrectomy

400
Q

Eighty percent of patients with ___________ develop angiomyolipomas, which are usually multiple and bilateral.

a. Alport syndrome
b. Ask-Upmark kidneys
c. Sturge-Weber syndrome
d. Tuberous sclerosis

A

d. Tuberous sclerosis

401
Q

A patient presents with hypertension and a small, 2cm renal mass. Electron microscopic findings reveal tumor cells containing protogranules. Sonography reveals a solid echogenic mass. Renal vein catheterization would reveal increased levels of _________ in the renal venous blood supply.

a. renin
b. angiotensin
c. uric acid
d. aspartate transaminase

A

a. renin

402
Q

Angiography would be most useful for the diagnostic evaluation of renal ________ :

a. leiomyoma
b. hemangioma
c. endometrioma
d. juxtaglomerular cell tumors

A

b. hemangioma

403
Q

What is the most common primary malignant tumor of the kidney?

a. adenocarcinoma
b. lymphoma
c. squamous cell carcinoma
d. transitional cell carcinoma

A

a. adenocarcinoma

404
Q

What is the most common abdominal neoplasm of infancy and childhood?

a. fetal renal harmartoma
b. adenocarcinoma
c. neuroblastoma
d. nephroblastoma

A

d. nephroblastoma

405
Q

A child presents with a large intrarenal tumor that distorts the calyces and pelvis, and impaired renal function. Ultrasonography shows a homogeneous, echogenic renal mass. Which of the following should be performed on this patient?

a. oblique lumbar views
b. skull series
c. chest series
d. pelvis series

A

c. chest series

406
Q

Which of the following represents a tumor of the renal pelvis?

a. lymphangioma
b. adenocarcinoma
c. nephroblastoma
d. squamous cell carcinoma

A

d. squamous cell carcinoma

407
Q

Transitional cell tumors account for what percentage of malignant tumors of the renal pelvis?

a. approximately 10%
b. approximately 50%
c. approximately 90%
d. approximately 99%

A

c. approximately 90%

408
Q

What is the most common source of metastases to the kidney?

a. lung
b. colon
c. bone
d. liver

A

a. lung

409
Q

Metastatic tumors to the kidney are _________ as common as primary renal tumors.

a. twice
b. three times
c. five times
d. ten times

A

a. twice

410
Q

A patient complains of acute flank pain, fever and chills. Radiographs reveal a loss of renal outline and psoas shadow, a scoliotic convexity, and immobilization of the kidney on respiration. Gas is noted within the interstitium of the renal parenchyma. Which of the
following conditions is likely to be associated with this renal pathology?

a. hypercholesterolemia
b. elevated renin-angiotensin levels
c. diabetes
d. pregnancy

A

c. diabetes

411
Q

What is the most common location of renal artery aneurysms?

a. bifurcation of the renal artery
b. renal capsular margin
c. segmental renal branches
d. distal renal arterioles

A

a. bifurcation of the renal artery

412
Q

Which of the following statements regarding renal artery occlusion is correct?

a. renal artery occlusion occurs most commonly as a result of trauma
b. renal function loss is irreversible
c. retrograde pyelography reveals a decrease in the calyceal system size
d. involvement is always unilateral

A

c. retrograde pyelography reveals a decrease in the calyceal system size

413
Q

Renal vein thrombosis is found most commonly in which age group?

a. infants
b. children
c. middle-aged adults
d. older adults

A

b. children

414
Q

Renovascular hypertension and progressive renal failure may be associated with:

a. renal artery stenosis
b. retroaortic left-renal vein
c. bifurcated renal artery
d. polyarteritis nodosa

A

a. renal artery stenosis

415
Q

Renal infarcts are associated with:

a. multilocular cystic nephroma
b. renal angiomyolipoma
c. chronic glomerulonephritis
d. polyarteritis nodosa

A

d. polyarteritis nodosa

416
Q

What is the renal necrotic disease that may be associated with a number of antecedent conditions such as severe burns, multiple fractures, internal hemorrhage, or severe infections?

a. perirenal abscess
b. renal papillary necrosis
c. renal medullary necrosis
d. bilateral cortical necrosis

A

d. bilateral cortical necrosis

417
Q

The abuse of analgesics over prolonged periods is associated with a chronic form of this
disease?

a. perirenal abscess
b. renal papillary necrosis
c. renal medullary necrosis
d. bilateral cortical necrosis

A

b. renal papillary necrosis

418
Q

Following renal trauma, which imaging technique offers physiologic information on renal
function?

a. doppler ultrasound
b. retrograde pylogram
c. radionuclide urogram
d. abdominal CT

A

c. radionuclide urogram

419
Q

All of the following conditions are associated with nephrocalcinosis but one. Choose one of the following:

a. Hyperparathyroidism
b. Multiple myloma
c. Cushings disease
d. Gout

A

d. Gout

420
Q

A rent in the renal collecting system or kidney can result in persistent extravasation of urine or blood into the perirenal space. This mass, which often displaces the kidney upward and rotates it laterally is known as a(n):

a. calycine cyst
b. urinoma
c. parapelvic cyst
d. renal vein thrombosis

A

b. urinoma

421
Q

Which of the following is a complication of an IVP?

a. Cortical necrosis
b. Acute renal failure
c. Hypovolemic shock
d. Thrombosis

A

b. Acute renal failure

422
Q

Which of the following diseases presents with hemoptysis from pulmonary hemorrhage and glomerulonephritis?

a. Group A Beta-hemolytic streptoccocus infection
b. Goodpasture’s disease
c. Hemophilia
d. Cirrhosis of the liver

A

b. Goodpasture’s disease

423
Q

Which of the following is considered to be the best means of evaluating the kidney?

a. Computer tomography
b. Intravenous pylogram
c. Ultrasound
d. Based on suspected condition

A

d. Based on suspected condition

424
Q

Which of the following is the most common pathogen responsible for the development of a renal abscess?

a. Amebiasis
b. Staphylococcus aureus
c. E.coli
d. Cryptococcus

A

b. Staphylococcus aureus

425
Q

Renal papillary necrosis is now recognized to be ischemic in origin rather than infectious. However, it can occur in the context of infection. Which of the following is a condition in which renal papillary necrosis can be seen?

a. Diabetes mellitus
b. Ulcerative colitis
c. Xanthogranulomatous Pyelonephritis
d. Tuberculosis

A

a. Diabetes mellitus

426
Q

What is the maximal normal difference between the length of both kidneys, as seen on a radiograph?

a. 1.5 cm
b. 2.5 cm
c. 3.5 cm
d. 4.5 cm

A

a. 1.5 cm

427
Q

Which of the following choices can be helpful in distinguishing Wilm’s tumor from neuroblastoma?

a. Calcifications are more commonly seen in Wilm’s tumor
b. Calcifications are more commonly seen in neuroblastoma
c. The majority of neuroblastomas are seen in children over 5 years of age
d. The majority of Wilm’s tumors are seen in infants soon after birth

A

b. Calcifications are more commonly seen in neuroblastoma

428
Q

The most common type of fusion anomaly involving the kidneys is…

a. Crossed ectopic with fusion
b. Pancake kidney
c. Horseshoe kidney
d. Waffle kidney

A

c. Horseshoe kidney

429
Q

Which of the following is not a characteristic of medullary sponge kidney?

a. Absence of hematuria
b. Calculi are frequently found in the cysts
c. Function is usually preserved
d. Changes are confined to the renal medulla

A

a. Absence of hematuria

430
Q

All of the following are causes of nephrocalcinosis except…

a. Renal tubular acidosis
b. Sarcoidosis
c. Hypothyroidism
d. Wilson’s disease

A

c. Hypothyroidism

431
Q

Angiomyolipomas are usually multiple when relating to…

a. Muticystic kidney disease
b. Goodpasture’s syndrome
c. Medullary sponge kidney
d. Tuberous sclerosis

A

d. Tuberous sclerosis

432
Q

The most common abdominal neoplasm of infancy and childhood is…

a. Wilm’s tumor
b. Neuroblastoma
c. Pancreatic carcinoma
d. Colon carcinoma

A

a. Wilm’s tumor

433
Q

What percentage of renal collecting systems are bifid and/or duplicated?

a. 1%
b. 10%
c. 25%
d. none of the above

A

b. 10%

434
Q

Which of the following is not true concerning renal cell carcinoma (RCC)?

a. accounts for 85% of all renal neoplasms
b. up to 10% of cases present as cystic or multicystic forms
c. up to 10% of cases show stippled calcification
d. up to 10% of cases are bilateral

A

d. up to 10% of cases are bilateral

(only 2% of cases are bilateral)

435
Q

Which of the following is correct concerning renal cell carcinoma?

a. 30% involve the renal vein
b. 10% extend into the inferior vena cava
c. 40% have metastases at diagnosis
d. all of the above are correct

A

d. all of the above are correct

436
Q

Which of the following is false concerning angiomyolipoma?

a. benign mesenchymal tumor representing 1 to 3% of renal neoplasms
b. 80% are solitary and unilateral
c. 20% are associated with tuberous sclerosis
d. rarely associated with hemorrhage

A

d. rarely associated with hemorrhage

(angiomyolipomas are prone to hemorrhage due to thin-wall of vessels)

437
Q

A solid renal tumor demonstrating fat density in the absence of calcification is diagnostic for which of the following:

a. renal cell carcinoma
b. angiomyolipoma
c. oncocytoma
d. xanthogranulomatous pyelonephritis

A

b. angiomyolipoma

438
Q

Which of the following is not true concerning oncocytoma?

a. represents 3 to 6% of renal neoplasms
b. associated with a “spoke-wheel” configuration of vessels
c. hemorrhage and necrosis are common
d. most are solitary

A

c. hemorrhage and necrosis are common

(hemorrhage and necrosis are rare)

439
Q

T or F: the kidney is commonly involved by metastatic lymphoma and most cases are non-Hodgkin type.

A

TRUE

440
Q

Which of the following is NOT true concerning xanthogranulomatous pyelonephritis?

a. associated with presence of a staghorn calculus
b. associated with chronic infection of proteus mirabilis
c. renal parenchyma contains lipid-laden macrophages
d. none of the above

A

d. none of the above

441
Q

Which of the following is not true concerning simple renal cysts?

a. they are found in half the population older than 55 yoa
b. they are commonly multiple and bilateral
c. they show edge enhancement with contrast administration
d. they are the most common type of renal mass

A

c. they show edge enhancement with contrast administration

(show no enhancement with contrast administration)

442
Q

Which of the following is true concerning multilocular cystic nephroma?

a. found in males under 4 yoa
b. found in females age 40 to 60
c. all of the above are correct
d. none of the above are correct

A

c. all of the above are correct

443
Q

Which of the following disease is characterized by multiple noncommunicating cysts that progressively replace renal parencyma?

a. autosomal dominant polycystic disease
b. autosomal recessive polycystic disease
c. tuberous sclerosis
d. Von Hippel-Lindau disease

A

a. autosomal dominant polycystic disease

444
Q

Which of the following is false concerning autosomal dominant polycystic disease?

a. cysts are rarely complicated by internal hemorrhage
b. 60% of cases have associated cysts in the liver
c. 10% of cases have associated cysts in the pancreas
d. 20% of cases have associated intracranial aneurysms

A

a. cysts are rarely complicated by internal hemorrhage

(cysts are commonly complicated by internal hemorrhage)

445
Q

Which of the following diseases is assoicated with pheochromocytomas, multiple and bilateral renal cell carcinomas, retinal angiomas and cerebellar hemangioblastomas?

a. autosomal dominant polycystic disease
b. autosomal recessive polycystic disease
c. Von Hippel-Lindau disease
d. tuberous sclerosis

A

c. Von Hippel-Lindau disease

446
Q

Which of the following diseases is characterized by multiple renal cysts and multiple angiomyolipomas along with hamartomas elsewhere?

a. autosomal dominant polycystic disease
b. autosomal recessive polycystic disease
c. tuberous sclerosis
d. Von Hippel-Lindau disease

A

c. tuberous sclerosis

447
Q

Which of the following diseases is associated with long-term hemodialysis?

a. uremic medullary cystic disease
b. Tuberous sclerosis
c. acquired uremic cystic kidney disease
d. multiple simple cysts

A

c. acquired uremic cystic kidney disease

448
Q

Which of the following diseases is the result of a diffuse dilation of the collecting tubules?

a. autosomal dominant polycystic disease
b. autosomal recessive polycystic disease
c. Von Hippel-Lindau disease
d. Acquired uremic cystic kidney disease

A

b. autosomal recessive polycystic disease

449
Q

Which of the following disease is characterized by dysplastic dilation of the collecting tubules in
the papilla in a cylindric or saccular configuration?

a. uremic medullary cystic disease
b. medullary sponge kidney
c. multicystic dysplastic kidney
d. tuberous sclerosis

A

b. medullary sponge kidney

450
Q

Which of the following diseease is characterized by progressive tubular atrophy with glomerular
sclerosis and formation of medullary cysts?

a. acquired uremic cystic kidney disease
b. medullary sponge kidney
c. uremic medullary cystic disease
d. tuberous sclerosis

A

c. uremic medullary cystic disease

451
Q

Which of the following is associated with an atretic ureter and calcification?

a. multicystic dysplastic kidney
b. tuberous sclerosis
c. medullary sponge kidney
d. Von Hippel-Lindau disease

A

a. multicystic dysplastic kidney

452
Q

Which of the following is associated with bilateral enlargement of the kidneys?

a. medullary sponge kidney
b. multicystic dysplastic kidney
c. autosomal recessive polycystic disease
d. none of the above

A

c. autosomal recessive polycystic disease

453
Q

Which of the following is characterized by relative preservation of kidney size?

a. autosomal dominant polycystic disease
b. medullary sponge kidney
c. muticystic dysplastic kidney
d. autosomal recessive polycystic disease

A

b. medullary sponge kidney

454
Q

Which of the following is false concerning chronic pyelonephritis?

a. vesicoureteral reflux of infected urine is the most common cause in children
b. calculi and chronic obstruction are the most common cause in adults
c. the hallmark is a focal cortical scar that overlies a blunted calyx
d. all of the above are true

A

d. all of the above are true

455
Q

Parenchymal destruction and cavity fomation, multiple parenchymal masses, strictures of the collecting system and ureters, and calcification are the hallmarks for which of the following diseases?

a. renal tuberculosis
b. reflux nephropathy
c. renal cell carcinoma
d. chronic pyelonephritis

A

a. renal tuberculosis

456
Q

Which of the following is not true concerning renal tuberculosis?

a. may follow primary pulmonary tuberculosis by as much as 10 to 15 years
b. active pulmonary tuberculosis is present in only 10% of cases
c. 30% of cases show no radiographic evidence of previous pulmonary infection on chest films
d. all of the above are true

A

d. all of the above are true

457
Q

Which of the following statements is false concerning nephrocalcinosis?

a. cortical nephrocalcinosis is more common than medullary nephrocalcinosis
b. cortical nephrocalcinosis is usually related to cortical necrosis or chronic glomerulonephritis
c. medullary nephrocalcinosis is usually related to hypercalcemic or hypercalciuric states
d. all of the above are false

A

a. cortical nephrocalcinosis is more common than medullary nephrocalcinosis

(only 5% of cases are cortical nephrocalcinosis)

458
Q

Gradient-echo chemical shift MR imaging of the adrenal glands demonstrate increased signal intensity on the out-of-phase image relative to the in-phase. Which of the following is the most likely diagnosis with these findings?

a. Myelolipoma
b. Adenoma
c. Metastasis
d. Pheochromocytoma

A

c. Metastasis

459
Q

About ___ of patients with Cushing’s syndrome have hyperplastic adrenals, and ___ have an adenoma or carcinoma.

a. 25%; 75%
b. 50%; 50%
c. 75%; 25%
d. 60%; 5%

A

c. 75%; 25%

460
Q

What is the most common malignancy in infants and children?

a. Neuroblastoma
b. Nephroblastoma
c. Nephroblastomatosis
d. Wilm’s Tumor

A

a. Neuroblastoma

461
Q

Which of the following statements best defines the Weigert-Meyer rule?

a. A simple ureterocele consists of an intravesical dilation of the ureter proximal and adjacent to its connection with the bladder and resembles a cobra head.
b. A jet of opaque medium propelled by ureteral peristalsis that maintains the caliber of the ureter and when present excludes the possibility of obstruction or reflux.
c. In regards to duplication of the pelvis and ureter, the ureter that drains the upper pole is ventral to the lower one but crosses over and empties into the bladder in a more medial ectopic location via a ureterocele.
d. Refers to the mild asymmetry that can exist between the number of major and minor calyces of the kidneys, as long as the difference is not more than five form one side to the other.

A

c. In regards to duplication of the pelvis and ureter, the ureter that drains the upper pole is ventral to the lower one but crosses over and empties into the bladder in a more medial ectopic location via a ureterocele.

462
Q

Which of the following is the most common type of “backflow”, and actually represents papillary
tubular stasis rather than actual backflow?

a. Pyelointerstitial
b. Pyelotubular
c. Pyelolymphatic
d. Pyelovenous

A

b. Pyelotubular

463
Q

Which two conditions of the kidney are associated with tuberous sclerosis?

a. Poycystic kidney disease
b. Simple renal cyst
c. Medullary sponge kidney
d. Angiomyolipoma

A

d. Angiomyolipoma

464
Q

What is the most common malignancy of the GI tract?

a. colorectal adenocarcinoma
b. gastric adenocarcinoma
c. appendiceal carcinoid
d. esophageal squamous cell carcinoma

A

a. colorectal adenocarcinoma

(2nd most common of all malignant tumors in the US)

465
Q

The most common portion of the colon to be affected by adenocarcinoma is:

a. rectosigmoid region
b. descending colon
c. transverse colon
d. ascending colon

A

a. rectosigmoid region (50%)

466
Q

Which of the following is NOT true concerning colorectal adenocarcinoma?

a. nearly all develop from preexisting adenomas
b. 20% of patients have a second tumor at diagnosis
c. most tumors present as “apple-core” lesions
d. the most frequent complication is perforation

A

d. the most frequent complication is perforation

(most common complication is obstruction)

467
Q

Which of the following colon polyp sizes is incorrectly matched with the malignant potential?

a. less than 5mm - .5%
b. 5 to 10 mm - 1%
c. 10 to 20 mm - 25%
d. greater than 20 mm - 50%

A

c. 10 to 20 mm - 25%

(true incidence is 10%)

468
Q

Which statement concerning rectal polyps is incorrect?

a. inflammatory polyps account for less than .5%
b. hamartomatous polyps are a common cause for rectal bleeding in children
c. adenomatous polyps are found in 1% of the population over 40 years of age
d. nearly all hyperplastic polyps are smaller than 5 mm

A

c. adenomatous polyps are found in 1% of the population over 40 years of age

(true incidence is 5-10% of the populations over 40 yoa)

469
Q

Which statement is NOT true concerning colorectal tumors?

a. non-hodgkins is the most common lymphoma to affect the colon
b. the colon accounts for only 7% of GISTs
c. lipoma is the most common submucosal tumor of the colon
d. colon is more commonly involved in lymphoma than the stomach

A

d. colon is more commonly involved in lymphoma than the stomach

(colon is less commonly affected than stomach or small bowel)

470
Q

What percentage of lipomas present with intussusception?

a. 20%
b. 40%
c. 60%
d. 80%

A

b. 40%

471
Q

Which characteristic listed below is not associated with ulcerative colitis?

a. starts distally and moves proximally through the colon
b. rarely involves the terminal ileum
c. eccentric disease
d. risk of toxic megacolon

A

c. eccentric disease

(ulcerative colitis is usually circumferential)

472
Q

Which of the following characteristics is not associated with Crohn disease of the colon?

a. pseudodiverticula formation
b. fistula formation
c. rectal sparing
d. collar button ulcers

A

d. collar button ulcers

(collar button ulcers seen in UC)

473
Q

Which sign or characteristic appearance is not correctly matched with a disease?

a. accordion sign - pseudomembranous colitis
b. pneumatosis coli with marked dilation - toxic megacolon
c. appearance of Crohn colitis without terminal ileum involvement - amebiasis
d. thumbprinting and target sign - typhilitis

A

d. thumbprinting and target sign - typhilitis

(thumbprinting and target sign associated with ischemic colitis)

(typhilitis shows wall thickening in cecum and ascending colon with prominent pericolonic inflammatory changes)

474
Q

Which of the following is false concerning colon diverticulosis?

a. most common in the sigmoid colon
b. occurence of diverticula increases with age
c. colon diverticula are true diverticula
d. associated with low-fiber diet

A

c. colon diverticula are true diverticula

475
Q

Which of the following is true concerning diverticulitis?

a. complicates nearly all cases of diverticulosis
b. is a more common cause of obstruction than colon carcinoma
c. colon lumen is narrowed with abrupt margins
d. may show a “double track” sign

A

d. may show a “double track” sign

*Corrections to the false statements:

complicates only 20% of cases of diverticulosis
less common cause of obstruction than colon carcinoma
colon lumen is narrowed with tapered margins

476
Q

What is the most common cause of lower GI hemorrhage?

a. colon diverticula
b. angiodysplasia
c. colon carcinoma
d. polyps

A

a. colon diverticula

477
Q

Match the polyposis syndrome with its related finding(s)

Peutz-Jeghers Syndrome

A. osteomas of the skull, supernumerary teeth and dermal fibromas
B. tumors of the CNS
C. dark pigmented spots on the skin and mucous membranes
D. goiter and thyroid adenomas, facial papules, oral papillomas and palmoplantar keratoses
E. nail atrophy, brownish skin pigmentation and alopecia

A

C. dark pigmented spots on the skin and
mucous membranes

478
Q

Match the polyposis syndrome with its related finding(s)

Cronkhite-Canada Syndrome

A. osteomas of the skull, supernumerary teeth and dermal fibromas
B. tumors of the CNS
C. dark pigmented spots on the skin and mucous membranes
D. goiter and thyroid adenomas, facial papules, oral papillomas and palmoplantar keratoses
E. nail atrophy, brownish skin pigmentation and alopecia

A

E. nail atrophy, brownish skin pigmentation
and alopecia

479
Q

Match the polyposis syndrome with its related finding(s)

Gardner Syndrome

A. osteomas of the skull, supernumerary teeth and dermal fibromas
B. tumors of the CNS
C. dark pigmented spots on the skin and mucous membranes
D. goiter and thyroid adenomas, facial papules, oral papillomas and palmoplantar keratoses
E. nail atrophy, brownish skin pigmentation and alopecia

A

A. osteomas of the skull, supernumerary
teeth and dermal fibromas

480
Q

Match the polyposis syndrome with its related finding(s)

Turcot Syndrome

A. osteomas of the skull, supernumerary teeth and dermal fibromas
B. tumors of the CNS
C. dark pigmented spots on the skin and mucous membranes
D. goiter and thyroid adenomas, facial papules, oral papillomas and palmoplantar keratoses
E. nail atrophy, brownish skin pigmentation and alopecia

A

B. tumors of the CNS

481
Q

Match the polyposis syndrome with its related finding(s)

Cowden Syndrome

A. osteomas of the skull, supernumerary teeth and dermal fibromas
B. tumors of the CNS
C. dark pigmented spots on the skin and mucous membranes
D. goiter and thyroid adenomas, facial papules, oral papillomas and palmoplantar keratoses
E. nail atrophy, brownish skin pigmentation and alopecia

A

D. goiter and thyroid adenomas, facial
papules, oral papillomas and palmoplantar
keratoses

482
Q

What is the most common cause of large bowel obstruction?

a. Fecal impaction
b. Ischemia
c. Primary malignancies of the colon
d. Metastatic disease

A

c. Primary malignancies of the colon

483
Q

Which of the following special imaging techniques is best utilized for the diagnosis of a cavernous hemangioma of the liver?

a. Spiral computed tomography with the administration of intravenous contrast and images taken in a hepatic arterial phase and a portal venous phase.
b. Catheterized administration of contrast into the SMA with computed tomography.
c. Magnetic resonance imaging of the liver utilizing T1 and T2 weighted images.
d. Radionuclide imaging of the liver.

A

a. Spiral computed tomography with the administration of intravenous contrast and images taken in a hepatic arterial phase and a portal venous phase.

484
Q

A computed tomographic examination of the abdomen demonstrates low attenuation of the entire liver parenchyma with respect to the spleen and lack of mass effect on the intrahepatic vessels. What is the most likely diagnosis for these findings?

a. Chronic cirrhosis of the liver
b. Chronic hepatitis
c. Diffuse fatty infiltration of the liver
d. Hemochromotosis of the liver

A

c. Diffuse fatty infiltration of the liver

485
Q

A CT examination was performed on a 35 year old male patient with a history of alcoholism. Findings include multiple low attenuation nodules ranging from 3-10mm in diameter without evidence of a capsule or calcifications. Laboratory findings
demonstrate only mildly elevated liver function tests and the presence of alpha-fetoprotein was negative. What is the most likely diagnosis for the this patient?

a. Cirrhosis of the liver
b. Hepatocellular carcinoma
c. Fibrolamellar carcinoma
d. Focal nodular hyperplasia

A

a. Cirrhosis of the liver

486
Q

What is the most common malignant mass found in the liver?

a. Metastases
b. Hepatocellular carcinoma
c. Lymphoma
d. Kaposi’s sarcoma

A

a. Metastases

487
Q

What is the most common benign neoplasm found in the liver?

a. Focal nodular hyperplasia
b. Hepatic adenoma
c. Cavernous hemangioma
d. Simple hepatic cyst

A

c. Cavernous hemangioma

488
Q

An MRI evaluation of the abdomen demonstrates a heterogenous cystic lesion within the right liver lobe, therefore aspiration was performed to confirm the diagnosis. The extravasated material consited of an “anchovy paste” material. What was the most
likely diagnosis?

a. Echinococcus cyst
b. Pyogenic abcess
c. Amebic abcess
d. Infected hepatic cyst

A

c. Amebic abcess

489
Q

Which of following signs demonstrated on ultrasound of the liver is not associated with hydatid cyst disease?

a. the “beak” sign
b. the “racemose” appearance
c. the “water lily” sign
d. the “double line” sign

A

a. the “beak” sign

490
Q

A 40 year old male visits your office complaining of right upper quadrant pain since he returned from his vacation in Mexico. After a thorough history and examination, you discover that he has hepatitis B, has lost 25 pounds this month, and presents with ascites. What is your working diagnosis and what should you do next?

a. Acute pancreatitis - Do no imaging have your receptionist call 911 and have an ambulence come get him.
b. Cholecystitis - KUB and laboratory diagnostics to include a liver function panel and direct/indirect billirubin.
c. Hepatoma - MRIof the abdomen and laboratory diagnostics to include a liver function panel and alpha-fetoprotein.
d. Liver metastases - CT portography and laboratory diagnostics to include a CBC and liver function panel.

A

c. Hepatoma - MRIof the abdomen and laboratory diagnostics to include a liver function panel and alpha-fetoprotein.

491
Q

A CT examination was performed on a patient that has a history of multiple blood transfusions. Findings consisted of a diffuse increase in attenuation of the liver and hepatomegaly. What is the most likely diagnosis?

a. Gaucher’s disease
b. Diabetes Mellitus
c. Hemochromotosis
d. Hepatocellular carcinoma

A

c. Hemochromotosis

492
Q

Which of the following is caused by obstruction to the hepatic venous outflow and presents with a characteristis “flip-flop” pattern on contrast enhanced CT?

a. Gamna-Gandy bodies
b. Kaposi’s sarcoma
c. Budd-Chiari Syndrome
d. Gaucher’s disease

A

c. Budd-Chiari Syndrome

493
Q

What is the most common malignant mass in the liver?

a. metastatic disease
b. HCC
c. lymphoma
d. adenocarcinoma

A

a. metastatic disease

(20x more common than primaries)

494
Q

Which is not a common site of origin in hepatic metastasis?

a. gi tract
b. bone
c. breast
d. lung

A

b. bone

495
Q

Which of the following is true concerning cavernous hemangioma of the liver?

a. most common cause of liver mass
b. more common in men
c. most common benign liver neoplasm
d. most lesions are larger than 6 cm

A

c. most common benign liver neoplasm

*Corrections to false statements:

  • the 2nd most common liver mass behind mets
  • more common in women
  • most lesions are small than 5cm
496
Q

Which is not a major growth pattern of HCC?

a. diffuse infiltrative
b. focal infiltrative
c. solitary massive
d. multinodular

A

b. focal infiltrative

497
Q

What percentage of HCC are surrounded by a fibrous capsule?

a. 25%
b. 50%
c. 75%
d. 100%

A

a. 25%

498
Q

Calcification occurs in what percentage of HCC’s?

a. <1%
b. 10%
c. 20%
d. 85%

A

b. 10%

499
Q

T/F: Most HCC tumors are hypervascular and demonstrate marked arterial phase enhancement.

A

TRUE

500
Q

Which of the following is not a characteristic imaging finding of HCC?

a. invasion of portal and hepatic veins
b. sharply marginated, enhancing tumor capsule
c. mosaic appearance of the tumor on advanced imaging
d. decreased alpha-fetoprotein

A

d. decreased alpha-fetoprotein

501
Q

Which is FALSE concerning focal nodular hyperplasia (FNH)?

a. more common in women
b. necrosis and hemorrhage are common
c. contain a central fibrous scar
d. show normal or increased activity on sulfur colloid scans

A

b. necrosis and hemorrhage are common

(necrosis and hemorrhage are very, very uncommon)

502
Q

Which of the following is true concerning hepatic adenomas?

a. shows increased sulfur colloid activity
b. rarely associated with hemorrhage
c. most tumors are solitary, smooth and encapsulated
d. most common in women, but not related to oral contraceptive use

A

c. most tumors are solitary, smooth and encapsulated

*Corrections to false statements

  • shows decreased activity because kupffer cells are non-functional
  • known for hemorrhage due to poor connective tissue support
  • are more common in women and ARE related to oral contraceptive use
503
Q

Which is not true concerning fibrolamellar carcinoma?

a. more common in young adults
b. alpha-fetoprotein levels are usually normal
c. central scar with radiating septa is the characteristic appearance
d. hemorrhage and necrosis are common

A

d. hemorrhage and necrosis are common

(uncommon, only 10%)

504
Q

Which is true concerning benign hepatic cysts?

a. most common benign hepatic mass
b. cysts communicate with biliary tree
c. usually solitary
d. show as low on T1 and high on T2 with enhancement following Gad

A

c. usually solitary

*Corrections to false statements:

  • 2nd most common benign hepatic mass behind cavernous hemangioma
  • don’t connect to biliary tree
  • no enhancement with Gad
505
Q

Which is not true concerning pyogenic abcess in the liver?

a. caused by E. coli, S. aureus or streptococcus
b. peripheral rim enhancement seen with contrast
c. gas seen in lesion 20% of time
d. patients are frequently asymptomatic

A

d. patients are frequently asymptomatic

506
Q

Anchovy paste material found at biopsy is indicative of what diagnosis?

a. HCC
b. amebic abcess
c. pyogenic abcess
d. hydatid cyst

A

b. amebic abcess

507
Q

Which organ is most frequenty involved in echinococcal cysts?

a. liver
b. spleen
c. kidney
d. adrenals

A

a. liver

508
Q

Schistosomiasis is an infectious disease affecting more than 100 million people world wide, especially in tropical and subtropical countries. The organism matures in the body of snails were it then emerges into the water. Ingesting the water infects a human. Were does this organism reside within the human (possibly for 20-30 years)?

a. Small intestine
b. Large intestine
c. Portal venous system
d. Subcutaneously

A

c. Portal venous system

509
Q

A rare congenital cystic dilation of the intrahepatic ducts is known as:

a. Rokitansky-Aschoff disease
b. Klatskin tumor
c. Caroli’s disease
d. Wirsung’s disease

A

c. Caroli’s disease

510
Q

A Klatskin’s tumor is a ____________ tumor localized at the junction of the right and left hepatic ducts.

a. adenocarcinoma
b. cholangiocarcinoma
c. squamous cell carcinoma
d. cholangiosarcoma

A

b. cholangiocarcinoma

511
Q

Characteristically, the liver disorder associated with ulcerative colitis is which of the following?

a. pericholangitis
b. amebiasis
c. Caroli’s disease
d. portal vein hypertension

A

a. pericholangitis

512
Q

What is the name of the numbering system for hepatic segments?

a. couinaud
b. boyden
c. budd-chiari
d. caroli

A

a. couinaud

Eight segments:

caudate - I
left lateral superior - II
left lateral inferior - III
left medial - IVa and IVb
right anterior inferior - V
right posterior inferior - VI
right posterior superior - VII
right anterior superior - VIII

513
Q

Which of the following is not true regarding the liver on advanced imaging?

a. normal liver parenchyma is equal to or greater than the density of the spleen on non-enhanced CT
b. normal liver parenchyma enhancement is greater than that of the spleen during the arterial phase of CT with contrast
c. normal liver parencyma enhancement is equal to or greater than that of the spleen during the venous phase of CT with contrast
d. normal liver parencyma is of slightly higher signal intensity than the spleen on T1 MR

A

b. normal liver parenchyma enhancement is greater than that of the spleen during the arterial phase of CT with contrast

(enhancement is less than spleen owing to the liver getting most of its blood from portal venous system)

514
Q

Which is not an indication for fatty infiltration of the liver?

a. lower attenuation of liver parenchyma vs spleen on non-contrast CT
b. bulging liver contour with hepatic vessel displacement
c. increased echogenicity on US
d. loss of signal on out-of-phase vs. in-phase gradient echo MR

A

b. bulging liver contour with hepatic vessel displacement

515
Q

The most common segment involved in focal sparing in a diffuse fatty infiltration is?

a. caudate
b. anterior segment
c. posterior segment
d. medial segment

A

d. medial segment

(segment IV specifically)

516
Q

Which of the following is true concerning cirrhosis?

a. most common cause in US is hepatitis
b. most common cuase in Asia and Africa is alcoholism
c. can be caused by Wilson’s disease
d. hepatomegaly is seen in late disease

A

c. can be caused by Wilson’s disease

*Corrections to false statements:

  • m/c/c in US is alcoholism
  • Asia and Africa is hepatitis
  • hepatomegaly seen in early disease, atrophy seen in late disease
517
Q

Which is NOT true concerning regenerative nodules in cirrhosis?

a. may contain deposits of iron
b. variable appearance on T1 images but don’t show early enhancement with Gad
c. hyperintense on T2 images
d. may be same or higher attenuation vs normal tissue on unenhanced CT

A

c. hyperintense on T2 images

518
Q

Which is NOT true concerning portal hypertension?

a. causes downhill varices in the esophagus
b. causes include cirrhosis, portal vein thrombosis and schistosomiasis
c. associated with internal rectal hemorrhoids
d. indicated by a portal vein diameter greater than 13 mm

A

a. causes downhill varices in the esophagus

(causes uphill varices in the esophagus)

519
Q

Which is NOT true concerning budd-chiari syndrome?

a. caused by obstruction of portal venous outflow
b. primary form caused by suprahepatic IVC web
c. caudate is frequently involved in the secondary type
d. comma sign seen on CT and MR

A

c. caudate is frequently involved in the secondary type

(caudate usually spared due to separate venous drainage to IVC)

520
Q

Which is NOT true concerning hemochromotosis in the liver?

a. shows increased liver density to 75-130 HU on CT
b. MR shows high-low (T1-T2) signal pattern
c. chronic cases are at increased risk for HCC
d. may be primary or secondary

A

b. MR shows high-low (T1-T2) signal pattern

(liver is low-low on MR)

521
Q

Which of the following is true concerning gas in the portal venous system?

a. seen more peripherally compared to biliary tree gas
b. usually clinically insignificant in adults
c. usually clinically insignificant in infants
d. seen as streaky high attenuation areas on CT

A

a. seen more peripherally compared to biliary tree gas

(normally biliary gas not seen within 2 cm of peripheral edge of liver)

*Corrections to false statements:

  • usually sign of bowel ischemia/necrosis in adults
  • usually sign of _necrotizing entercoliti_s in infants
  • seen as low attenuation on CT (it’s gas for heaven’s sake)
522
Q

Which of the following statments is correct in regards to a normal variant that consists of an elongated inferior tip of the right lobe of the liver?

a. Referred to as Reidel’s and most commonly is seen in males.
b. Referred to as Reidel’s and is seen in children only.
c. Referred to as Reidel’s and most commonly is seen in females.
d. There is no variant such as this and this represents hepatomegaly.

A

c. Referred to as Reidel’s and most commonly is seen in females.

523
Q

Which of the following is part of a biphasic, CT intravenous contrast study of the liver?

a. Arterial phase
b. Cystic phase
c. nephrographic phase
d. Hepatic phase

A

a. Arterial phase

524
Q

What percentage of heaptic blood originates from the portal venous system?

a. 10%
b. 25%
c. 50%
d. 75%

A

d. 75%

525
Q

Which of the following represents an condition that consists of hepatic venous outflow obstruction secondary to various etiologies including congential webs, thrombosis, tumor and hypercoagulable states, and demonstrates a “flip-flop” pattern on
post-contrast CT with sparring of the caudate lobe.

a. Passive hepatic congestion
b. Budd-Chiari syndrome
c. Portal vein thrombosis
d. Portal hypertension

A

b. Budd-Chiari syndrome

526
Q

Which of the following is the most common benign heaptic lesion?

a. Hemangioma
b. Lipoma
c. Adenoma
d. Simple heaptic cyst

A

a. Hemangioma

527
Q

Which of the following is the second most common benign heaptic lesion?

a. Hemangioma
b. Lipoma
c. Adenoma
d. Simple hepatic cyst

A

d. Simple hepatic cyst

528
Q

What percentage of liver abscesses are bacterial or pyogenic in origin?

A. 15%

B. 50%

C. 75%

D. 85%

A

D. 85%

529
Q

Which of the following is the most common visceral malignancy worldwide?

a. Colorectal carcinoma
b. Lung cancer
c. Hepatocellular carcinoma
d. Breast cancer

A

c. Hepatocellular carcinoma

530
Q

Which of the following is not a characteristic of fibrolamellar carcinoma?

a. Low signal stellate scar on T2WI.
b. Calcifications in 55% of cases.
c. Elevated alpha fetoprotein levels.
d. Subtype of hepatoma that occurs in younger patients without risk factors for hepatoma.

A

c. Elevated alpha fetoprotein levels.

531
Q

Metastases are ____ more common than primary liver malignancies.

A. 5 times

B. 20 times

C. 30 times

D. 50 times

A

B. 20 times

532
Q

True or False

Macrocystic serous cystadenoma is not distinguishable on imaging studies from mucinous cystic neoplasm

B+H 3rd pg 789

A

True

533
Q

What percentage of patients with multiple hepatic cysts have polycystic kidney disease?

20%

40%

60%

80%

A

60%

Paul and Juhl’s, Essentials of Radiological Imaging, 7th ed., Lippincott, 1998, p.516.

534
Q

Which is not true concerning intraductal papillary mucinous neoplasm

a. most common in the uncinate process of the pancreas
b. shows marked dilation of the pancreatic duct
c. main duct type may appear as focal group of small cysts capsule
d. is formed by adjacent atrophied parenchyma

B+H 3rd pg 790

A

c. main duct type may appear as focal group of small cysts capsule

branch duct type may appear as focal group of small cysts

535
Q

The most common cause of pelvic calcification, in a female patient, is…

a. Atherosclerotic calcification
b. Phleboliths
c. Ovarian cysts
d. Bladder stones

Baker, pg.

A

b. Phleboliths

536
Q

Which of the following would most likely demonstrate a “camalote sign”?

A. Simple hepatic cyst

B. Post-traumatic heaptic cyst

C. Hepatic hemangioma

D. Echinococcal cyst

Paul and Juhl’s, Essentials of Radiological Imaging, 7th ed., Lippincott, 1998, p.517.

A

D. Echinococcal cyst

537
Q

In regards to the biliary system, which of the following is associated with ulcerative colitis in approximately 50% of cases and demonstrates a “beaded” appearance?

  1. Sclerosing cholangitis
  2. Choledocholithiasis
  3. Caroli’s disease
  4. Choledochal cysts

Brant and Helms, Fundamentals of Diagnostic Radiology, 2nd ed., Lippincott, 1999, p.681.

A
538
Q

Which of the following statements is correct in regards to Klastkin’s tumor?

  1. They represent a form of hepatoma seen in the young and invade the left and right bile ducts at the site of their anastamosis.
  2. Represents adenocarcinoma of the gallbladder.
  3. It is an aggressive, poorly differentiated subtype of cholangiocarcinoma that occurs at the junction of the left and right bile ducts.
  4. Refers to the outpouchings seen with adenomyomatosis.

Brant and Helms, Fundamentals of Diagnostic Radiology, 2nd ed., Lippincott, 1999, p.682-83.

A
539
Q

Porcelain gallbladder is considered a risk factor in the development of adenocarcinoma of the gallbladder. What percentage best represents the incidence of developing carcinoma when associated with this risk factor?

  1. 10-20%
  2. 50%
  3. 60-70%
  4. 90%

Paul and Juhl’s, Essentials of Radiological Imaging, 7th ed., Lippincott, 1998, p. 533.

A
540
Q

Magnetic resonancecholangiopancreatography (MRCP) demonstrates a choledochal cyst characterized by saccular/fusiform dilation of the common bile duct. According to the Todani classification, what percentage of choledochal cysts demonstrate this appearance?

  1. 1-5%
  2. 2%
  3. 19%
  4. 80-90%

Brant and Helms, Fundamentals of Diagnostic Radiology, 2nd ed., Lippincott, 1999, p.683.

A
541
Q

The presence of psammomatous calcification in the abdomen should warrant further investigation to exclude what condition?

  1. Prostatitis
  2. Leiomyoma of the myometrium
  3. Cystadenocarcinoma of the ovary
  4. Medullary sponge kidney

Baker, pg.

A
542
Q

Ductal adenocarcinoma of the pancreas is a highly lethal tumor that is usually unresectable at presentation and carries a life expectancy of less than one year, and is second only to ______ as the most common digestive tract malignancy.

  1. Stomach
  2. Colorectal
  3. Hepatic
  4. Gallbladder

Brant and Helms, Fundamentals of Diagnostic Radiology, 2nd ed., Lippincott, 1999, p.693.

A
543
Q

Which of the following is best described as complete failure of fusion of the dorsal and ventral pancreatic ducts resulting in the majority of the pancreas being drained by the dorsal duct via the minor papilla?

  1. Pancreas divisum
  2. Wirsung’s pancreas
  3. Annular pancreas
  4. Santorinin’s pancreas

Paul and Juhl’s, Essentials of Radiological Imaging, 7th ed., Lippincott, 1998, p. 539.

A
544
Q

What percentage of the population has pancreas divisum?

  1. 10%
  2. 30%
  3. 50%
  4. 80%

Paul and Juhl’s, Essentials of Radiological Imaging, 7th ed., Lippincott, 1998, p. 539.

A
545
Q

Insulinoma is the most common islet cell tumor, accounting for ___ of cases.

  1. 20%
  2. 40%
  3. 60%
  4. 80%

Paul and Juhl’s, Essentials of Radiological Imaging, 7th ed., Lippincott, 1998, p. 543.

A
546
Q

Which of the following imaging techniques is best utilized to evaluate the patency of the pancreatic ducts?

  1. Pancreatic ultrasound
  2. Helical computed tomography
  3. Endoscopic retorgrade cholangiopancreatography
  4. Arteriography

Brant & Helms Fundamentals of Diagnostic Radiology p.691 Paul & Juhl’s p.537

A
547
Q

What is the best description for the location of the uncinate process of the pancreas when visualized on helical CT?

  1. Posterior to the inferior vena cava
  2. Anterior to the inferior vena cava but posterior to the superior mesenteric artery and vein.
  3. Anterior to the superior mesenteric artery and vein.
  4. Medial to the hilum of the spleen.

Brant & Helms Fundamentals of Diagnostic Radiology p.691-92

A
548
Q

Which of the following is the most common cause of chronic pancreatitis?

  1. Alcohol abuse
  2. Impacted gallstone
  3. Hyperlipidemia
  4. Parasitic infection

Brant & Helms Fundamentals of Diagnostic Radiology p.693

A
549
Q

What is the second most common malignancy of the gastrointestinal tract?

  1. Colorectal carcinoma
  2. Gastric carcinoma
  3. Pancreatic carcinoma
  4. Esophageal carcinoma

Brant & Helms Fundamentals of Diagnostic Radiology p.693

A
550
Q

Which of the following complications of acute pancreatitis is seen as an area of nonenhancing pancreatic tissue containing gas?

  1. Pseudocyst
  2. Pseudoaneurysm
  3. Infected necrosis
  4. Hemorrhage

Brant & Helms Fundamentals of Diagnostic Radiology p.693

A
551
Q

Which of the following functioning islet tumors is least likely to become malignant?

  1. Gastrinoma
  2. Somatostatinoma
  3. Glucogonoma
  4. Insulinoma

Brant & Helms Fundamentals of Diagnostic Radiology p.695-6

A
552
Q

Which of the following functional islet tumors presents with diabetes mellitus and painful glossitis and has an 80% malignant potential?

  1. Gastrinoma
  2. Somatostatinoma
  3. Glucagonoma
  4. Insulinoma

Brant & Helms Fundamentals of Diagnostic Radiology p.696

A
553
Q

Pancreatic pseudocysts are found in 30% of patients with this disease?

  1. Zollinger-Ellison Syndrome
  2. Autosomal recessive polycystic kidney disease
  3. Autosomal dominant polycystic kidney disease
  4. Hippel-Lindau syndrome

Paul & Juhl’s p. 543

A
554
Q

Ductal adenocarcinoma accounts for 95% of all malignant tumors of the pancreas. What is the most likely place for these malignancies to occur?

  1. Head
  2. Neck
  3. Body
  4. Tail

Paul & Juhl’s p.542

A
555
Q

Pancreatic adenocarcinoma has a very poor prognosis and 90% are unresectable at the time of diagnosis. However, resection is useful in some cases and requires the surgeon to remove the distal stomach, duodenum, and pancreas. What is the name of this procedure?

  1. Poirier’s resection
  2. Bilroth I procedure
  3. Bilroth II procedure
  4. Whipple’s procedure

Paul & Juhl’s p543

A
556
Q

Which of the following is false concerning pancreas anatomy

  1. neck, body and tail of the pancreas lie anterior to the splenic vein
  2. the head of the pancreas wraps around the junction of the superior mesenteric vein and splenic vein
  3. the uncinate process of the pancreatic head extends just anterior to the inferior vena cava
  4. the pancreas is intraperitoneal and located in the less sac

Brant & Helms, 3rd pg 782

A

pancreas is extraperitoneal and located in anterior pararenal space

557
Q

What is the most common cause of acute pancreatitis in an adult?

  1. trauma
  2. malignancy
  3. idiopathic
  4. gallstone

Brant & Helms 3rd pg 784 table 28.1

A
558
Q

What is the most common cause of chronic pancreatitis in an adult

  1. alcoholism
  2. trauma
  3. penetrating ulcer
  4. infection

Brant & Helms 3rd pg 784 table 28.1

A

Brant & Helms3rd pg 784 table 28.1

559
Q

Which of the following is not a morphological change associated with chronic pancreatitis

  1. dilation of the pancreatic duct
  2. increase in visible pancreatic parenchyma
  3. pancreatic calcifications
  4. fluid collections, both intra- and extra-pancreatic

Brant & Helms 3rd pg 785

A
560
Q

Which of the following is not true regarding pancreatic carcinoma

  1. average survival is 5-8 months
  2. seen as non-enhancing low-signal tumor on T1 images
  3. is the second most common GI tract malignancy
  4. calcifations are very rare

Brant & Helms 3rd pg 786-7

A

Pancreatic Ca enhances with contrast administration

561
Q

Match the Islet cell tumor w/ clinical symptoms

Insulinomas -

Gastrinomas -

Glucagonoma -

Somatostatinoma -

VIPoma -

a. Zollinger-Ellison syndrome
b. diabetes and steatorrhea
c. diabetes mellitus with glossitis
d. massive watery diarrhea
e. VIPoma
f. hypoglycemi

A

Brant & Helms 3rd pg 787

562
Q

Which is false concerning islet cell tumors of the pancreas

  1. malignant potential varies with cell type for functioning tumors
  2. functional tumors are usually large
  3. non-functional tumors are usually large
  4. non-functional tumors have a high rate of malignancy

Brant & Helms, 3rd pg 787

A

functional tumors are small (<4cm) insulinomas 10%; gastrinomas 60%; glucagonoma 80%

malignant nonfunctional tumors are 6-20cm 80% of nonfunctional tumors are malignant

563
Q

Which of the following is not true concerning pancreatic psuedocysts

  1. most common pacreatic cystic lesion
  2. fluid density with fibrous wall
  3. internal septations are uncommon
  4. fibrous wall can calcify

Brant & Helms 3rd pg 788

A
564
Q

Which is not true concerning true pancreatic cysts

  1. found in 30% of patients with von Hippel-Lindau syndrome
  2. found in 60% of patients with autosomal-dominant polycystic kidney disease
  3. associated with islet cell tumors in patients that also have von Hippel-Lindau syndrome
  4. sometimes seen with cystic fibrosis
A

Brant & Helms 3rd pg 789

found in 10% of patients with autosomal-dominant polycystic kidney disease

565
Q

Which is not true concerning microcystic adenoma aka serous cystadenoma

  1. composed of innumerable small cysts
  2. contains a central stellate fibrous scar
  3. most patients are under 60 yoa
A

Brant & Helms, 3rd pg 789

  • most are over 60 aka glycogen rich cystadenoma associated with von Hippel-Lindau syndrome shows marked enhancement
566
Q

True or False

Macrocystic serous cystadenoma is not distinguishable on imaging studies from mucinous cystic neoplasm

Brant & Helms, 3rd pg 789

A
567
Q

Which is not true concerning intraductal papillary mucinous neoplasm

  1. most common in the uncinate process of the pancreas
  2. shows marked dilation of the pancreatic duct
  3. main duct type may appear as focal group of small cysts
  4. capsule is formed by adjacent atrophied parenchyma

Brant & Helms 3rd pg 790

A

branch duct type may appear as focal group of small cysts

568
Q

Which of the following is true concerning the normal anatomy of the pharynx and esophagus

  1. hypopharynx extends from the soft palate to the epiglottis
  2. valleculae are lateral recesses in the hypopharynx
  3. piriform sinuses are lateral pouches at the base of the tongue near the epiglottis
  4. the esophagus lacks a serosal layer allowing for easy spread of tumor
A

Brant & Helms3rd - pg 799

  • hypopharynx extends from hyoid to crycopharyngeus muscles piriform sinuses are bilateral lateral recesses next to the larynx valleculae are lateral pouches between the base of the tongue and epiglottis
569
Q

Which of the following is not true of multiple regular transverse folds in the esophagus

  1. is called feline esophagus
  2. the tubulovestibular junction is marked by the A ring
  3. the sqaumocolumnar junction is marked by the B ring
  4. the Z line the esophageal vestibule is located at the superior most aspect of the esophagus
A

Brant & Helms 3rd - pg 800

  • esophageal vestibule is the saccular termination of the esophagus
570
Q

Which of the following signs of pharyngeal dysfunction is not correctly defined

  1. pharyngeal stasis - impaired pharyngeal transports that results in residual filling of the valleculae and piriform sinuses
  2. laryngeal penetration - entry of swallowed material into the laryngeal vestibule without passage below the vocal cords
  3. aspiration - swallowed material passing below the vocal cords
  4. nasal regurgitation - swallowed material passing into the nasal pharynx due to a poor seal between soft palate and posterior pharyngeal wall none of the above

Brant & Helms 3rd - pg 801

A
571
Q

A cricopharyngeal bar is NOT indicative of what pathology

  1. cricopharyngeal achalasia
  2. esophageal achalasia
  3. normal variant
  4. neuromuscular disorders of the pharynx

Brant & Helms, 3rd - pg 801-2

A
572
Q

Select the true statement

  1. MCC of neurologic dysfunction of swallowing is cerebrovascular disease and stroke
  2. Myasthenia gravis and muscular dystrophy affect the lower third of the esophagus
  3. The esophagus is uncommonly affected by scleroderma
  4. Diffuse esophageal spasm is characterized by multiple primary esophageal contractions

Brant & Helms 3rd - pg 803

A

Striated muscle disease (myastenia gravis, muscular dystrophy, etc) affect the proximal third; esophagus involved in 80% of cases of scleroderma; diffuse esophageal spasm is characterized by multiple tertiary waves in middle-aged patients

573
Q

Which is not associated with esophageal achalasia

  1. deficiency of ganglion cells in the myenteric plexus
  2. failure of the lower gastroesophageal sphincter to relax
  3. tapered beak appearance
  4. infection by Trypanosoma cruzi

Brant & Helms, 3rd pg 803

A

T. cruzi is found in Chagas disease which appears similar to achalasia

574
Q

Which is false concerning gastroesophageal reflux disease (GERD)

  1. results from incompetence of the lower esophageal sphincter
  2. can result in development of Barrett’s esophagus
  3. 20% of normal individuals show spontaneous reflux on upper GI exams
  4. there is a high correlation between hiatal hernia and GERD

Brant & Helms, 3rd - pg 804

A

poor correlation between hiatal hernia and GERD has been found

575
Q

Which type of hiatal hernia is the most common

  1. sliding type
  2. paraesohageal type
  3. mixed type
  4. compound type

Brant & Helms, 3rd - pg 804-5

A

sliding 95%, mixed and compound are the same thing and are a subtype of paraesophageal (the most common subtype)

576
Q

Which statement is not true concerning diverticula of the pharynx and esophagus

  1. Lateral pharyngeal diverticula are commonly seen wind instrument players
  2. Zenker’s diverticulum occurs at Killian’s dehiscence
  3. Epiphrenic diverticula are more common on the right
  4. Intramural tracking is associated with midesophageal diverticula

Brant & Helms 3rd pg 805-6

A

d. Intramural tracking is associated with midesophageal diverticula

intramural tracking is seen with intramural pseudodiverticulum

577
Q

Concerning esophagitis, which is correct

  1. Ulcers are a hallmark finding
  2. Reflux esophagitis demonstrates nodules with very distinct borders
  3. Barrett’s esophagus is associated with a 15% prevalence of squamous cell carcinoma
  4. UGI exams are sensitive for even mild cases of esophagitis

Brant & Helms, 3rd pg 806-7

A

a. Ulcers are a hallmark finding
* reflux esophagitis nodules have indistinct borders; distinct borders seen in candida barrett’s esophagus associated with adenocarcinoma upper gi exams show fewer than half of mild cases

578
Q

Which is false concerning infectious esophagitis

  1. candida albicans is the most common cause
  2. discrete ulcers on a background of normal mucosa is characteristic of herpes infection
  3. esophagus is the least common portion of the GI tract to be involved by tuberculosis infectious
  4. esophagitis is commonly found in immunocompetent patients

Brant & Helms 3rd pg 807-8

A

d. esophagitis is commonly found in

immunocompetent patients

usually found in immune-compromised patients

579
Q

Concerning strictures of the esophagus, which is false

  1. strictures are most commonly caused by fibrosis from inflammation and neoplasm
  2. all should be evaluated endoscopically
  3. reflux esophagitis is the most common cause
  4. malignant strictures are usually smoothly tapered

Brant & Helms 3rd pg 809

A

malignant strictures are usually smoothly tapered

malignant strictures are usually abrupt and irregular, but can be smooth

580
Q

True or False

Schatzki ring is a pathologic ring-like stricture at the level of the A ring caused by reflux esophagitis

Brant & Helms, 3rd pg 810

A

FALSE

Schatzki ring found at the B ring

581
Q

Which is true concerning webs of the pharynx and esophagus

  1. pharyngeal webs are most common at the posterior wall of the hypopharynx
  2. esophageal webs are most common in the distal esophagus
  3. most are incidental findings
  4. webs are usually thick membranes measuring greater than 5 mm

Brant & Helms, 3rd pg 810

A

most are incidental findings

pharyngeal webs seen at anterior wall of hypopharynx

esophageal webs are seen most commonly just distal to cricopharyngeus webs usually are thin measuring 1-2mm and can occasionally cause dysphagia

582
Q

Which is false concerning thickened esophageal folds

  1. uphill varices are due to portal hypertension
  2. downhill varices are due to superior vena caval obstruction
  3. varices are usually rigid and persistent, even during peristalsis
  4. thickened folds in esophagitis are most commonly associated with reflux esophagitis

Brant & Helms, 3rd pg 810-11

A

varices are usually rigid and persistent, even during peristalsis

  • rigid and persistent folds are associated with varicoid carcinoma
583
Q

Which is false concerning esophageal neoplasm

  1. most esophageal carcinomas are of the squamous cell type
  2. leiomyoma is the most common benign neoplasm of the esophagus
  3. adenocarcinoma of the esophagus is associated with Barrett’s esophagus
  4. esophageal carcinoma frequently has a good prognosis

Brant & Helms3rd pg 811-12

A

esophageal carcinoma frequently has a good prognosis

  • 5 year survival is 5% for Esophageal Ca
584
Q

Which is true concerning esophageal filling defects

  1. duplication cysts are most common in the upper esophagus
  2. polyps are most common in the lower esophagus
  3. aberrant right subclavian artery can produce a filling defect on the posterior aspect of the esophagus
  4. extrinsic masses rarely affect the esophagus

Brant & Helms, 3rd pg 813

A

aberrant right subclavian artery can produce a filling defect on the posterior aspect of the esophagus

  • duplication cysts seen in the lower esophagus; ddx includes bronchogenic and neurenteric cycsts
  • polyps most common in upper esophagus
  • adenopathy, bronchogenic carcinoma and vascular lesions can produce mass effect on the esophagus
585
Q

True or false

The most common cause of esophageal perforation is instrumentation

A

TRUE

586
Q

True or false

Boorhaave syndrome occurs most commonly at the left posterior wall at the level of the diaphragm

A

TRUE

587
Q

Which of the following is not a radiographic sign of pneumoperitoneum?

  1. Gas outlining the both sides of the bowel (Rigler’s sign).
  2. Gas outlining the peritoneal cavity (the “football” sign).
  3. Gas outlining the falciform ligament.
  4. Gas outlining the fat within Grota’s fascia.

Brant & Helms Fundamentals of Diagnostic Radiology p.656

A

Gas outlining the fat within Grota’s fascia

588
Q

Which of the following is not a cause of pneumoperitoneum?

  1. Introduction per vaginam
  2. Introduction per rectum
  3. Pneumothorax
  4. Pneumomediastinum

Stephen Chapman Radiological Differential Diagnosis p.196

A

Introduction per rectum

589
Q

Which of the following is the most common cause of pneumoperitoneum?

  1. Idiopathic
  2. Perforation
  3. Iatrogenic
  4. Pneumomediastinum

Brant & Helms Fundamental of Diagnostic Radiology p.656

A

Perforation

590
Q

A large retroperitoneal mass, medial coursing of the ureters, and narrowing of the middle third of the ureters is probably due to?

  1. Lymphoma
  2. Abdominal aortic aneurysm
  3. Horseshoe kidney
  4. Retroperitoneal fibrosis

Taveras and Ferruci

A

Retroperitoneal fibrosis

591
Q

What is the most common cause of retroperitoneal gas?

  1. Perforation of the duodenum
  2. Perforation of perirenal abscess
  3. Postoperative
  4. Gallbladder perforation

Eisenberg 3rd ed. p.950

A

Perforation of the duodenum

592
Q

What is the most common source of small bowel obstruction in the Western World?

  1. Malignancy
  2. Intussusception
  3. Gallstone ileus
  4. Postsurgical adhesions

Brant & Helms Fundamental of Diagnostic Radiology p.658

A

Postsurgical adhesions

593
Q

What is the most common cause of small bowel obstruction in developing nations?

  1. Malignancy
  2. Incarcerated hernia
  3. Parasites
  4. Foreign body

Brant & Helms Fundamental of Diagnostic Radiology p.658

A

Incarcerated hernia

594
Q

Enteroclysis was performed on a 37 year old male patient with a distended abdomen. Upon review of the barium study, you notice a “stack of coins” appearance within the right lower quadrant. What is the most likely cause of this radiographic finding?

  1. Colo-colic intussusception as a sequelae of adenocarcinoma.
  2. Ileocolic intussesception caused by a polypoid tumor.
  3. Enteroenteric intussusception resulting from non-tropical sprue.
  4. Gastroenteric intussusception as a result of fibrosis in the duodenal bulb.

Brant & Helms Fundamental of Diagnostic Radiology p.659

A

Ileocolic intussesception caused by a polypoid tumor.

595
Q

Gas or fluid-filled loops of small bowel should normally be no more than what size on a KUB film:

  1. 1.5 cm
  2. 2.0 cm
  3. 3.0 cm
  4. 4.0 cm

Taveras Ch. 2, p3

A

3.0 cm

596
Q

A supine KUB of a patient with gastric ulcer perforation demonstrates visualization of both the intraluminal and peritoneal surfaces of the bowel wall. Which of the following signs describes this appearance?

  1. Football sign
  2. Kirklin sign
  3. String sign
  4. Rigler’s sign

Brant and Helms, Fundamentals of Diagnostic Radiology, 2nd ed., Lippincott, 1999, p.656.

A

Rigler’s sign

597
Q

_________ is a condition characterized by aphthous ulcers, mucosal fold thickening and distortion, segments of normal bowel interspersed between pathological ones and abdominal abscesses that can be very large.

  1. Ulcerative Colitis
  2. Behçet’s
  3. Whipple’s Disease
  4. Crohn’s Disease

Paul and Juhl’s, Essentials of Radiological Imaging, 7th ed., Lippincott, 1998, p. 605.

A

Crohn’s Disease

598
Q

Which of the following is the most common motility disorder with regards to the mesenteric small bowel?

  1. Scleroderma
  2. Paralytic ileus
  3. Jejunal atresia
  4. Imperforate anus syndrome

Paul and Juhl’s, Essentials of Radiological Imaging, 7th ed., p. 605.

A

Paralytic ileus

599
Q

The “Stierlin sign”, although non-specific, may help in the diagnosis of ….

  1. Sprue
  2. Tuberculosis
  3. Lymphoma
  4. Crohn’s disease
A

Tuberculosis

600
Q

The term used to describe a distended loop of bowel adjacent to an acute inflammatory process is?

  1. Dynamic ileus
  2. Sentinel loop
  3. Peritonitis
  4. Intestinal pseudo-obstruction
A

Sentinel loop

601
Q

The combination of gas in the biliary duct system and simple obstruction at the iliocecal valve by a calcific concretion is most suggestive of…

  1. Mecomium ileus
  2. Gallstrone ileus
  3. Small left colon syndrome
  4. Carcinoid tumor
A

Gallstrone ileus

602
Q

A periodic acid-schiff (PAS) test that proves positive corresponds with which of the following gastro-intestinal disorders…

  1. Sprue
  2. Amyloidosis
  3. Scleroderma
  4. Whipple’s disease
A

Whipple’s disease

603
Q

Which statement is false concerning small bowel neoplasms

  1. Approximately 30% of GISTs are found in the small intestine and they tend to be more aggressive than similarly sized lesion in the stomach
  2. adenomas account for 20% of benign small bowel neosplasms
  3. lipomas account for 17% of benign small bowel neoplasms and are most commonly found in the ileum
  4. hemangioma accounts for 50% of benign small bowel neoplasms

Brant & Helms 3rd pg 837

A

hemangioma accounts for 50% of benign small bowel neoplasms

  • hemangioma account for fewer than 10% of benign small bowel tumors
604
Q

What is the most common solid mesenteric mass?

  1. lymphoma
  2. metastases
  3. GIST
  4. mesenteric desmoid

Brant & Helms 3rd pg 838

A

lymphoma

605
Q

Which of the following statements is incorrect in regards to mesenteric masses

  1. mesenteric desmoid tumors are usually multiple and are associated with Gardner’s syndrome
  2. mesenteric cysts are lymphangiomas that arise in the root of the small bowel
  3. mesentery misty mesentery is a CT finding associated with GI duplication cysts
  4. mesenteric teratomas frequently contain fat or calcium

Brant & Helms 3rd pg 839

A

mesentery misty mesentery is a CT finding associated with GI duplication cysts

  • misty mesentery suggestive of sclerosing mesenteritis
606
Q

Which of the following is not true

  1. the hallmark of mechanical small bowel obstruction is a transition point between dilated and non-dilated bowel
  2. straight and regular thickened folds in the small bowel indicate infiltration by fluid
  3. scleroderma produces a hide-bound appearance most commonly in the duodenum and jejunum
  4. adult celiac disease results in an increased number of folds per inch in the jejunum

Brant & Helms 3rd pg 840-1

A

adult celiac disease results in an increased number of folds per inch in the jejunum

  • folds decrease in the jejunum and increase in the ileum
607
Q

Long segment, tapered strictures are suggestive of what diagnosis

  1. radiation enteritis
  2. lymphangiectasia
  3. eosinohilic gastroenteritis
  4. amyloidosis

Brant & Helms 3rd pg 842

A

radiation enteritis

608
Q

Which is true concerning amyloidosis

  1. 50% of secondary cases are associated with tuberculosis
  2. the large bowel is the most common site of GI involvement
  3. most cases are localized
  4. associated with intracellular infiltration of amorphous protein material

Brant & Helms 3rd pg 843

A

50% of secondary cases are associated with tuberculosis

  • small bowel is most common site of GI involvement
  • most cases are systemic only 10-20% are localized
  • associated w/ extracellular infiltration of amorphous protein material
609
Q

Tiny (1 mm) sandlike nodules on the mucosa of the small intestine suggests which disease?

  1. amyloidosis
  2. whipple disease
  3. eosinophilic gastroenteritis
  4. systemic mastocytosis

Brant & Helms 3rd pg 843-4

A

whipple disease

610
Q

Which of the following is not seen with Crohn disease

  1. cobblestone pattern
  2. string sign
  3. skip lesions
  4. collar button ulcers

Brant & Helms 3rd pg 844,855

A

collar button ulcers

  • collar button ulcers w/ ulcerative colitis
611
Q

What is the most common pattern of involvement in Crohn disease

  1. colon alone
  2. small bowel alone
  3. colon and terminal ileum
  4. small bowel but sparing the terminal ileum

Brant & Helms 3rd pg 844

A

colon and terminal ileum

612
Q

Which age range most commonly presents with Crohn disease

  1. < 10
  2. 10-40
  3. 40-70
  4. 70+

Brant & Helms 3rd pg 844

A

10-40

613
Q

Which of the following is not associated with Crohn disease

  1. fistulae formation
  2. increased risk of carcinoma
  3. megaloblastic anemia
  4. decreased incidence of gallstones and renal stones

Brant & Helms, 3rd pg 844-5

A

decreased incidence of gallstones and renal stones

  • increased incidence of gallstones and renal stones
  • fistulae formation in 19% also associated with arthritis/spondylitis that mimics ankylosing spondylitis
614
Q

Which is NOT true concerning Yersinia entercolitis

  1. children and adults are most often affected
  2. self-limited infection
  3. radiographic findings are most pronounced in the first 20 cm of the ileum
  4. nodular lymphoid hyperplasia may occur during healing

Brant & Helms, 3rd pg 845

A

radiographic findings are most pronounced in the first 20 cm of the ileum

  • findings most pronounced in the last 20 cm of the ileum
615
Q

Which of the following is NOT true concerning tuberculosis involvement in the intestines

  1. most commonly affects the duodenum
  2. fewer than half of patients have concurrent evidence of pulmonary TB
  3. produces transverse or stellate ulcers
  4. ileocecal valve is stiff and gaping with a narrowed ileum and cecum

Brant & Helms3rd pg 846

A

most commonly affects the duodenum

  • most commonly affects the ileocecal area
616
Q

Which is not true concerning small bowel diverticulum

  1. most common in the jejunum
  2. found along the antimesenteric border
  3. commonly multiple
  4. associated with megaloblastic anemia

Brant & Helms3rd pg 846

A

found along the antimesenteric border

  • found on the mesenteric border protruding between the leaves of the mesentery
  • antimesenteric outpouchings are pseudodiverticula caused by small bowel disease such as Crohn or Scleroderma
617
Q

Which radionuclide material shows increased uptake on 62% of scans of meckel’s diverticulum

  1. Tc-99m pertechnetate
  2. Tc-99m sulfur colloid
  3. Tc-99m DTPA
  4. Gallium-67 citrate

Brant & Helms 3rd pg 846

A

a. Tc-99m pertechnetate

618
Q

The classic symptoms of small bowel obstruction include: crampy abdominal pain, bloating, nausea, vomiting and decrease stool output. Diffuse abdominal tenderness and peritoneal signs are common. What is the rate of mortality if there is a complete obstruction without timely medical intervention.

  1. 5%
  2. 30%
  3. 60%
  4. 90%

Eisenberg 3rd ed. p.421

A

c. 60%

619
Q

Radiographic signs of complete bowel obstruction include distended small bowel loops containing gas and fluid. How quickly after the bowel obstruction is there radiographic evidence?

  1. 15-30 minutes
  2. 3-5 hours
  3. 1-2 days
  4. There is no radiographic evidence

Eisenberg 3rd ed. p.421

A

b. 3-5 hours

620
Q

Which of the following is not a cause of adynamic ileus?

  1. Surgical manipulation
  2. Electrolyte imbalance
  3. Renal stones
  4. Crohn’s disease

Eisenberg 3rd ed. p.442

A

d. Crohn’s disease

621
Q

Giardiasis lamblia is a protozoan parasite harbored by millions of asymptomatic individual throughout the world. It can be seen in travelers to endemic areas (Leningrad, India and yes, the Rocky Mountains of Colorado). Clinically significant infestations occur predominantly in children. What other disease entity does it appear identical to?

  1. Celiac sprue
  2. Whipple disease
  3. Chagas disease
  4. Amyloidosis

Eisenberg 3rd ed. p.479

A

a. Celiac sprue

622
Q

Which of the following is the second most common benign neoplasm of the small bowel?

  1. Lipoma
  2. Hemangioma
  3. Adenoma
  4. Leiomyoma

Eisenberg 3rd ed. p.495

A

c. Adenoma

623
Q

What is the most frequent congenital anomaly of the intestinal tract?

  1. Meckel’s diverticula
  2. Duplication Cyst of the stomach
  3. Double barrel esophagus
  4. Zenker’s diverticulum

Eisenberg 3rd ed. p.548

A

a. Meckel’s diverticula

624
Q

What clinical symptom is the hallmark of Crohns’s disease?

  1. Gross bleeding
  2. Gall bladder stones
  3. Diarrhea
  4. Upper abdominal pain (colicky or crampy)

Eisenberg 3rd ed. p.610

A

c. Diarrhea

625
Q

Gastrointestinal juvenile polyposis with associated hyperpigmentation, alopecia, and atrophy and subsequent loss of fingernails and toenails, is characteristic of:

  1. Ruvalcaba-Myhre-Smith syndrome
  2. Juvenile Polyposis syndrome
  3. Turcot’s syndrome
  4. Cronkhite-Canada syndrome

Eisenberg, third edition p. 724

A

d. Cronkhite-Canada syndrome

626
Q

What is the most frequent congenital anomaly of the intestinal tract?

  1. a diverticula arising within 100 cm of the ileocecal valve
  2. pseudodiverticuli associated with scleroderma
  3. thin-walled outpouchings lacking muscular components, located in the jejunum
  4. sac of duodenal mucosa originating in the 2nd portion of the duodenum near papilla of vater

Eisenberg, third edition p. 546

A

a diverticula arising within 100 cm of the ileocecal valve

  • Meckels
  • Rule of 2’s
627
Q

Periodic acid-Schiff-positive macrophages may be found in:

  1. Crohn’s disease
  2. Pancreatitis
  3. Whipple’s disease
  4. Mastocytosis

Eisenberg, 3rd edition p. 343

A

c. Whipple’s disease

628
Q

A characteristic sign of jejunal atresia is?

  1. double-bubble sign
  2. tripple bubble sign
  3. single bubble sign
  4. inverted “3” sign

Eisenberg, third edition p. 437

A

b. triple-bubble sign

629
Q

The “string sign”, which resembles a frayed cotton string, represents incomplete filling of the terminal ileum due to the irritability and spasm accompanying severe ulceration, and is considered to be a pathognomonic radiographic manifestation of?

  1. Ulcerative colitis
  2. Amebiasis
  3. Yersinia enterocolitis
  4. Crohn’s disease

Eisenberg, third edition p. 589

A

d. Crohn’s disease

630
Q

Pneumatosis intestinalis in infants is usually associated with underlying:

  1. ischemic necrosis due to mesenteric vascular disease
  2. necrotizing enterocolitis
  3. adynamic ileus
  4. gastrointestinal endoscopy

Eisenberg, third edition p. 927

A

b. necrotizing enterocolitis

631
Q

Mast cell proliferation in the reticuloendothelial system and skin, in combination with sclerotic bone lesions, hepatosplenomegaly, flushing, pruritus, tachycardia, asthma and nausea, may be seen with:

  1. Behcet’s disease
  2. Uticarial pigmentosa
  3. Lymphangiectasia
  4. Yersinia infection

Eisenberg, third edition p. 486

A

b. Uticarial pigmentosa

632
Q

Penetrating ulcers or fissures, confluent linear ulcers, a discontinuous segmental pattern and a thickened bowel wall are characteristic of:

  1. Ulcerative colitis
  2. Crohn’s colitis
  3. Caustic colitis
  4. Campylobacter fetus colitis

Eisenberg, third edition p. 608

A

Crohn’s colitis

633
Q

A patient presents with isolated, tiny, discrete erosions (aphthoid ulcers) visualized in the terminal ileum, abdominal pain, and a history of renal calculi. The diagnosis is most likely:

  1. Amebic colitis
  2. Behcet’s syndrome
  3. Crohn’s disease
  4. Yersinia colitis
A

c. Crohn’s disease

634
Q

Thickened folds in the non-dilated small bowel that are smooth and regular are characteristic of:

  1. Zollinger-Ellison syndrome
  2. hypoproteinaemia
  3. Lymphoma
  4. Crohn’s disease

Chapman, 3rd p. 230

A

b. hypoproteinaemia

635
Q

In regards to Meckel’s diverticulum, which of the following is not part of the “rule of 2s”?

  1. Average length of 2 inches.
  2. Located within 2 inches of the ileocecal valve.
  3. Symptoms usually present by age 2.
  4. Present in approximatley 2% of the population.

Federle, Jeffrey, Desser, Anne and Eraso, Diagnostic Imaging, Abdomen, Amirsys, 2005, p. I-4-8.

A

b. Located within 2 inches of the ileocecal valve.

636
Q

Fold thickness of the small bowel should normally measure no more than _____ .

  1. 1 mm
  2. 3 mm
  3. 6 mm
  4. 9 mm
A

3 mm

637
Q

Complete loss of the mucosal folds in the small intestine, seen in advanced sprue, is called what?

  1. String sign
  2. Mexican hat sign
  3. Steppladder appearance
  4. Moulage sign
A

Moulage sign

638
Q

Stricture of the iliocecal region maybe identical and impossible to differentiate radiographically in which two processes?

  1. Ulcerative colitis and Crohn’s disease
  2. Actinomycosis and Ulcerative colitis
  3. Crohn’s disease and Tuberculosis
  4. Tuberculosis and Actinomycosis
A

Crohn’s disease and Tuberculosis

639
Q

Bowel obstruction during the immediate post-natal period, termed meconium ileus, is related to which of the following disorders?

  1. Systemic lupus erythematosis
  2. Inspissated milk syndrome
  3. Cystic fibrosis
  4. Hirsprong’s disease
A

Cystic fibrosis

640
Q

Which is false concerning normal small bowel anatomy?

  1. jejunum is defined as the proximal two fifths of the small bowel
  2. the root of the small bowel is located at the ligament of Treitz
  3. the lumen diameter is less than 3 cm, the fold thickness is less than 3 mm and the wall thickness is less than 3 mm
  4. the ilium has more prominant valvulae conniventes

Brant & Helms3rd pg 833

A

the ilium has more prominant valvulae conniventes

  • valvulae conniventes more prominent in the jejunum
641
Q

Which of the following is the most common neoplasm of the small intestine

  1. Adenocarcinoma
  2. Carcinoid
  3. Lymphoma
  4. Lipoma

Brant & Helms 3rd pg 834

A

b. Carcinoid
* accounts for 1/3 of all small bowel tumors

642
Q

Which of the following is true concerning carcinoids of the small intestine

  1. account for 20% of all carcinoids
  2. most commonly occur in the jejunum
  3. most cases present with carcinoid syndrome
  4. 70% are multiple

Brant & Helms 3rd pg 834

A

account for 20% of all carcinoids

  • most occur in the ilium
  • only those with mets to the liver (7%) have carcinoid syndrome
  • only 30% are multiple
643
Q

A sunburst pattern in the mesenteric fat, bowel wall thickening, small, lobulated mass in the ilium and occasional central calcification are pathognomonic for what disease of the small bowel?

  1. Carcinoid
  2. Adenocarcinoma
  3. Lymphoma
  4. GIST

Brant & Helms 3rd pg 834

A

Carcinoid

644
Q

Which of the following is false concerning adenocarcinoma of the small bowel?

  1. most frequent in the distal ilium
  2. characteristic appearance is that of an “apple core” lesion
  3. adult celiac disease, crohn disease and peutz-jeghers syndrome are risk factors
  4. prognosis is poor, with a 5 year survival rate of 20%

Brant & Helms, 3rd pg 834-5

A

most frequent in the distal ilium

  • rare in the distal ilium
  • most common in the proximal jejunum and duodenum
645
Q

Which is true concerning Lymphoma of the small bowel

  1. most cases are Hodgkin lymphoma
  2. responsible for 20% of all small bowel malignancies
  3. most common in the proximal jejunum
  4. multiple sites of involvement found in up to 75% of cases

Brant & Helms 3rd pg 835

A

responsible for 20% of all small bowel malignancies

  • most cases are non-hodgkin b-cell type most common in the distal ilium multiples sites found in up to 25% of cases
646
Q

Which is not true concerning lymphoma in the small bowel

  1. associated with the sandwich sign
  2. exophytic lymphomas usually show prominant enhancement
  3. burkitt lymphoma usually affects the ileocecal area in children
  4. may produce aneurysmal dilation of the small bowel

Brant & Helms, 3rd pg 835-6

A

exophytic lymphomas usually show prominant enhancement

  • exophytic lymphomas usually show little or no enhancement
  • enhancement seen in adenocarcinoma and GIST
647
Q

Which statement is false concerning Metastases to the small bowel

  1. peritoneal seeding usually involves the mesenteric border
  2. hematogenous spread usually involves the anti-mesenteric border
  3. most common location of implantation is near the ligament of trietz
  4. common primaries include melanoma, lung, breast and colon

Brant & Helms3rd pg 836-7

A

most common location of implantation is near the ligament of trietz

  • Most common site of implantation is terminal ilium, favored by gravity
648
Q

Which of the following is not true concerning Meckel’s diverticulum?

  1. most common congenital anomaly of the GI tract
  2. diverticulum may be attached to umbilicus by a remnant of the vitelline duct
  3. seen as blind sac on the mesenteric side of the ileum
  4. diverticulum may contain ectopic gastric mucosa

Brant & Helms3rd pg 846 attached to antimesenteric side of the ileum

A

seen as blind sac on the mesenteric side of the ileum

649
Q

Ivemark’s syndrome is a constellation of congential abnormalities characterized by bilateral right-sidedness, midline liver, bilateral three-lobed lungs, and cardiac anomalies. Which of the following, if any, anomalies of the spleen are associated with this syndrome?

  1. Splenomegaly
  2. Polysplenia
  3. Wandering spleen
  4. Asplenia

Brant and Helms, Fundamentals of Diagnostic Radiology, 2nd ed., Lippincott, 1999, p. 700.

A

Asplenia

650
Q

Which of the following is the most common primary neoplasm of the spleen?

  1. Adenoma
  2. Lymphangioma
  3. Hemangioma
  4. Capsular fibroma

Brant and Helms, Fundamentals of Diagnostic Radiology, 2nd ed., Lippincott, 1999, p.703.

A

Hemangioma

  • MC benign splenic mass
651
Q

What is the average length of the adult spleen?

3 cm

7 cm

12 cm

15 cm

Brant & Helms Fundamentals of Diagnostic Radiology p.699

A

15 cm

652
Q

Which of following pathologies is the most common cause of splenomegaly?

  1. Portal hypertension
  2. Lymphoma
  3. Malaria
  4. Gaucher’s disease

Brant & Helms Fundamentals of Diagnostic Radiology p701

A

Portal hypertension

653
Q

Which of the following congenital splenic anomalies are found with Ivemark’s syndrome?

  1. Splenosis
  2. Splenic regeneration
  3. Polysplenia
  4. Asplenia

Brant & Helms Fundamentals of Diagnostic Radiology p700

A

Asplenia

654
Q

A CT examination of the spleen demonstrates multiple low density lesions measuring approximately 5-10mm in an immunocompromised patient. What is the least likely diagnosis?

  1. Kaposi’s sarcoma
  2. Hydatid disease
  3. Cytomegalovirus
  4. Leukemia

Brant & Helms Fundamentals of Diagnostic Radiology p.701-2

A

Hydatid disease

655
Q

Which of the following would not be considered in a differential diagnosis for multiple splenic calcifications?

  1. Tuberculosis
  2. Hemangioma
  3. Phleboliths
  4. Gamma-Gandy bodies

Brant & Helms Fundamentals of Diagnostic Radiology p703

A

Gamma-Gandy bodies

656
Q

Which of the following is the most common primary malignancy of the spleen?

  1. Hodgkins lymphoma
  2. Non-hodgkins lymphoma
  3. Leukemia
  4. Angiosarcoma

Brant & Helms Fundamentals of Diagnostic Radiology p.701

A

Hodgkins lymphoma

657
Q

Calcification within the spleen are most commonly due to…

  1. Echinococcal cysts
  2. Epidermoid tumor
  3. Previous thorotrast injection
  4. Histoplasmosis
A

Histoplasmosis

658
Q

Concerning accessory spleens, which is not true

  1. found in up to 16% of normal individuals
  2. appear as small, round masses with the same imaging appearance as normal spleen parenchyma
  3. usually located near the aorta
  4. can be a site of splenic regeneration following splenectomy

Brant & Helms3rd - pg 791 usually found in the splenic hilum

A

usually located near the aorta

659
Q

Identify the intracellular formation that appears following splenectomy and then disappears with splenic regeneration

  1. Howell-Jolly bodies
  2. Gamna-Gandy bodies
  3. siderotic nodules
  4. Reed-Sternberg bodies

Brant & Helms3rd - pg 792

A

Howell-Jolly bodies

660
Q

Which of the following is not part of Ivemark’s syndrome

  1. congenital absence of a spleen
  2. bilateral three-lobed lungs
  3. normal liver
  4. bilateral right-sidedness

Brant & Helms3rd - pg 792 liver is midline

A

normal liver

  • liver is usualyl midline in this syndrome
661
Q

Which of the following is not an indication of splenomegaly

  1. any spleen dimension greater than 14 cm
  2. projection of the spleen anterior to the anterior axillary line
  3. inferior spleen tip extending below the inferior liver tip inferior
  4. spleen tip extending below the upper pole of the left kidney

Brant & Helms3rd - pg 792

A

spleen tip extending below the upper pole of the left kidney

  • splenomegaly when seen extending below the lower pole of the left kidney
662
Q

Which of the following is true concerning splenic cystic lesions

  1. post-traumatic cysts are false cysts that calcify commonly and account for 30% of all splenic cysts
  2. true splenic cysts nearly always have calcifications in the wall
  3. pancreatic pseudocysts complicate up to 5% of patients with pancreatitis
  4. hydatid cysts are frequently seen in the spleen

Brant & Helms3rd - pg 793-4

A

pancreatic pseudocysts complicate up to 5% of patients with pancreatitis

  • post-traumatic cysts account for 80% of splenic cysts,
  • true splenic cysts have wall calcification 5% of the time
  • hydatid cysts found in the spleen 2% of the time (liver is most common location for hydatid cysts)
663
Q

Which of the fallowing is not true concerning solid lesions of the spleen

  1. lymphoma is the most common malignant tumor of the spleen
  2. calcification of metastases to the spleen is common
  3. hemangioma is the most common primary neoplasm of the spleen
  4. gamna-gandy bodies are associated with portal hypertension

Brant & Helms, 3rd - pg 794-5

A

calcification of metastases to the spleen is common

  • calcification is rare in mets to the spleen
664
Q

Which of the following is not true concerning splenic malignancies lymphoma

  1. is associated with splenic infarction metastases from melanoma
  2. are commonly cystic in appearance
  3. angiosarcoma is the most common malignancy arising in the spleen
  4. gamna-gandy bodies are associated with angiosarcoma

Brant & Helms, 3rd p795-6

A

gamna-gandy bodies are associated with angiosarcoma

  • gamna-gandy bodies assoc w/ portal hypertension aka siderotic nodules
665
Q

The cells responsible for producing pepsinogen and hydrochloric acid are located in which portion(s) of the stomach?

  1. Fundus and antrum
  2. Body
  3. Antrum
  4. Body and fundus

Brant and Helms, 2nd ed., Lippincott, 1999, p.723.

A

Body and fundus

666
Q

A __________ is a metastasis to the ovary from a mucin-producing tumor of the gastrointestinal tract.

  1. Klatskin tumor
  2. Melanoma
  3. Krukenberg tumor
  4. Leiomyosarcoma

Brant and Helms, Fundamentals of Diagnostic Radiology, 2nd ed., Lippincott, 1999, p.726, 867.

A

Krukenberg tumor

667
Q

Which of the following metastatic tumors to the stomach cause linitis plastica?

  1. Breast carcinoma
  2. Melanoma
  3. Hepatocellular carcinoma
  4. Fibrolamellar carcinoma

Brant and Helms, 2nd ed., p.727.

Paul & Juhl’s,7th ed.,1998, p. 588.

A

Breast carcinoma

668
Q

Aberrant pancreatic tissue known as “rests” are most commonly found in which portion of the stomach?

  1. Fundus
  2. Body
  3. Antrum
  4. Pylorus

Paul and Juhl’s, 7th ed.,1998, p. 573.

A

Antrum

669
Q

Which of the types of hernias is described as that in which the gastroesophageal junction (B ring) is displaced cephalad?

  1. Spigelian
  2. Sliding
  3. Paraesophageal
  4. Zenker’s
A

Sliding

670
Q

A “marginal ulcer” is frequently found as a complication to …

  1. Chronic gastritis
  2. Superficial carcinoma of the stomach
  3. Zollinger-Ellison syndrome
  4. Post gastrectomy
A

Post gastrectomy

671
Q

Which is not true of villous tumors of the stomach

  1. they adenomatous polypoid masses
  2. carry no malignant potential
  3. produce a characteristic soap-bubble appearance on contrast exams
  4. are usually solitary

Brant & Helms, 3rd p 822

A

carry no malignant potential

  • malig. potential varies w/ size:
    • 50% for lesions 2-4cm;
    • 80% 4 lesions >4cm
672
Q

Which is not true concerning gastric polyps

  1. associated signs include the bowler hat sign and the mexican hat sign
  2. hyperplastic polyps are neoplasms with no malignant potential
  3. adenomatous polyps account for 15% and carry a malignant potential
  4. hamartomatous polyps are found with Peutz-Jeghers syndrome and have no malignant potential

Brant & Helms3rd pg 822

A

hyperplastic polyps are neoplasms with no malignant potential

  • hyperplastic polyps are not neoplastic but are the most common (80%)
673
Q

What is the most common location for ectopic pancreas in the stomach

  1. antrum
  2. lesser curvature
  3. greater curvature
  4. fundus

Brant & Helms 3rd pg 822

A

antrum

674
Q

Which of the following is not true concerning gastritis

  1. erosive gastritis is the most common type and is caused by alcohol and NSAID
  2. hallmark of gastritis is thickened gastric folds and erosions
  3. crohn gastritis most commonly affects the antrum
  4. atrophic gastritis shows destruction of the fundic mucosa with antral sparing

Brant & Helms 3rd pg 823

A

erosive gastritis is the most common type and is caused by alcohol and NSAID

  • h. pylori gastritis is the MC type of gastritis
675
Q

Which is not true concerning the various types of gastritis?

  1. streptococci is the most common cause of phlegmonous gastritis
  2. E. coli and Clostridium welchii are common culprits in emphysematous gastritis
  3. eosinophilic gastroenteritis is associated w/ a cobblestone appearance of the mucosa
  4. Menetrier disease is characterized by decreased mucus production

Brant & Helms, 3rd p 823

A

Menetrier disease is characterized by decreased mucus production

  • Menetrier aka giant hypertrophic gastritis characterized by increased mucus production
676
Q

Which is false concerning gastrointestinal stromal tumors (GIST)

  1. MC mesenchymal tumor of the GI tract
  2. most arise in the stomach
  3. most are malignant
  4. calcification is common

Brant & Helms 3rd pg 820-1

A

most are malignant

  • only 10-30% are malignant
677
Q

Which is true concerning gastric ulcers

  1. 95% are malignant ulcers
  2. are multiple in 80% of patients
  3. defined as a full-thickness defect in the mucosa
  4. most benign ulcers are caused by NSAID use

Brant & Helms3rd pg 824

A

defined as a full-thickness defect in the mucosa

  • 95% are benign
  • 20% are multiple
  • most benign ulcers caused by h. pylori
678
Q

Which of the following is not a characteristic of a benign ulcer?

  1. demonstration of Hampton’s line
  2. smooth shoulder mound with tapering edges
  3. depth of ulcer less than width
  4. radiating folds extending into the crater

Brant & Helms3rd pg 825

A

depth of ulcer less than width

  • depth should be greater than width
679
Q

Brant & Helms3rd pg 837-8, 852-3

Which of the following is not a cause of gastric ulcers?

  1. Granulomatous disease
  2. Radiation
  3. Lymphoma
  4. Amyloidosis

Eisenberg 3rd ed. p.193

A

Amyloidosis

680
Q

Which of the following is false in regards to marginal ulcers?

  1. Seen after a Billroth-II anastomosis
  2. Located on the proximal side of the anastomosis
  3. Suggestive of malignancy
  4. Development of the ulcer within 2 weeks post-operative

Eisenberg 3rd ed. p.200,202

A

Located on the proximal side of the anastomosis

681
Q

The most common cause of Linitis Plastica is which of the following?

  1. Lymphoma
  2. Crohn’s disease
  3. Scirrhous carcinoma
  4. Radiation

Eisenberg 3rd ed. p.208

A

Scirrhous carcinoma

682
Q

A hallmark radiographic finding of Menetrier’s disease is which of the following?

  1. Benign gastric ulcer with radiating folds
  2. Enlarged rugal folds
  3. Multiple polypoid lesions through out the proximal stomach
  4. Linitis plastica

Eisenberg 3rd ed. p.234

A

Enlarged rugal folds

683
Q

A situation in which the terminal esophagus remains in its normal position, and the esophagogastric junction is situated above the diaphragm is known as?

  1. traumatic hernia
  2. hiatal hernia
  3. paraesophageal hernia
  4. foramen of Bochdalek hernia

Eisenberg, third edition p. 166

A

paraesophageal hernia

684
Q

The “double track” sign is associated with:

  1. annular pancreas
  2. antral mucosal diaphragm
  3. hypertrophic pyloric stenosis
  4. prolapsed antral mucosa

Eisenberg, third edition p. 297

A

hypertrophic pyloric stenosis

685
Q

Which of the following diseases produces gastric dilatation without outlet obstruction?

  1. renal colic
  2. carcinoma of the head of the pancreas
  3. amyloidosis
  4. annular pancreas

Eisenberg, third edition p. 300

A

renal colic

686
Q

Which of the following represents a complication after partial gastric resection with Billroth I or II anastomoses, resulting in markedly enlarged rugal folds, in which the stomach retains its pliability?

  1. gastric stump carcinoma
  2. suture granuloma
  3. gastric stump lymphoma
  4. bezoars

Eisenberg, third edition p. 280

A

gastric stump carcinoma

687
Q

What is the most common cause of gastric outlet obstruction?

  1. antral carcinoma
  2. peptic ulcer disease
  3. Crohn’s disease
  4. hypertrophic pyloric stenosis

Eisenberg, third edition p. 289

A

peptic ulcer disease

688
Q

The presence of gastric varices without esophageal varices has been thought to be a specific sign of:

  1. isolated splenic vein occlusion
  2. portal hypertension
  3. a short left gastric vein
  4. hepatic artery atherosclerotic plaquing

Eisenberg, third edition p. 238

A

isolated splenic vein occlusion

689
Q

Which of the following imaging characteristics are pathognomonic of a benign gastric ulcer?

  1. central location of the ulcer within the mound
  2. radiation of mucosal folds to the edge of the crater
  3. smooth, sharply delineated, gently sloping, and symmetrically convex tissue around the ulcer
  4. smooth, obtuse angle at which the margins of the mound join the adjacent normal gastric wall

Eisenberg, 3rd ed. p. 185

A

radiation of mucosal folds to the edge of the crater

690
Q

Extensive mucosal edema and lack of distensibility of the gastric wall, which may extend considerably beyond the limits of the ulcer itself is known as:

  1. Hampton line
  2. ulcer collar
  3. ulcer mound
  4. ulcer crater

Eisenberg, 3rd ed. p. 183

A

ulcer mound

691
Q

A Kirklin complex and Carman’s meniscus sign are diagnostic of:

  1. ruptured gastric ulcer
  2. Curling’s ulcer
  3. radiation-induced ulcer
  4. malignant gastric ulcer

Eisenberg, third edition p. 192

A

malignant gastric ulcer

692
Q

A thin, sharply demarcated lucent line with parallel straight margins at the base of an ulcer crater is called?

  1. Ulcer collar
  2. Carman’s line
  3. Hampton line
  4. crescent sign

Eisenberg, third edition p. 183

A

Hampton line

693
Q

Hypersecretory states that result in large amounts of retained gastric fluid despite fasting and lack of any organic obstruction is especially prominent in:

  1. Zollinger-Ellison syndrome
  2. Menetrier’s disease
  3. pediatric hypertrophic gastropathy
  4. eosinophilic gastritis

Eisenberg, third edition p. 233

A

Zollinger-Ellison syndrome

694
Q

What is the most common cause of discrete filling defects in the stomach?

  1. hyperplastic polyps
  2. adenomatous polyp
  3. leiomyoma
  4. hamartoma

Eisenberg, third edition p. 245

A

hyperplastic polyps

695
Q

Giant hypertrophic gastritis, characterized by massive enlargement of rugal folds due to hyperplasia and hypertrophy of the gastric glands is most commonly found in which portion of the stomach?

  1. lesser curvature
  2. greater curvature
  3. antrum
  4. pylorus

Eisenberg, third edition pg. 234

A

greater curvature

696
Q

Linitis plastica may occur as a result of:

  1. eosinophilic enteritis
  2. post-vagotomy
  3. pancreatitis
  4. scleroderma

Chapman, 3rd p. 218

A

eosinophilic enteritis

697
Q

Patients with massive localized rugal thickening along with hypoproteinemia and intestinal edema will often demonstrate which of the following lab findings?

  1. Positive carcinoembryonic antigen and anemia
  2. Increased alkaline phosphatase and anemia
  3. Eosinophilia and anemia
  4. Positive blood cultures
A

Eosinophilia and anemia

698
Q

An asymptomatic intramural filling defect seen in the stomach or first or second portion of the duodenum, with a central pit, is most likely…

  1. Distal gastric varices
  2. A malignant ulcer
  3. Pedunculated gastric polyp
  4. An aberrant pancreatic nodule
A

An aberrant pancreatic nodule

699
Q

Thickening, fibrosis, and a rigid antral wall which does not change shape during fluoroscopic examination probably represents…

  1. Extopic pancreatic rest
  2. Zollinger-Ellison syndrome
  3. Menetrier’s disease
  4. Schirrous carcinoma of the stomach
A

Schirrous carcinoma of the stomach

700
Q

A gastrinoma, an islet cell tumor of the pancreas, leads to excessive gastric acid production and the syndrome known as?

  1. Turcot syndrome
  2. Mastocytosis
  3. Zollinger-Ellison syndrome
  4. Mallory-Weiss syndrome
A

Zollinger-Ellison syndrome

701
Q

What is the most common location for gastric diverticulum to occur?

  1. Body
  2. Fundus
  3. Cardia
  4. Antrum
A

Fundus

702
Q

The incisura angularis seperates what two portions of the stomach

  1. fundus and body
  2. fundus and antrum
  3. antrum and pyloris
  4. body and antrum

Brant & Helms 3rd pg 816

A

body and antrum

703
Q

Which of the following is not true concerning h. pylori infection

  1. major cause of chronic gastritis, benign ulcers and gastric adenocarcinoma
  2. responsible for 70% of gastric peptic ulcers
  3. responsible for 50% of duodenal ulcers
  4. responsible for 50% of gastric adenocarcinomas

Brant & Helms 3rd pg 818

A

responsible for 50% of duodenal ulcers

  • accounts for 95% of duodenal ulcers
704
Q

Which is correct concerning gastric carcinoma

  1. polypoid masses and ulcerative masses account for 2/3 of cases
  2. most are squamous cell carcinomas
  3. prognosis is good with high five year survival
  4. most common in women

Brant & Helms3rd pg 818

A

polypoid masses and ulcerative masses account for 2/3 of cases

  • most are adenocarcinomas
  • five year survival is 10 to 20%
  • most common in 50-70 yo men
705
Q

Which morphological type of gastric carcinoma is associated with linitis plastica

  1. polypoid mass
  2. ulcerative mass
  3. scirrhous carcinoma
  4. superficial spreading

Brant & Helms 3rd pg 818

A

scirrhous carcinoma

706
Q

Which area of the stomach is the most common site of cancer

  1. antrum
  2. lesser curvature
  3. greater curvature
  4. cardia

Brant & Helms 3rd pg 819

A

cardia

  • cardia is the site of 44% of cancers
707
Q

Which of the following is not true concerning gastric lymphoma

  1. the stomach is the most common site of primary GI lymphoma
  2. most gastric lymphomas are non-hodgkins
  3. submucosal nodules may produce a bull’s eye appearance on UGI series
  4. poorer prognosis as compared to gastric carcinoma

Brant & Helms 3rd pg 820

A

poorer prognosis as compared to gastric carcinoma

  • much better prognosis for gastric lymphoma w/ 5yr survival rate 60-90%
708
Q

Intravesical dilatation of the ureter immediately proximal to its orifice in the bladder resembling a cobra head in appearance, is known as a (n):

  1. retrocaval ureter
  2. simple ureterocele
  3. ectopic ureterocele
  4. ureteral diverticula

P&J p.655

A

simple ureterocele

709
Q

The leading cause of ureteral injuries is:

  1. blunt trauma
  2. penetrating injury
  3. lumbar pedicle fracture
  4. iatrogenic trauma

Taveras, Ch 11, pg.9

A

iatrogenic trauma

710
Q

What is the most common benign ureteral tumor?

  1. fibroepithelial polyp
  2. leiomyoma
  3. papilloma
  4. hemangioma

P&J pg. 706

A

papilloma

711
Q

What is the most common cause of ureteral obstruction in the presence of orthotopic ureters?

  1. primary megaureter
  2. polyarteritis
  3. retroperitoneal fibrosis
  4. amyloidosis

P&J pg. 708

A

primary megaureter

  • responsible for up to 20% of cases of neonatal hudronephrosis
712
Q

Bergmen’s sign, a coiling of the catheter tip in the region of localized dilation during retrograde pyelography, is a diagnostic sign of:

  1. ureteral tumor
  2. ureteral calculi
  3. megaureter
  4. polyarteritis

P&J pg. 707

A

ureteral tumor

713
Q

Which of the following is associated with the drooping lily appearance?

  1. ureteral duplicaton
  2. bifid renal pelvis
  3. ureteropelvic junction obstruction
  4. retrocaval ureter

Brant & Helms3rd pg 889 caused

A

ureteral duplicaton

  • caused by obstructed upper pole system displacing lower pole system downward
714
Q

True or False

The Weigert-Meyer rule states that with complete ureteral duplication, the ureter draining the lower pole passes through the bladder wall to insert inferior and medial to the normally placed ureter that drains the upper pole.

Brant & Helms 3rd pg 888

A

FALSE

  • The Weigert-Meyer rule states that with complete ureteral duplication, the ureter draining the lower pole passes through the bladder wall to insert inferior and medial to the normally placed ureter that drains the upper pole.
  • Vowels obstruct (Upper-Obstructs)
  • Constants reflux (Lower-Refluxes)
715
Q

Which of the following is not true concerning ureteropelvic junction obstruction

  1. it is a congenital anomaly that may go undiagnosed until adulthood
  2. bilateral in 30% of cases
  3. the ureter is frequently dilated
  4. an aberrant renal vessel cause the obstruction in 15 to 20% of cases

Brant & Helms 3rd pg 889

A

the ureter is frequently dilated

  • ureter is not dilated in uteropelvic junction obstruction
716
Q

Which structure persist to produce a retrocaval ureter

  1. right subcardinal vein
  2. right supracardinal vein
  3. left subcardinal vein
  4. left supracardinal vein

Brant & Helms 3rd pg 889

A

right subcardinal vein

  • S-type, fish-hook, or J-type deformity of ureter can present
  • site of narrowing/obstruction (if present) is proximal to the vena cava at the lateral edge of the psoas and caused by pressure of the retroperitoneal fascia over the muscle
  • aka Circumcaval ureter
717
Q

Which of the following is not true concerning nephrolithiasis

  1. nearly 25% of the population will form a renal stone in their lifetime
  2. 80% of renal calculi are radiopaque on plain film
  3. struvite is the most common component of staghorn calculi
  4. nearly all stones are opaque on noncontrast CT

Brant & Helms 3rd pg 889

A

nearly 25% of the population will form a renal stone in their lifetime

  • only 10% will form a renal stone in their lifetime
718
Q

Which of the following is not a normal point of ureteral narrowing

  1. ureteropelvic junction
  2. ureterovesical junction
  3. where the ureters cross the iliac arteries
  4. where the ureters cross the inferior vena cava

Brant & Helms 3rd pg 888

A

where the ureters cross the inferior vena cava

  • ureters don’t normally cross the IVC
719
Q

Which of the following is the most common cause of filling defects on a contrast-filled collecting systems ?

  1. Blood clots
  2. Calculi
  3. Transitional cell carcinoma
  4. Metastases

Brant & Helms,3rd pg 893

A

Calculi

720
Q

Which of the following causes of filing defects in the pelvicaliceal system** and **ureter is not correctly matched with its unenhanced CT attenuation value

  1. Calculi-100 HU
  2. Blood clots - < 50 HU
  3. Transitional cell carcinoma-8 to 30 HU
  4. all of the above are correctly matched

Brant & Helms 3rd pg 893-4

A

all of the above are correctly matched

721
Q

Which of the following is correct concerning the age/sex distribution of transitional cell carcinoma of the ureters and renal pelvis

  1. men age 40 to 60
  2. men age 60+
  3. women age 40 to 60
  4. women age 60+

Brant & Helms 3rd pg 894

A

men age 60+

722
Q

Multiple small, smooth, round filling defect in the ureter are characteristic for which of the following

  1. transitional cell carcinoma
  2. squamous cell carcinoma
  3. fibroepithelial polyp
  4. pyeloureteritis cystica

Brant & Helms 3rd pg 895

A

pyeloureteritis cystica

723
Q

Which of the following is considered premalignant in the ureter ?

  1. malacoplakia
  2. leukoplakia
  3. fibroepithelial polyp
  4. none of the above
A

none of the above

  • Leukoplakia is considered premalignant in the bladder but not in the ureter

Brant & Helms 3rd pg 895-6

724
Q

The proximal 2 cm of the bulbous urethra is called:

  1. verumontanum
  2. pars nuda
  3. utricle
  4. none of the above

Brant & Helms 3rd pg 905

A

pars nuda

725
Q

Which of the following is not true about urethral strictures

  1. short-segment strictures are usually traumatic
  2. long-segment strictures are associated with inflammatory response
  3. most inflammatory strictures are attributable to chlamydia
  4. carcinoma of the urethra is a potential complication of urethral stricture

Brant & Helms 3rd pg 906-7

A

most inflammatory strictures are attributable to chlamydia

  • Most inflammatory strictures are due to gonorrhea
726
Q

What is the most common complication of urethral stricture?

  1. periurethral abscess
  2. false passage
  3. stasis and infection
  4. carcinoma of the urethra

Brant & Helms 3rd pg 907

A

false passage

  • usually iatrogenic
  • complications of urthera injury include strictures and impotence
  • Weissleder pg. 229 has great diagram of the types of urethral injuries by types 1-3.
  • Current and widely accepted classification is by Goldman nad is categorized into 5 types
727
Q

Regarding carcinoma of the urethra, match the most common cell type with the most common location

  1. squamous cell carcinoma, posterior urethra
  2. squamous cell carcinoma, anterior urethra
  3. adenocarcinoma, posterior urethra
  4. adenocarcinoma, anterior urethra

Brant & Helms 3rd pg 907

A

squamous cell carcinoma, anterior urethra

728
Q

What percentage of pelvic fractures have accompanying injury to the posterior urethra

  1. 10%
  2. 20%
  3. 40%
  4. 80%

Brant & Helms 3rd pg 907

A

10%

729
Q

What is the most common site of injury to the urethra

  1. bladder and prostatic urethral junction
  2. prostatic and membranous urethral junction
  3. membranous and bulbous urethral junction
  4. bulbous and penile urethral junction

Brant & Helms 3rd pg 908

A

prostatic and membranous urethral junction

730
Q

A rupture of the membranous urethra at the apex of the prostate with extravasation above the urogenital diaphragm represents which type of posterior urethral injury

  1. Type I
  2. Type II
  3. Type III
  4. Type IV

Brant & Helms 3rd pg 908

A

Type II

there is no type IV

  • type 1: stretch injury with pelvic hematoma
  • type 3: rupture of both membranous and bulbous urethra with extravasation above and below the urogenital diaphragm
  • Note the classification system has been upgraded by Goldman.
731
Q

A straddle injury most commonly affects which portion of the urethra

  1. prostatic
  2. membranous
  3. bulbous
  4. penile

Brant & Helms 3rd pg 908

A

bulbous

732
Q

Infection, vesicoureteral reflux, hydronephrosis, enuresis, and bladder distension are signs and symptoms found almost exclusively in male patients, most often in children, and are associated with:

  1. urethral diverticula
  2. urethral tumor
  3. posterior urethral valves
  4. urethral calculi

P&J pg. 718

A

posterior urethral valves

733
Q

A pelvic soft tissue mass seen in a female patient with mottled calcifications and smoothly rounded and lobulated borders is most suggestive of what?

  1. Hydatid mole
  2. Endometrial carcinoma
  3. Leiomyoma
  4. Adnexal cyst
A

Leiomyoma

734
Q

Opacification of the uterine cavity and fallopian tubes by injection of opaque contrast material into the uterus followed by the injection of a water-soluble iodinated contrast material sufficient to opacify the endometrial cavity is known as:

  1. endovaginal scan
  2. hysterosalpingography
  3. spin-echo T2-weighted MRI
  4. excretory urography
A

hysterosalpingography

735
Q

Match the polyposis condition with the appropriate type of polyp

peutz-jeghers syndrome

  1. hamartomatous

cronkhite-canada syndrome

  1. inflammatory

gardner syndrome

  1. hyperplastic

juvenile GI polyposis

  1. adenomatous

turcot syndrome

cowden disease

A